Health Assessment in Nursing Weber & Kelley 5th edition Ch 1-34

¡Supera tus tareas y exámenes ahora con Quizwiz!

12. A client complains of scrotal pain, and the nurse elicits a positive Prehn sign, in which passive elevation of the testes relieves the scrotal pain. The nurse should refer the client for treatment of which of the following? A) Strangulated hernia B) Tortuous varicocele C) Epididymitis D) Scrotal mass

C) Epididymitis

8. The nurse is assessing a client whose electronic health record notes a diagnosis of esotropia. When examining this client, the nurse should expect what finding? A) Eye turning outward B) Eye malalignment C) Eye turning inward D) Eye oscillating

C) Eye turning inward

15. A nurse has completed an assessment of a client with cholecystitis and is about to document the findings. Which statement best reflects accurate documentation? A) Client appears upset about upcoming surgery. B) Client was interviewed about previous history of hypertension. C) Skin pale, warm, and dry without evidence of lesions. D) Client's oral intake is satisfactory.

C) Skin pale, warm, and dry without evidence of lesions.

18. The nurse is assessing the apices of the client's lungs. The nurse should locate them at which position? A) At the level of the diaphragm B) Near the level of the eighth rib C) Slightly above the clavicle D) At about the tenth rib

C) Slightly above the clavicle

16. The emergency department (ED) nurse is assessing for kidney tenderness in a client who has presented with complaints of dysuria and back pain. What assessment technique should the nurse utilize? A) Deep palpation B) Indirect percussion C) Moderate palpation D) Blunt percussion

D) Blunt percussion

3. A hospital nurse has identified a need to improve her critical thinking skills in an effort to improve client care. The nurse should identify which of the following characteristics of critical thinking? A) It is an innate skill that some individuals possess and which others do not. B) It does not include past experiences. C) It is based primarily on getting correct and timely information. D) It involves reflections on thoughts before reaching conclusions.

D) It involves reflections on thoughts before reaching conclusions.

19. Assessment of a client's skin reveals several individual and distinct 2-mm lesions on the client's back. The nurse would document the configuration as which of the following? A) Discrete B) Linear C) Annular D) Confluent

A) Discrete

19. A nurse is working on an acute neurological unit. Which assessment form would the nurse most likely use to document assessment data? A) Open-ended form B) Focused assessment form C) Frequent assessment form D) Ongoing assessment form

B) Focused assessment form

24. A nurse is using the Katz Activities of Daily Living tool to assess an older adult's functional status. What question will the nurse include in this assessment? A) Who generally prepares your meals and snacks? B) Do you require any assistance when showering or bathing? C) Do you feel like you have enough support from your family? D) Are you able to shop for your own groceries?

B) Do you require any assistance when showering or bathing?

18. A nurse is reviewing the various causes associated with abdominal distention. Which of the following should the nurse identify? Select all that apply. A) Fat B) Stool C) Gas D) Hernia E) Fibroid tumors

A) Fat B) Stool C) Gas E) Fibroid tumors

11. A nurse is providing in-service training to a group of nurses in a facility that has just begun to use an integrated cued checklist for documentation. Which of the following would the nurse identify as a major advantage of this type of documentation? A) It helps nurses to cluster assessment data. B) It provides lines for the nurses' comments. C) It includes specialized data particular to each client. D) It standardizes data collection.

A) It helps nurses to cluster assessment data.

8. A nurse determines that the liver span of an older adult male client measures 6 cm. The nurse would interpret this as indicating which of the following? A) It is a normal-sized liver. B) The liver is larger than normal. C) It is a smaller-than-normal liver. D) The liver has atrophied.

A) It is a normal-sized liver.

10. A group of students is reviewing the events associated with the cycle of violence. Which statement by the students demonstrates understanding of the topic? A) It occurs in a predictable pattern. B) There are five phases in the cycle. C) During phase 2, the victim can often stop the abuser. D) Criticism is the defining part of phase 3.

A) It occurs in a predictable pattern.

5. A nurse is admitting a client to a long-term care facility. In order to elicit reliable and valid data during the spiritual assessment, the nurse understands that the focus must be on which of the following? A) Objectivity when performing the assessment B) Sharing a common religious affiliation with the client C) Repeating the assessment in several weeks D) Providing spiritual interventions prior to assessment

A) Objectivity when performing the assessment

26. A nurse has completed the assessment of a client's direct pupillary response and is now assessing consensual response. This aspect of assessment should include which of the following actions? A) Observing the eye's reaction when a light is shone into the opposite eye B) Shining a light into one eye while covering the other eye with an opaque card C) Moving a finger into the client's peripheral vision field and asking the client to state when he or she sees the finger D) Comparing the difference between the client's dilated pupil and a constricted pupil

A) Observing the eye's reaction when a light is shone into the opposite eye

1. The nurse is planning a community assessment using the Community as a Partner model. The nurse will prioritize which of the following when utilizing this model? A) People of the community B) History of the community C) Demographics of the community D) Health of the community

A) People of the community

4. Assessment reveals that an older adult client has osteomalacia. Which of the following would be most important to include in the client's teaching plan? A) Practice risk prevention for fractures. B) Keep exercise to a minimum to decrease pain. C) Minimize movements to maintain joint stability. D) Treat secondary arthritis proactively.

A) Practice risk prevention for fractures.

16. A nurse is assessing a client who is exhibiting decorticate posturing. Which of the following would the nurse observe? A) Extended upper extremities B) Internally rotated lower extremities C) Pronated forearms D) Flexed hands at the side of the body

B) Internally rotated lower extremities

20. The nurse has assessed the thorax and lungs of an elderly client, as well as reviewing the results of lung function testing. Which of the following findings should the nurse attribute to possible pathology rather than expected, age-related changes? A) Respiratory rate of 30 breaths per minute B) Decreased vital capacity C) Increased residual volume D) Presence of a slight barrel chest

A) Respiratory rate of 30 breaths per minute

30. An adult client has sought care at the clinic, stating that she believes she has a raging yeast infection. The nurse would expect to assess what type of vaginal discharge? A) Thick, white vaginal discharge B) Copious clear, foul-smelling discharge C) Yellowish discharge with a metallic odor D) Blood-tinged vaginal discharge

A) Thick, white vaginal discharge

4. During the health history, a client describes recent episodes of intermittent facial pain lasting several minutes. The nurse should recognize that this complaint is suggestive of what health problem? A) Trigeminal neuralgia B) Migraine headache C) Meningitis D) Temporomandibular joint dysfunction

A) Trigeminal neuralgia

21. The pediatric nurse is obtaining the nursing history of a 4-year-old girl who is accompanied by her mother. What question should the nurse pose to the child's mother? A) Is your daughter able to pick out her name from a page of writing? B) Do you think your daughter can see others' points of view? C) Does your daughter often ask 'why'? D) Does your daughter like to collect things?

C) Does your daughter often ask 'why'?

14. When assessing the temperature of the feet of an older client with diabetes, the nurse would use which part of the hand to obtain the most accurate assessment data? A) Finger pad surface B) Palmar hand surface C) Dorsal hand surface D) Ulnar hand surface

C) Dorsal hand surface

25. Assessment of an adult female client's face reveals a moon shape, increased hair distribution, and a reddened tone to the client's cheeks. What collaborative problem is most clearly suggested to the nurse by these assessment data? A) RC: Thyroid crisis B) RC: Cerebrovascular accident C) RC: Cushing's syndrome D) RC: Acromegaly

C) RC: Cushing's syndrome

20. A nurse on the hospital's subacute medical unit is planning to perform a client's focused assessment. Which of the following statements should inform the nurse's practice? A) The focused assessment should be done before the physical exam. B) The focused assessment replaces the comprehensive database. C) The focused assessment addresses a particular client problem. D) The focused assessment is done after gathering subjective data.

C) The focused assessment addresses a particular client problem.

14. As part of a mental status assessment, the nurse asks a client to draw the face of a clock. This will allow the nurse to assess which of the following domains of mental status? A) Concentration and orientation B) Perceptions and thought processes C) Visual perceptual and constructional ability D) Expressions and feelings

C) Visual perceptual and constructional ability

1. The nurse is conducting a functional assessment of an older adult client. The nurse should focus questions on which area? A) Feelings about aging B) Quality of life C) Recent personal losses D) Activities of daily living

D) Activities of daily living

16. An emergency department nurse asks a client to complete an intimate partner violence assessment screening. How should the nurse best explain the rationale for this assessment? A) We are required by law to ask you these questions. B) We routinely screen everyone because violence affects so many people. C) We don't think you're abused but we have to ask. D) This is just something we need to do for reimbursement.

B) We routinely screen everyone because violence affects so many people.

5. Which of the following client situations would the nurse interpret as requiring an emergency assessment? A) A pediatric client with severe sunburn B) A client needing an employment physical C) A client who overdosed on acetaminophen D) A distraught client who wants a pregnancy test

C) A client who overdosed on acetaminophen

18. The nurse is preparing to palpate the anatomic snuffbox. At which location would the nurse palpate? A) At the anterior area of the sternoclavicular joint B) At the posterior temporomandibular joint C) At the olecranon process of the elbow D) At the back of the wrist and extended thumb

D) At the back of the wrist and extended thumb

5. A nurse is preparing to assess a client's cerebellar function. Which of the following aspects of neurological function should the nurse address? A) Remote memory B) Sensation C) Judgment D) Balance

D) Balance

24. A nurse is a preparing to assess a male client's anus and rectum. How should the nurse best prepare the client for this assessment? A) Ask the client if he is feeling anxious or fearful about the exam. B) Assist the client into the supine position. C) Administer a dose of analgesia 15 minutes before the exam. D) Position the client in a left side-lying position.

D) Position the client in a left side-lying position.

15. When describing the expansion of the depth and scope of nursing assessment over the past several decades, which of the following would the nurse identify as being the primary force? A) Documentation B) Informatics C) Diversification D) Technology

D) Public mistrust of physicians

5. The nurse is performing an otoscopic examination of an infant's ears. Which of the following actions should the nurse do? A) Pull the pinna forward and down. B) Pull the pinna up and back. C) Pull the pinna straight back. D) Pull the pinna down and back.

D) Pull the pinna down and back.

13. A nurse is auscultating a client's heart sounds. What action should the nurse perform during this assessment? A) Start by auscultating the client's breath sounds. B) Auscultate prior to inspection and palpation. C) Use the bell rather than the diaphragm. D) Systematically listen to the entire precordium.

D) Systematically listen to the entire precordium.

15. A nurse is reviewing a client's electrocardiogram (ECG). The nurse should identify which component as indicating ventricular repolarization? A) P wave B) QRS complex C) ST segment D) T wave

D) T wave

5. The nurse is conducting an assessment of an older adult client who has a diagnosis of chronic heart failure. How can the nurse best assess the effects of the client's stroke volume? A) Take the blood pressure while the client is standing. B) Measure the strength of the radial pulse. C) Add the radial pulse and the systolic blood pressure. D) Calculate the difference between the diastolic and systolic pressures.

D) Calculate the difference between the diastolic and systolic pressures.

27. A nurse is performing transillumination as part of the assessment of a client's swollen scrotum. What finding constitutes a normal scrotum? A) The testes transilluminate, but the other regions of the scrotum do not. B) Transillumination of the scrotum results in a pale yellow or white glow. C) Transillumination of the scrotum results in a red glow. D) Contents of the scrotum do not transilluminate.

D) Contents of the scrotum do not transilluminate.

6. The nurse is assessing an adolescent client and notes 45-degree flexion of the cervical spine. What is the nurse's most appropriate action? A) Facilitate a referral for medical follow up. B) Palpate the spinous processes. C) Perform the LasËgue test. D) Continue the exam because this curve is normal.

D) Continue the exam because this curve is normal.

15. A nurse who provides care on a medical unit utilizes the Alcohol Use Disorders Identification Test (AUDIT) as part of the standard admission protocol. After obtaining a score of 9 from a recently admitted client, the nurse should recognize the possibility of which of the following? A) Hazardous and harmful alcohol use B) Imminent liver disease C) Acute pancreatitis D) Alcoholism

A) Hazardous and harmful alcohol use

15. A nurse has completed the assessment of a client's breasts. The nurse should suspect that the client has fibroadenomas based on which findings? A) Lobular, ovoid, or round lesions B) Irregular, firm cysts C) Round, defined mobile cysts D) Nondefined, mobile cysts

A) Lobular, ovoid, or round lesions

7. A nurse is having difficulty eliciting a patellar reflex during a client's neurological assessment. Which of the following would be most appropriate for the nurse to have the client do? A) Lock the fingers together and pull against each other. B) Clench the jaw tightly. C) Squeeze a thigh with the opposite hand. D) Stretch the arms over head.

A) Lock the fingers together and pull against each other.

5. The nurse is presenting a program about sexually transmitted infections, including HIV, to a group of young men. The nurse would include which of the following as the having the highest incidence of HIV infection in the United States? A) Men having sex with men B) Heterosexual partners C) Bisexual individuals D) Intravenous drug users

A) Men having sex with men

16. When testing the range of motion of the cervical spine, the nurse notes impaired range of motion and neck pain. A review of the client's history reveals fever, chills, and headache. Which of the following would the nurse suspect? A) Meningitis B) Cervical strain C) Compression fracture D) Cervical disc degeneration

A) Meningitis

3. A group of students is reviewing the structures of the heart, noting that the thickest layer of the heart is made up of contractile muscle cells. The students are correct in identifying this layer as which of the following? A) Myocardium B) Epicardium C) Endocardium D) Pericardium

A) Myocardium

6. The nurse is preparing to assess an older adult client's near vision. Which of the following pieces of equipment would be most appropriate for the nurse to use? A) Newspaper B) Snellen chart C) Ophthalmoscope D) Penlight

A) Newspaper

14. Which of the following findings should the nurse document after assessing the thyroid gland of an older adult without abnormalities? A) Nodularity B) Tenderness C) Enlargement D) Bruits

A) Nodularity

23. During the nursing history of a newly admitted client, the nurse is reviewing a client's current medication regimen. What medication category creates a risk for decreased bone density? A) Beta-adrenergic blockers B) Corticosteroids C) Nonsteroidal anti-inflammatories (NSAIDs) D) Calcium channel blockers

B) Corticosteroids

10. A group of students is reviewing material on cultural competence. The students demonstrate understanding of this concept when they identify which of the following as the starting point? A) Cultural awareness B) Cultural desire C) Cultural skill D) Cultural knowledge

B) Cultural desire

10. While assessing the scrotum of an adult client, the nurse notes thin and rugated scrotal skin with little hair dispersion. The nurse interprets this finding as which of the following? A) Reiter's syndrome B) Normal findings C) Effects of chemotherapy D) Gonorrhea

B) Normal findings

15. A client has been admitted following an unexplained weight loss of 15 pounds over the past 3 months. How should the nurse best assess the subjective component of the client's nutritional status? A) Ask the client to explain MyPlate. B) Obtain a 24-hour diet recall. C) Ask about the contents of one typical meal. D) Elicit the client's favorite foods.

B) Obtain a 24-hour diet recall.

17. While performing the initial assessment of a client, the client tells the nurse that this is his first hospitalization and that he has no previous surgeries. The nurse should document which of the following? A) Client denies prior hospitalizations and surgeries B) Client has not been hospitalized before nor has he had any surgery C) Client answered no to previous hospitalizations or surgery D) Negative for past hospitalizations

A) Client denies prior hospitalizations and surgeries

8. A nurse is palpating the head and neck of a newly referred client. Which of the following would the nurse suspect if assessment reveals that the client's skull and facial bones are larger and thicker than normal? A) Acromegaly B) Brain tumor C) Paget disease D) Parkinson disease

A) Acromegaly

2. During a prenatal class, a participant says that she was told that her breasts are not large enough to breastfeed. When responding to this client, the nurse should understand that the functional capacity of the breast is primarily determined by which of the following variables? A) Amount of glandular tissue B) Breast size and weight C) Amount of fatty tissue D) Depth of the subcutaneous fat layer

A) Amount of glandular tissue

4. A 55-year-old client is being evaluated for a suspected hearing impairment. Which of the nurse's health interview questions is most likely to yield relevant data? A) Are you having difficulty hearing high-frequency sounds? B) Do you notice any drainage from your ears? C) Are you experiencing any pain in your ears? D) Have you felt any popping sensations in your ears?

A) Are you having difficulty hearing high-frequency sounds?

4. An older adult client presents with cramping-type leg pain when walking, which is relieved by rest. The client also has cool, pale feet and capillary refill in the toes of 4 to 6 seconds. Which of the following would the nurse suspect? A) Arterial insufficiency B) Musculoskeletal weakness C) Venous insufficiency D) Diabetic neuropathy

A) Arterial insufficiency

10. Which technique should the nurse use to perform scoliosis screening in a school-age child? A) Have the child bend forward at the waist. B) Measure the length of each of the child's legs. C) Measure the distance between the child's knees and ankles. D) Ask the child to walk across the room.

A) Have the child bend forward at the waist.

15. After teaching a group of students about families and family assessment, the instructor determines that the teaching was successful when the students describe family as which of the following? A) Two individuals who are related by marriage B) Whoever the family says they are C) Two or more individuals residing together D) Individuals connected by time and blood

B) Whoever the family says they are

2. A nurse is interpreting and validating information from an older adult client who has been experiencing a functional decline. The nurse is in which phase of the interview? A) Introductory B) Working C) Summary D) Closing

B) Working

6. The nurse assesses brisk reflexes in a client during a neurological assessment. The nurse should document this finding as which of the following? A) 1+ B) 2+ C) 3+ D) 4+

C) 3+

15. A nurse in the intensive care unit is calculating an acutely ill client's 24-hour fluid balance. The nurse should include insensible fluid losses of what volume when performing this assessment? A) 100 to 300 mL B) 450 to 650 mL C) 800 to 1000 mL D) 1200 to 1400 mL

C) 800 to 1000 mL

3. A nurse shines a light into one of the client's eyes during an ocular exam and the pupil of the other eye constricts. The nurse interprets this as which of the following? A) Direct reflex B) Optic chiasm C) Consensual response D) Accommodation

C) Consensual response

7. Which of the following would the nurse do first when collecting subjective data from a client when domestic violence is suspected? A) Discuss legal reporting requirements. B) Talk about limits of confidentiality. C) Create a safe environment. D) Have a witness to the conversation.

C) Create a safe environment.

1. When examining a newborn male infant, the nurse notes that neither testicle is descended. The nurse documents this finding as which of the following? A) Epididymitis B) Orchitis C) Cryptorchidism D) Varicocele

C) Cryptorchidism

5. A nurse suspects intra-abdominal bleeding in a client who was recently involved in a motor vehicle accident. Which finding would most likely lead the nurse to this suspicion? A) Tenderness on palpation B) Diastasis recti C) Cullen's sign D) Tympany on percussion

C) Cullen's sign

12. The nurse attends a Native-American Alcoholic Anonymous support group and develops close relationships with three group members. The nurse is demonstrating which of the following? A) Cultural desire B) Cultural awareness C) Cultural encounter D) Cultural knowledge

C) Cultural encounter

18. An elderly client's history reveals the use of antihistamines. When inspecting the client's mouth, which of the following would the nurse expect to find? A) Resorption of the gum ridge B) Swollen, red tongue C) Decreased saliva production D) Pocketing of food

C) Decreased saliva production

7. When auscultating the heart of an elderly client, the nurse detects a soft systolic murmur at the base of the heart. The nurse understands that this is most likely the result of which of the following? A) Calcification of the aortic and mitral valves B) Accumulation of amyloid in the pacemaker cells C) Enlargement of the heart muscle D) Regurgitation through a stenotic valve

A) Calcification of the aortic and mitral valves

22. A nurse is providing health education about osteoporosis to a community group. What ethnicity is considered to be an independent risk factor for osteoporosis? A) Caucasian B) African American C) South Asian D) Native American

A) Caucasian

5. A nurse is reviewing the laboratory test results of an adult client who has numerous chronic health challenges. Which assessment result would alert the nurse to potential malnutrition? A) Hemoglobin of 13.1 g/dL B) Hematocrit of 40% C) Serum albumin of 2.6 g/dL D) Total protein of 7 g/dL

C) Serum albumin of 2.6 g/dL

22. The nurse is using the mnemonic "COLDSPA" to assess a client's complaint of lower abdominal pain. The nurse asks the client to rate the pain on a scale of 0 to 10. The nurse is assessing which aspect of the complaint? A) Character B) Onset C) Severity D) Pattern

C) Severity

7. A nurse is assessing a community's environmental protection. The nurse should address which of the following? A) Major local employers B) Local law enforcement agencies C) Sewage treatment facilities D) Educational facilities

C) Sewage treatment facilities

2. A group of students is reviewing information about pain transmission and the fibers involved. The students demonstrate understanding when they state that A-delta primary afferent fibers transmit pain that is felt as which of the following? A) Burning B) Throbbing C) Sharp D) Aching

C) Sharp

15. What action on the part of a middle-aged client would best exemplify Erikson's concept of generativity? A) Being able to accurately evaluate the merits of others' ideas B) Emphasizing the importance of one's knowledge and skill set C) Consistently increasing one's income D) Guiding and mentoring individuals who are younger

D) Guiding and mentoring individuals who are younger

19. The nurse has completed a focused assessment of a client's mouth, nose, and throat. Which of the following findings would a nurse interpret as being normal? A) Absence of red glow on transillumination of sinuses B) Nasal mucosa pale pink and swollen C) Tonsils 2+ D) Pinkish, spongy soft palate

D) Pinkish, spongy soft palate

14. The nurse is collecting data from a client about his nutrition. Which of the following would the nurse document as objective data? A) Client states he is not eating well. B) Client complains of nausea and vomiting. C) Clients experiences urinary frequency. D) Tenting of client's skin observed upon skin pinch.

D) Tenting of client's skin observed upon skin pinch.

3. A group of community nurses are reviewing the theories associated with abuse. The nurses are addressing the psychopathology theory when they identify that violence results from which of the following? A) The abuser's need to control others B) The abuser's desire to show authority C) The abuser's need to be in charge D) The abuser's personality disorder

D) The abuser's personality disorder

27. The nurse is completing a comprehensive nutritional assessment and has assessed and documented the client's triceps skin fold thickness (TSF) using calipers. This assessment finding allows the nurse to determine which of the following? A) The client's ratio of muscle to adipose tissue B) The client's body mass index C) The client's proportion of muscle mass D) The amount of the client's subcutaneous fat stores

D) The amount of the client's subcutaneous fat stores

5. During palpation of a young child's abdomen, the nurse assesses the liver. Which of the following would the nurse expect to find? A) The liver can be palpated 4 cm below the right costal margin. B) The liver is not palpable. C) The liver is found at the left costal margin. D) The liver is located 2 cm below the right costal margin.

D) The liver is located 2 cm below the right costal margin.

8. A client's medical assessment reveals no heart disease. An electrocardiogram is performed and a dysrhythmia is noted. The nurse interprets this finding as most likely reflecting which of the following age-related changes? A) Decreased ventricular compliance B) Peripheral vascular disease C) Widening pulse pressure D) Collagen deposits around pacemaker cells

D) Collagen deposits around pacemaker cells

13. When assessing a client's deep tendon reflexes, which technique would be most appropriate for the nurse to use? A) Use the blunt end of the reflex hammer to strike a smaller area. B) Strike the area slowly and methodically. C) Hold the reflex hammer between the thumb and index finger. D) Percuss the area of the tendon to be struck for the reflex.

C) Hold the reflex hammer between the thumb and index finger.

11. An emergency department nurse has utilized the Confusion Assessment Method (CAM) in the assessment of a 79-year-old client with a new onset of urinary incontinence. This assessment tool will allow the nurse to confirm the presence of what health problem? A) Delirium B) Vascular dementia C) Schizophrenia D) Psychosis

A) Delirium

2. A nurse is preparing to assess an adult client's carotid pulses. Which of the following actions would be contraindicated? A) Asking the client to flex his or her neck B) Compressing the arteries bilaterally C) Performing the examination while the client is seated D) Asking the client to swallow water

B) Compressing the arteries bilaterally

3. A nurse is attempting to view a whole family as a unit to obtain a view of the family composition. How can the nurse best achieve this goal? A) Compile a list of family members. B) Construct a family genogram. C) Determine the internal power structure. D) Identify communication patterns.

B) Construct a family genogram.

20. While inspecting the lower extremities of a client, the nurse observes an ulcer. Which of the following would lead the nurse to suspect that the ulcer is the result of arterial insufficiency? Select all that apply. A) Irregular border B) Deep C) Circular in shape D) Moderate leg edema E) Client report of severe pain

B) Deep C) Circular in shape E) Client report of severe pain

9. A client's history suggests a need to assess eye muscle strength and cranial nerve function. What assessment should the nurse consequently perform? A) Corneal light reflex test B) Eye positions test C) Cover test D) Visual fields test

B) Eye positions test

12. The nurse is assessing the skin of a 12-hour-old infant. Which assessment finding would be cause for concern? A) Milia B) Jaundice C) Erythema toxicum D) Mongolian spot

B) Jaundice

26. The nurse is completing a head-to-toe assessment of a newborn infant. How should the nurse determine if the infant's anus is patent? A) Spread the infant's buttocks to facilitate inspection. B) Observe for the passage of meconium. C) Insert a gloved finger 0.5 to 1 cm into the rectum. D) Auscultate for bowel sounds to all four abdominal quadrants.

B) Observe for the passage of meconium.

10. A nurse is presenting a class for new parents about infant care. To decrease the risk of sudden infant death syndrome, the nurse should encourage parents to place their sleeping infants in what position? A) Prone B) Supine C) High Fowler's D) Low Fowler's

B) Supine

13. The nurse is preparing to assess the peripheral pulses of a client. The nurse should place the client in which position? A) Sitting upright B) Supine C) Sims position D) Prone

B) Supine

27. A nurse is performing a head and neck assessment of a client who is newly admitted to the hospital unit. When preparing to assess the client's thyroid gland, what landmarks should the nurse first identify? Select all that apply. A) Sternocleidomastoid muscle B) Hyoid bone C) Cricoid cartilage D) Carotid artery E) Esophagus

B, C B) Hyoid bone C) Cricoid cartilage

26. A nurse is beginning a community assessment and needs to access the age and gender characteristics of residents. What data source should the nurse consult? A) Census data B) Local birth records C) School enrollment listings D) Local archives

A) Census data

12. During a family assessment, the nurse closely observes family interactions for which main reason? A) To determine if members support and nurture one another B) To analyze each family member's response to stress C) To provide feedback on the family's communication problems D) To determine the transmission of beliefs and values

A) To determine if members support and nurture one another

25. A nurse health promotion teaching is focusing on hygiene and the prevention of illness. When instructing clients how to clean their ears, what action should the nurse recommend? A) Washing with a warm, moist washcloth B) Gently irrigating with normal saline C) Cleaning with cotton-tipped applicator D) Irrigating with mildly soapy water

A) Washing with a warm, moist washcloth

21. A client reports that she has been pregnant four times, had two babies born at term, no preterm births, two spontaneous abortions, and has two living children. The nurse should document the client's gravida and para status as which of the following? A) G2 P4202 B) G4 P2022 C) G2 P4044 D) G4 P4220

B) G4 P2022

28. Inspection of a client's foot reveals an enlarged, painful, inflamed bursa (bunion) on the medial side of the foot. The nurse should make a referral for what health problem? A) Osteomalacia B) Hallux valgus C) Pes planus D) Gouty arthritis

B) Hallux valgus

7. The nurse is percussing a client's liver and is assessing liver descent. The nurse should have the client do which of the following? A) Cough forcefully B) Hold the breath C) Breathe in and out deeply D) Perform the Valsalva maneuver

B) Hold the breath

15. The nurse is gathering data about home health services in a community. The nurse would evaluate the community for which types of services? Select all that apply. A) Diagnostic services B) Homemaker services C) Nutritional consultation D) Substance abuse services E) Skilled care

B) Homemaker services C) Nutritional consultation E) Skilled care

26. The nurse has placed her hands behind the client's head and flexed the client's neck forward as far as the client can tolerate. During the test, the client experiences leg pain and bends his knees. This assessment finding is suggestive of what health problem? A) Ischemic stroke B) Meningitis C) Bell's palsy D) Brain stem lesion

B) Meningitis

1. A nurse has completed a comprehensive assessment of a client and has begun the process of data analysis. Data analysis should allow the nurse to produce which of the following direct results? A) Outcomes evaluation B) Nursing diagnoses C) Holistic interventions D) An interdisciplinary plan of care

B) Nursing diagnoses

15. A nurse has collected extensive data relating to a client's spirituality. Which type of data would the nurse need to validate the information obtained during this assessment? A) Subjective data B) Objective data C) Informal data D) Formal data

B) Objective data

1. A nurse is completing the intake assessment of an older adult who has just relocated to a long-term care facility. Which of the following nursing actions would be most important to ensure accurate data when gathering the resident's information? A) Documenting the data B) Validating the data C) Identifying client support systems D) Determining client needs

B) Validating the data

17. A nurse auscultates a client's heart rate and rhythm and finds the rhythm to be irregular. Which of the following should the nurse do next? A) Inspect for a lift. B) Palpate for a thrill. C) Auscultate for pulse rate deficit. D) Listen for a ventricular gallop.

C) Auscultate for pulse rate deficit.

3. When examining the skin of an elderly client, the presence of which skin lesions should indicate a need for referral? A) Cherry angioma B) Actinic keratosis C) Seborrheic keratosis D) Acrochordons

B) Actinic keratosis

13. The nurse is reviewing a client's spirituality using the SPIRIT Spiritual Assessment Tool. Which of the following would the nurse assess when addressing the letter "P"? A) Powers B) Personal spirituality C) Spiritual prognosis D) Prayer

B) Personal spirituality

19. A nurse is creating a genogram of a client's family health history. The nurse should use which of the following symbols to denote the client's female relatives? A) Circle B) Square C) Triangle D) Rectangle

A) Circle

10. Which of the following would the nurse interpret as a positive response to the Phalen test for a client suspected of having carpal tunnel syndrome? A) Numbness B) Atrophy of the thenar prominence C) No tingling D) Hard, painless Bouchard nodes

A) Numbness

11. The nurse is assessing an elderly client who is receiving tube feedings via a nasogastric tube. The nurse should assess the client for signs and symptoms of which of the following? A) Gingivitis B) Sinusitis C) Epiglottitis D) Cellulitis

B) Sinusitis

16. The nurse is observing a client for evidence of pain. Which of the following would most likely lead the nurse to suspect that the client may be experiencing pain? A) Frequent questioning B) Slumped posture C) Eye contact D) Periodic position changes

B) Slumped posture

9. Which of the following should a nurse suspect if dullness is percussed at the last left interspace at the anterior axillary line on deep inspiration? A) Hepatomegaly B) Splenomegaly C) Abdominal mass D) Intestinal air

B) Splenomegaly

5. A nurse is preparing to discuss the cycle of violence with a group of women who have been victims of abuse. Which of the following would the nurse include as part of phase 3 of this cycle? A) A reconciliation period B) Shoving and slapping C) Victim separation from support group D) Acute battering

A) A reconciliation period

12. A nurse is planning a community-based health intervention. Which of the following should the nurse cite as an example of a geopolitical community? Select all that apply. A) A state B) A school district C) An Alzheimer's association D) The state nursing association E) A census tract

A) A state B) A school district E) A census tract

17. During which part of the comprehensive assessment would the nurse auscultate after inspecting but before percussing? A) Abdomen B) Anterior chest C) Neck D) Heart

A) Abdomen

13. A client has presented for care with complaints of persistent lower back pain. When using the mnemonic COLDSPA, which question should the nurse use to evaluate the "P"? A) "What makes it worse?" B) "When did it start?" C) "How does it feel?" D) "How would you rate your pain?"

A) "What makes it worse?"

24. A nurse recognizes the need to assess a client's spirituality after the client has been admitted from the emergency department to the medical unit. How should the nurse best initiate this assessment? A) "Would you describe yourself as being a religious or spiritual type of person?" B) "What is the belief system that you most closely adhere to?" C) "What church do you attend at Christmas and Easter?" D) "Do you consider yourself to be a moral person with beliefs about the supernatural?"

A) "Would you describe yourself as being a religious or spiritual type of person?"

19. A nurse obtains the following information: right arm brachial pressure, 160 mm Hg; left arm brachial pressure, 150 mm Hg; right ankle pressure, 80 mm Hg; left ankle pressure, 94 mm Hg. The nurse determines that the right ankle-brachial index would be which of the following? A) 0.50 B) 0.53 C) 0.59 D) 0.63

A) 0.50

22. A client who takes oral contraceptives states that she often experiences breast pain just before her menstrual cycle begins. When using the COLDSPA mnemonic to assess the client's pain, the nurse should begin by asking which of the following? A) "How would you describe your pain? Is it sharp? Is it an ache?" B) "Has the pain changed over time?" C) "Would you describe the pain as being constant or as intermittent?" D) "Is there anything that makes the pain worse or better?"

A) "How would you describe your pain? Is it sharp? Is it an ache?"

25. The nurse is beginning the inspection of a young adult client's breasts. The client states, "My left breast has always been a bit bigger than the right." How should the nurse best respond to the client's statement? A) "Many women have this, and it's rarely a sign of a health problem." B) "That's very normal, and it usually resolves over time as you get older." C) "If you lose some weight, the size disparity will likely decrease." D) "I'll make sure to refer to the doctor to get this assessed further."

A) "Many women have this, and it's rarely a sign of a health problem."

14. When taking the Daily Spiritual Experiences Scale, a client says the word "God" in the scale is bothersome. Which response by the nurse would be most helpful in encouraging a client to complete the scale? A) "Substitute whatever word you prefer that would represent the divine or holy." B) "You can skip those questions and answer only those you are comfortable with." C) "Don't be concerned about the wording; just answer the best way you know how." D) "It is perfectly fine to leave out any question that contains the word 'God.'"

A) "Substitute whatever word you prefer that would represent the divine or holy."

2. During chest auscultation, the nurse hears a quiet murmur immediately upon placing the stethoscope on the client's chest. The nurse interprets this as which grade? A) 1 B) 2 C) 3 D) 4

B) 2

16. When evaluating a client's risk for cerebrovascular accident, which client should the nurse identify as being at highest risk? A) A 42-year-old Caucasian female who smokes B) A 68-year-old African-American male with hypertension C) A 70-year-old Caucasian male who has one to two beers a day D) A 35-year-old African-American male who has sleep apnea

B) A 68-year-old African-American male with hypertension

28. A nurse has selected several nursing diagnoses in the process of data analysis of a client with poorly controlled type 1 diabetes. One of these diagnoses is Ineffective Health Maintenance related to infrequent blood glucose monitoring as manifested by elevated HgA1C. The nurse recognizes the need to corroborate this diagnosis with the client. How should the nurse best do this? A) "I think you have a nursing diagnosis of Ineffective Health Maintenance." B) "Would you agree that there's room for improvement in your routines around blood sugar monitoring?" C) "After assessing you, I believe that you're not maintaining your health effectively, specifically around your diabetes." D) "How do you think that you could better maintain your health?"

B) "Would you agree that there's room for improvement in your routines around blood sugar monitoring?"

4. A newly pregnant client says that she has heard that her nipples will leak milk during the pregnancy. The nurse should tell the client that she should expect to be able to express colostrum from her nipples beginning at how many weeks' gestation? A) 6 to 8 B) 12 to 14 C) 24 to 28 D) 34 to 36

C) 24 to 28

18. The nurse is measuring the fundal height of a woman who is at 28 weeks' gestation. Which measurement would the nurse expect? A) 12 cm B) 18 cm C) 28 cm D) 32 cm

C) 28 cm

18. Assessment of a client's radial pulse reveals that it is bounding and does not disappear with moderate pressure. The nurse documents the pulse amplitude as which of the following? A) 1+ B) 2+ C) 3+ D) 4+

C) 3+

14. Assessment reveals that a client has slight weakness with active range of motion against some resistance. The nurse would document this as which of the following? A) 2/5 B) 3/5 C) 4/5 D) 5/5

C) 4/5

22. During an educational inservice, nursing have been encouraged to conduct a self-appraisal of their critical thinking skills. Which of the following questions can best guide this appraisal? A) "Do I tend to make errors in my nursing practice?" B) "Do I get good feedback from clients and their families?" C) "Am I open to the fact that I may not be right?" D) "Am I a resource to my colleagues during a crisis?"

C) "Am I open to the fact that I may not be right?"

5. A nurse is providing feedback to a colleague after observing the colleague's interview of a newly admitted client. Which of the following would the nurse identify as an example of a closed-ended question or statement? A) "Tell me about your relationship with your children?" B) "Tell me what you eat in a normal day?" C) "Are you allergic to any medications?" D) "What is your typical day like?"

C) "Are you allergic to any medications?"

14. A medical nurse has completed the review of systems component of the client's health history. Which assessment finding should the nurse document under the review of systems? A) "High school diploma plus 2 years of college" B) "Caregiver reliable source of information" C) "Menarche at age 13" D) "Lungs clear to auscultation bilaterally"

C) "Menarche at age 13"

16. The nurse is participating in a vision-screening program for children age 3 to 10 years. The nurse would expect a child to have 20/20 vision at what age? A) 3 to 4 B) 4 to 5 C) 5 to 6 D) 6 to 7

D) 6 to 7

22. A client has sought care because she states that she has begun to see halos around headlights and streetlights when she is out at night. The nurse should recognize the need to refer the client for further assessment related to what health problem? A) Episcleritis B) Strabismus C) Macular degeneration D) Glaucoma

D) Glaucoma

25. A nurse has been clustering the data that he collected during the initial assessment of a frail elderly client. When making inferences about the data clusters, the nurse is unsure whether to associate a cluster of data with a nursing diagnosis or with a collaborative problem. What question may best guide the nurse's decision? A) "Can an unlicensed care provider meet this person's needs?" B) "Is this problem acute or is it chronic?" C) "Can this issue be addressed on an outpatient basis?" D) "Does this issue require medical intervention?"

D) "Does this issue require medical intervention?"

11. A nurse has admitted a client to the medical unit and is describing the purpose for obtaining a comprehensive health history. Which of the following purposes should the nurse describe? A) "This helps us to complete your health record accurately." B) "This helps us to establish a trusting interpersonal relationship." C) "This helps us to evaluate the seriousness of your risk factors for disease." D) "This helps us have an appropriate focus for the physical examination."

D) "This helps us have an appropriate focus for the physical examination."

19. When the nurse palpates the neck of an infant, he notes the presence of crepitus at the right shoulder area. The infant also exhibits decreased movement in the right arm. Which of the following should the nurse suspect? A) Osteomyelitis B) Down syndrome C) Fractured humerus D) Fractured clavicle

D) Fractured clavicle

27. The nurse is completing an assessment of a 50-year-old female client who has sought care for recurrent migraines that have not responded to treatment. Following the review of systems, how should the nurse best document unremarkable results of the subjective portion of the gastrointestinal assessment? A) "Client's gastrointestinal health is within reference ranges for age." B) "Client denies GI signs and symptoms." C) "Gastrointestinal problems are absent." D) "Client denies recent constipation, diarrhea, bowel incontinence, or abdominal pain."

D) "Client denies recent constipation, diarrhea, bowel incontinence, or abdominal pain."

30. The nurse is applying the Community as Partner model to assess a community. The nurse will assess which of the following aspects of the community? Select all that apply. A) Communication in the community B) Education in the community C) The community's economy D) Recreational facilities in the community E) The aspirations of the community

A) Communication in the community B) Education in the community C) The community's economy D) Recreational facilities in the community

27. The nurse is performing the bulge test during the assessment of a client's knee. This test will allow the nurse to make what determination? A) Whether the client's swollen knee is caused by tissue swelling or by fluid accumulation B) Whether the size of the client's knee changes throughout the joint's range of motion C) Whether swelling in the knee joint is a normal age-related change or a pathological finding D) Whether the client's knee joint is capable of adduction and abduction

A) Whether the client's swollen knee is caused by tissue swelling or by fluid accumulation

19. The nurse is preparing to perform a rectovaginal examination on a client. Which statement by the nurse would be most appropriate? A) I have to do this exam to make sure everything is okay, so just bear with me. B) You might feel uncomfortable, almost like you have to move your bowels. C) Just relax, it will only take a minute and then I'll be all finished. D) I want you to hold your breath as I insert my fingers into the openings.

B) You might feel uncomfortable, almost like you have to move your bowels.

29. A nurse is performing a detailed pain assessment of a client who has sought care for debilitating migraines. When assessing for precipitating factors, what question should the nurse ask? A) Is there anything that's given you relief in the past? B) Have your migraines gotten more severe in the last few months? C) What were you doing immediately before your last migraine? D) How long does a typical migraine last?

C) What were you doing immediately before your last migraine?

13. A nurse is applying the diagnostic reasoning process in the care of a client. What is the correct sequence of the steps that the nurse should perform? A) Check for defining characteristics. B) Draw inferences. C) Propose possible nursing diagnoses. D) Identify abnormal data and strengths. E) Cluster data.

D, E, B, C, A D) Identify abnormal data and strengths. E) Cluster data. B) Draw inferences. C) Propose possible nursing diagnoses. A) Check for defining characteristics.

23. A client who is in her first trimester states, "I've always been a fairly inactive person, but I'm determined to start going to exercise classes every day so my baby's as healthy as possible." How should the nurse respond to the client's statement? A) "Usually, your doctor will recommend against starting brand new exercise programs while you're pregnant." B) "Good for you. Regular physical activity tends to make labor and delivery go much smoother." C) "That's an excellent idea, and it really reduces your risk of developing high blood sugar during pregnancy." D) "Remember to start low and go slow to avoid putting stress on your baby."

A) "Usually, your doctor will recommend against starting brand new exercise programs while you're pregnant."

23. A nurse has admitted a client to the medical unit who has just been diagnosed with endocarditis secondary to IV drug use. The nurse has completed the collection of objective and subjective data. What question should guide the next step in the nurse's data analysis? A) "What are this client's strengths?" B) "What is this client's prognosis?" C) "Why does this client use opioids?" D) "What are this client's hopes for the future?" Page 5

A) "What are this client's strengths?"

13. The nurse is preparing to measure the head circumference of a newborn. In a healthy newborn, the nurse should expect the circumference of the infant's head to be within what range? A) 33 to 35.5 cm B) 35 to 37.5 cm C) 37 to 39.5 cm D) 39 to 41.5 cm

A) 33 to 35.5 cm

24. The nurse has completed the initial assessment of a client and is now performing data analysis. The nurse obtained a blood pressure reading of 114/70 mm Hg. What is this client's pulse pressure? A) 44 mm Hg B) 92 mm Hg C) 114 mm Hg D) 184 mm Hg

A) 44 mm Hg

29. In which of the following male clients would gynecomastia be considered to be an expected assessment finding? A) A 14-year-old boy who began puberty last year B) An older adult who takes antihypertensive medications C) A 59-year-old man who has been exposed to heavy metals in the workplace D) A male client who has been diagnosed with breast cancer

A) A 14-year-old boy who began puberty last year

6. A nurse who works at an outpatient ophthalmic clinic has a large number of clients. Which client would be at the highest risk for developing cataracts? A) A 55-year-old female client B) A 40-year-old with arteriosclerosis C) A client who has severe environmental allergies D) A male client who is obese

A) A 55-year-old female client

27. A nurse is assessing an African-American client who has a longstanding diagnosis of hypertension. The nurse should be aware that the client may experience a greater-than- average effect of what medication? A) A diuretic B) An angiotensin-converting enzyme inhibitor C) A calcium channel blocker D) A beta-adrenergic blocker

A) A diuretic

25. During the assessments of infants' genitalia, what finding most clearly warrants referral for further assessment? A) A newborn male has an undescended testicle. B) A newborn female has bloody vaginal discharge. C) A newborn female has engorged labia. D) A newborn male has intact foreskin.

A) A newborn male has an undescended testicle.

10. A client at 26 weeks' gestation appears at the clinic for her first prenatal visit. During the health interview, she states that she has been a habitual cocaine user. The nurse understands that this client is at risk for which of the following? A) Abruptio placenta B) Thrombophlebitis C) Placenta previa D) Gestational diabetes

A) Abruptio placenta

7. A nurse has completed a client's initial assessment and is now interpreting and making inferences from the data. The nurse is involved in which phase of the nursing process? A) Analysis B) Planning C) Implementation D) Evaluation

A) Analysis

9. The nurse has appeared before a community governance committee and encourages them to respond to the views of the citizens. Long-term lack of response by lawmakers is likely to prompt what reaction on the part of citizens? A) Apathy B) Hostility C) Confusion D) Proactivity

A) Apathy

13. The nurse notes multiple elevated masses with irregular transient borders that are superficial, raised, and erythematous in a client who complains of an itching rash. Which question would be most important for the nurse to ask? A) Are you allergic to foods, medications, or other substances? B) Does anyone else in your family have a rash like this? C) How painful is your rash? D) What have you been doing to control the itching?

A) Are you allergic to foods, medications, or other substances?

2. During a health visit, a client says, I know that arteries and veins are both blood vessels, but what's the difference? Which of the following would the nurse include in the response? A) Arteries have thicker walls than veins. B) Arteries carry 70% of the body's blood volume. C) Arteries have a lower pressure than veins. D) Arteries carry waste from the tissues.

A) Arteries have thicker walls than veins.

7. A 4-year-old boy is brought to the emergency department by his parents, who state that he has been crying and saying his tummy hurts. Which method would be most appropriate for the nurse to initially assess the problem? A) Ask the child to point with one finger where it hurts. B) Inspect, palpate, percuss, and then auscultate the abdomen. C) Determine the time and character of the child's last bowel movement. D) Ask the child to describe the character of his pain.

A) Ask the child to point with one finger where it hurts.

27. The nurse is assessing a client's respiratory rate and rhythm during the beginning of a shift. The nurse knows that a normal breathing rate is between approximately 10 and 20 breaths per minute, but the client's rate is 29 breaths per minute. How should the nurse respond to this assessment finding? A) Ask the client if she has recently exerted herself. B) Report the finding to the client's primary care provider. C) Ask the client if she has smoked recently. D) Palpate the client's anterior and posterior thorax.

A) Ask the client if she has recently exerted herself.

30. The nurse is performing light palpation of the client's abdomen. How can the nurse best prevent voluntary guarding during this phase of assessment? A) Ask the client to breathe slowly and deeply. B) Perform auscultation prior to palpation. C) Explain the procedure to the client before palpating. D) Position the client sitting upright.

A) Ask the client to breathe slowly and deeply.

7. A nurse is assessing an adult client's eyes and vision. When performing the cover test, the nurse would cover one of the client's eyes and then do which of the following? A) Ask the client to focus on a distant object, looking for movement in the other eye. B) Ask the client to close the other eye then open that eye quickly. C) Ask the client to follow the nurse's finger with the other eye. D) Ask the client to look directly at a light with the other eye.

A) Ask the client to focus on a distant object, looking for movement in the other eye.

4. The nurse notes that an older adult client is wearing multiple layers of clothing on a warm fall day. Which of the following would be the nurse's priority assessment at this time? A) Asking whether the client often feels cold B) Assessing the client's developmental level C) Reviewing the client's culture for possible influence D) Observing the client's overall hygiene

A) Asking whether the client often feels cold

11. The nurse assesses the respirations of a 2-week-old infant and identifies periods of apnea lasting longer than 20 seconds. What should the nurse do next? A) Assess the apical heart rate. B) Percuss the lungs for consolidation. C) Auscultate the lungs for adventitious sounds. D) Inspect the shape of the thorax.

A) Assess the apical heart rate.

23. The nurse's assessment of a child's hair reveals that it is clean and neatly trimmed but exceptionally dry and brittle. What is the nurse's best response to this finding? A) Assess the child for signs and symptoms of impaired nutrition. B) Assess the child for indications of abuse or neglect. C) Facilitate a referral to a dermatologist. D) Encourage the child's mother to ensure that the child gets adequate exposure to sunlight.

A) Assess the child for signs and symptoms of impaired nutrition.

7. After teaching a group of students about the phases of the nursing process, the instructor determines that the teaching was successful when the students identify which phase as being foundational to all other phases? A) Assessment B) Planning C) Implementation D) Evaluation

A) Assessment

29. The nurse palpates a client's pulse and notes that the rate is 71 beats per minute, with an irregular rhythm. How should the nurse follow up this assessment finding? A) Auscultate the client's apical pulse. B) Palpate the client's ulnar pulse. C) Administer a dose of nitroglycerin. D) Reposition the client in a side-lying position.

A) Auscultate the client's apical pulse.

10. The nurse is auscultating a client's blood pressure and identifies the portion of the blood pressure cycle reflecting the break in sounds occurring between the first and second sounds. This is known as which of the following? A) Auscultatory gap B) Korotkoff sounds C) Phase V D) Diastolic value

A) Auscultatory gap

25. The nurse has positioned a client supine and asked her to perform the heel-to-shin test. An inability to run each heel smoothly down each shin should prompt the nurse to perform further assessment in what domain? A) Balance and coordination B) Light touch sensation C) Deep tendon reflexes D) Leg strength

A) Balance and coordination

15. A nurse is observing the red reflex in a client during an eye assessment. During this component of the assessment, the client states, I hope you can see it because I have cataracts. What finding should the nurse expect? A) Black spokes pointing inward B) White arc around the limbus C) Thickened bulbar conjunctiva D) A red spot on the retina

A) Black spokes pointing inward

29. During the nurse's assessment of the client's exercise and activity habits, the client laughs and then states, "Unless you're including channel surfing, I don't really do much of anything." How should the nurse best follow up this client's statement? A) Briefly describe some of the potential benefits of regular exercise. B) Ask the client if he understands the risk factors for heart disease and diabetes. C) Explain to the client that he should be performing aerobic exercise for 20 to 30 minutes at least three times a week. D) Document the nursing diagnosis of Risk for Activity Intolerance related to sedentary lifestyle.

A) Briefly describe some of the potential benefits of regular exercise.

14. The nurse is inspecting the client's vaginal musculature and asks the client to bear down. Which finding would lead the nurse to suspect that the client has a cystocele? A) Bulging of the anterior vaginal wall B) Protrusion of the cervix C) Urine leakage D) Protrusion at the back of the vaginal wall

A) Bulging of the anterior vaginal wall

30. A nurse has performed the corneal light reflex test during a client's eye examination. During this test, the nurse held a penlight 1 foot from the client's eyes and appraised the client's eye alignment in which of the following ways? A) By comparing the reflection of the light on the client's eye surface B) By comparing the speed of pupillary constriction C) By comparing how quickly the client blinks each eyelid D) By comparing the relative color of the sclerae before and after light exposure

A) By comparing the reflection of the light on the client's eye surface

9. A nurse provides care in a rural hospital that serves a community that has few minority residents. When interviewing a client from a minority culture, the nurse has enlisted the assistance of a "culture broker." How can this individual best facilitate the client's care? A) By interpreting the client's language and culture B) By evaluating the client's culturally based health practices C) By teaching the client about health care D) By making the client feel comfortable and safe

A) By interpreting the client's language and culture

15. During the health history, a client reports a decrease in his ability to smell. During the physical assessment, the nurse would make sure to assess which cranial nerve? A) CN I B) CN II C) CN VII D) CN IX

A) CN I

22. An emergency department nurse is assessing a client's complaint of upper abdominal pain. Using the COLDSPA mnemonic, with what assessment question would the nurse begin? A) Can you describe to me how your pain feels? B) How would you rate your pain on a 10-point scale? C) Is your pain affecting your ability to cope? D) Would you describe your pain as acute, or as chronic?

A) Can you describe to me how your pain feels?

6. A nurse is preparing a program to address family violence prevention. Which of the following would be most important for the nurse to incorporate into the program? A) Children raised with intimate partner violence are more likely to use violence as adults. B) One out of ten women who are pregnant often fall victim to intimate partner violence. C) Victims of abuse account for approximately 2,500 visits to their health care providers yearly. D) Abuse is most commonly perpetrated by people with low levels of education.

A) Children raised with intimate partner violence are more likely to use violence as adults.

12. A nurse observes the posture of a male client and finds him leaning forward and bracing himself while sitting on the exam table. Which of the following would the nurse most likely suspect? A) Chronic obstructive pulmonary disease B) Neurological deficit C) Metabolic disorder D) Vestibular disorder

A) Chronic obstructive pulmonary disease

5. When assessing a family that is known to be highly functional, the nurse would most likely find which type of boundaries? A) Clearly defined B) Diffuse C) Rigid D) Hierarchical

A) Clearly defined

29. The nurse is assessing CN V (trigeminal nerve) in a newly admitted client. What instruction should the nurse provide to the client during this phase of assessment? A) Clench your teeth together tightly. B) Close your left eye and look at me with your right. C) Look straight at me while I shine this light in your eye. D) Open your mouth wide and say 'ah.'

A) Clench your teeth together tightly.

4. A client weighs 106 pounds and is 5 feet 5 inches tall. As a result, her ideal body weight is 120 pounds. After determining the client's percentage of ideal body weight, which of the following should the nurse conclude? A) Client is mildly malnourished. B) Client is experiencing moderate malnutrition. C) Severe malnutrition is present. D) The client's body weight is within 10% of ideal body weight.

A) Client is mildly malnourished.

13. Assessment of a client's lower extremities reveals unilateral edema of the right foot and ankle. Which of the following would be most appropriate for the nurse to do next? A) Compare measurements of both extremities. B) Perform the Allen test. C) Check for bilateral varicosities. D) Palpate the femoral pulses.

A) Compare measurements of both extremities.

14. After teaching a group of students about geriatric syndromes, the instructor determines that the teaching was successful when the students identify which of the following as an example? A) Confusion B) Pneumonia C) Heart failure D) Renal failure

A) Confusion

7. A client has just been diagnosed with a sinus infection accompanied by large amounts of exudate. Which of the following assessment findings should the nurse anticipate along with this condition? A) Crepitus over the maxillary sinuses B) Frontal sinuses nontender to palpation C) Red, tender tympanic membrane D) Increased amounts of saliva production

A) Crepitus over the maxillary sinuses

30. A nurse admits to a colleague, I sometimes tend to avoid clients from other cultures because it's awkward and it's usually frustrating for me and for the client. This nurse is likely lacking in what construct of cultural competency? A) Cultural desire B) Cultural knowledge C) Cultural health D) Cultural harmony

A) Cultural desire

30. A nurse has collecting extensive data during a client assessment and is performing the first step in the process of data analysis. Successful completion of this step requires the nurse to do which of the following? A) Differentiate between expected findings and abnormal findings. B) Validate nursing diagnoses with the client and the client's family. C) Integrate the client's medical diagnosis with nursing diagnoses. D) Perform health promotion education.

A) Differentiate between expected findings and abnormal findings.

23. During a new client's nutritional assessment, the nurse asks the client's height and usual weight. The client states that he has no idea how much he weighs. How should the nurse respond? A) Do you feel like your weight has increased, decreased, or stayed the same lately? B) Why do you feel that it's not important to monitor your weight? C) In a typical day, what do you eat and drink? D) How would you describe your feelings around your body type and body mass?

A) Do you feel like your weight has increased, decreased, or stayed the same lately?

22. The nurse is using the COLDSPA mnemonic to assess a client's history of chest pain. What interview question addresses the A in this assessment model? A) Do you have any other symptoms together with your chest pain, such as nausea, sweating? B) In your experience, what kinds of activities tend to cause your chest pain? C) Would you describe your chest pain as being acute, or is it chronic? D) What changes do you have to make in order to accommodate your chest pain?

A) Do you have any other symptoms together with your chest pain, such as nausea, sweating?

10. A nurse assesses a client's capillary refill and finds it to be less than 2 seconds. Which of the following should the nurse do next? A) Document this finding as normal. B) Recheck in 5 minutes after elevating the arm. C) Reassess after applying warm compresses. D) Refer the client for medical follow-up.

A) Document this finding as normal.

5. A nurse is assessing a client of East Asian descent. Which biological variation would the nurse expect? A) Dry cerumen in the client's ears B) Profuse perspiration in the client's axillary area C) Strong body odor D) Longer eustachian tubes

A) Dry cerumen in the client's ears

21. The nurse is assessing a client who has been referred to the clinic because of possible arterial insufficiency. What assessment finding should the nurse identify as most consistent with this diagnosis? A) Dry, shiny, hairless shins and feet B) Pitting edema to the feet and ankles C) Numbness and tingling of the lower extremities D) Reddish-blue coloration of the shins and feet

A) Dry, shiny, hairless shins and feet

22. A 36-year-old woman has been a client of a fertility clinic for 2 years and has now scheduled an appointment, believing that she is pregnant. The nurse who provides care at the clinic should screen the woman for intimate partner violence (IPV) at what time? A) During the woman's first prenatal visit to the clinic B) As soon as an appointment can be scheduled with the woman's partner C) At a point when the woman states she is comfortable with being screened D) Once the woman begins her second trimester of pregnancy

A) During the woman's first prenatal visit to the clinic

6. The nurse should prioritize assessments related to overhydration for a client experiencing which of the following health problems? A) Early congestive heart failure B) Chronic emphysema C) Newly diagnosed hepatitis C virus infection D) Adult respiratory distress syndrome

A) Early congestive heart failure

5. Which of the following factors should a nurse include when discussing risk factors about breast cancer for a group of women? A) Early menarche B) One or more pregnancies before age 20 C) Consumption of a high-protein diet D) Early menopause

A) Early menarche

2. A nurse is preparing to assess an adult client's body temperature. At which time of the day would the nurse expect to obtain the lowest body temperature? A) Early morning B) Early afternoon C) Late afternoon D) Late evening

A) Early morning

10. A nurse is performing an eye assessment of an 81-year-old male client. Which of the following would the nurse document as a normal finding? A) Ectropion B) Episcleritis C) Chalazion D) Exophthalmos

A) Ectropion

11. A male Hispanic client describes the fact that he mixed hot and cold foods, causing them to lump together and get stuck in his intestines, causing diarrhea and abdominal pain. The nurse would document this as which of the following? A) Empacho B) Susto C) Mal ojo D) Mal puesto

A) Empacho

5. The nurse's assessment of an adult female client reveals the presence of excessive hair on her face and chest. The nurse should plan further evaluation of which body system? A) Endocrine B) Neurologic C) Cardiovascular D) Genitourinary

A) Endocrine

23. A nurse is admitting a 30-year-old female client and recognizes the need to screen the client for abuse. Prior to doing so, the nurse should do which of the following? A) Ensure a private setting. B) Perform a physical assessment. C) Have the client sign an informed consent form. D) Teach the client about intimate partner violence (IPV).

A) Ensure a private setting.

2. A young adult client has come to the clinic for her scheduled Pap (Papanicolaou) test and pelvic examination. The nurse is implementing actions to help reduce a client's anxiety during the physical exam. Which of the following would be most appropriate? A) Ensuring client's privacy by providing an examination gown B) Providing a comfortable, warm room temperature C) Arranging exam equipment on a bedside tray table D) Explaining why standard precautions are being used

A) Ensuring client's privacy by providing an examination gown

1. A nurse is preparing to assess a client who is new to the clinic. When beginning the collection of the client database, which of the following actions should the nurse prioritize? A) Establishing a trusting relationship B) Determining the client's strengths C) Identifying potential health problems D) Making clinical inferences

A) Establishing a trusting relationship

2. A pregnant client asks the clinic nurse what she can use to relieve her nasal "stuffiness." The nurse bases the answer on the most likely cause of the congestion, which is attributable to which hormone? A) Estrogen B) Progesterone C) Thyroxine D) Relaxin

A) Estrogen

2. During the health history, a postmenopausal client mentions that she is experiencing vaginal dryness. When explaining the most likely reason to the client, the nurse should explain the role of which hormone? A) Estrogen B) Progesterone C) Follicle-stimulating hormone (FSH) D) Oxytocin

A) Estrogen

21. A nurse's reflection of his practice reveals that he tends to see his own culture as the gold standard to which all other cultures should aspire. This nurse should create learning goals to address what phenomenon? A) Ethnocentrism B) Unconscious incompetence C) Stereotyping D) Acculturation

A) Ethnocentrism

2. A nurse is admitting a client who is from another culture. Prior to caring for a client from another culture, the nurse should place primary importance on which action? A) Examining personal biases and prejudices B) Researching characteristics of the specific culture C) Asking colleagues about ways to approach the client D) Developing awareness of the culture's health practices

A) Examining personal biases and prejudices

16. A group of nurses are reviewing information about the potential opportunities for nurses who have advanced assessment skills. When discussing phenomena that have contributed to these increased opportunities, what should the nurses identify? A) Expansion of health care networks B) Decrease in client participation in care C) The shrinking cost of medical care D) Public mistrust of physicians

A) Expansion of health care networks

9. A female client is told that she needs a pelvic exam and Papanicolaou (Pap) smear. She says Absolutely not! There's no way I'll let you do that to me! Which response by the nurse would be most appropriate? A) Explain the importance of the pelvic exam and Pap smear, but respect the client's wishes and omit the exam. B) Tell the client that this is the only way she can be checked for cancer. C) Ask the client if she would prefer another practitioner to perform the exam. D) Proceed with the pelvic exam and document the client's protests in the health record.

A) Explain the importance of the pelvic exam and Pap smear, but respect the client's wishes and omit the exam.

4. A nurse interviews a pregnant client and learns that her beliefs around health care do not involve participation in comprehensive prenatal care. To which religious view would the client most likely adhere? A) Faith Assembly of Indiana B) Buddhist C) Christian Scientist D) Jehovah's Witness

A) Faith Assembly of Indiana

29. The nurse's assessment reveals that a family possesses a high level of affective functioning. What observation would most likely lead the nurse to this conclusion? A) Family members appear to be close to each other and considerate of each other. B) Family members communicate frequently and clearly. C) Family members agree that conflict is unacceptable. D) Family communication is characterized by direct statements.

A) Family members appear to be close to each other and considerate of each other.

4. The nurse is preparing to test a client's eyes for accommodation. The nurse would have the client focus on an object in which sequence for this test? A) Far, then near B) Lateral, then near C) Near, then far D) Lateral, then far

A) Far, then near

18. When assessing the client's legs, feet, and toes, which pulses would the nurse expect to palpate? Select all that apply. A) Femoral B) Brachial C) Temporal D) Dorsalis pedis E) Popliteal F) Posterior tibial

A) Femoral D) Dorsalis pedis E) Popliteal F) Posterior tibial

28. A nurse at a long-term care facility is completing the nutrition assessment of a man who has just moved to the facility. The nurse has lowered the client's arm and observed how long it takes for venous filling, then raised the same arm and watched how long it takes to empty. After determining that venous filling and emptying each take approximately 10 seconds, the nurse should perform further assessments related to what health problem? A) Fluid volume deficit B) Third spacing C) Ascites D) Malnutrition

A) Fluid volume deficit

3. The nurse is assessing a client who has been admitted for the treatment of severe dehydration. What might the nurse expect to hear when auscultating the lungs of a client with this fluid volume deficit? A) Friction rub B) Decreased breath sounds C) Sibilant wheeze D) Stridor

A) Friction rub

16. When assessing a family's structure, the nurse should gather information about which of the following? Select all that apply. A) Gender roles B) Rank order C) Religion D) Stage of growth E) Beliefs about an illness

A) Gender roles B) Rank order C) Religion

30. A nurse is having difficulty getting a 14-year-old child to open up during the health interview. What strategy is most likely to enhance the nurse's communication with this child? A) Give the child some control over the course and content of the interview. B) Teach the child about the negative consequences of an inadequate interview. C) Arrange for one of the child's parents to speak with him or her privately. D) Promise the child a reward for participating in the interview.

A) Give the child some control over the course and content of the interview.

20. During the physical exam, the nurse notes a very tender and painful, reddened, hot, and swollen metatarsophalangeal joint of the client's great toe. Which of the following would the nurse suspect? A) Gouty arthritis B) Rheumatoid arthritis C) Degenerative joint disease D) Plantar fasciitis

A) Gouty arthritis

15. The nurse is assisting a female client who has been physically abused about a safety plan. The client prefers to return home. Which of the following would the nurse need to do first? A) Have the client complete a danger assessment. B) Notify the neighbors about the abuse. C) Tell her to have her bags packed. D) Give the client the number of a shelter.

A) Have the client complete a danger assessment.

14. When assessing a client for possible varicose veins, the nurse should do which of the following actions? A) Have the client stand for the exam. B) Tell the client to raise his or her leg. C) Dorsiflex the client's foot. D) Obtain the ankle-brachial index.

A) Have the client stand for the exam.

11. The nurse is assessing an older adult client whose health problems include receding gums. The nurse notes gum ischemia and worn tooth surfaces. Which question would be most important for the nurse to ask? A) Have you lost any teeth recently? B) How would you describe your typical diet? C) Has your dentist screened you for oral cancer recently? D) Are you able to taste the food you eat?

A) Have you lost any teeth recently?

23. A female client has presented for a Pap smear test, and the nurse is discussing risk factors for cervical cancer. What risk factor should the nurse describe? A) Having multiple sexual partners B) Previous treatment for chlamydial infection C) Pregnancy before age 21 D) African-American ethnicity

A) Having multiple sexual partners

13. The nurse analyzes the data obtained from a client's nutritional assessment and develops a health promotion diagnosis related to nutrition for a client. Which of the following would be the best example? A) Health-seeking behaviors related to desire and request to alter amount of food intake B) Imbalanced nutrition: less than body requirements related to inadequate caloric intake C) Imbalanced nutrition: more than body requirements related to excessive caloric intake D) Ineffective thermoregulation related to decreased adaptability to cold secondary to decreased subcutaneous tissue

A) Health-seeking behaviors related to desire and request to alter amount of food intake

9. When talking to a client before starting the physical exam, the nurse notes that the client consistently tilts her head to one side. Which of the following should the nurse examine first? A) Hearing acuity B) Thyroid gland C) Mental status D) Lymph nodes

A) Hearing acuity

17. In the course of performing a client's physical assessment, the nurse has changed from using the diaphragm of the stethoscope to using the bell. The nurse is most likely assessing which of the following? A) Heart sounds B) Bowel sounds C) Breath sounds D) Femoral pulses

A) Heart sounds

11. When evaluating nutrition in an adult female client, which laboratory value would most concern the nurse? A) Hemoglobin A1c of 9% B) Serum albumin of 4.9 g/dL C) Total protein of 6.7 g/dL D) Hematocrit of 39%

A) Hemoglobin A1c of 9%

24. The nurse's inspection of a young child's anus reveals the presence of hemorrhoids. How should the nurse best interpret this assessment finding? A) Hemorrhoids are unusual in children and warrant further assessment. B) Hemorrhoids are a common indication of deficient fluid intake in children. C) Hemorrhoids are common in children until they attain bowel continence. D) Hemorrhoids in a child younger than 10 are suggestive of colorectal cancer.

A) Hemorrhoids are unusual in children and warrant further assessment.

3. A client presents with a cluster of upper airway complaints that include rhinorrhea. Which area of assessment would yield the most pertinent information to the etiology of rhinorrhea? A) History of allergies B) Incomplete immunization record C) History of epistaxis (nosebleeds) D) Prolonged tonsillar enlargement

A) History of allergies

27. The nurse is assessing an adult client for the presence of Piaget's formal operations stage of development. What assessment question should the nurse ask the client? A) How do you usually go about making difficult decisions? B) Do you consider yourself to be an intelligent person? C) How would you describe your relationship with authority figures? D) In relationships, do you consider yourself to be a 'giver' or a 'taker'?

A) How do you usually go about making difficult decisions?

22. The nurse is assessing a client who enjoys good health overall but who has brought a complaint of chronic nasal congestion and recurrent nosebleeds. What interview question should the nurse prioritize? A) How often do you use over-the-counter nasal sprays? B) How often do you take Tylenol? C) How many drinks of alcohol do you have in a typical day? D) Would you say that you eat a balanced diet?

A) How often do you use over-the-counter nasal sprays?

26. A 21-year-old woman has been admitted to the emergency department following an accident that is suspected of being a suicide attempt. When assessing the client's perceptions, what question should the nurse ask the client? A) How would you describe your health these days? B) Are you able to smell and taste as well as you've been able to in the past? C) If you found a stamped envelope on the street, what would you do? D) Can you tell me the circumstances surrounding your accident?

A) How would you describe your health these days?

9. A client has a history of emphysema. During the respiratory assessment, the nurse percusses the client's chest, expecting to find which of the following? A) Hyperresonance B) Dullness C) Resonance D) Tympany

A) Hyperresonance

2. A nurse is assessing a female client whose worsening sciatica has prompted her to seek care. Which of the client's following statements would the nurse most likely need to validate? A) I don't generally have problems with pain. B) I feel very weak and tired right now. C) I've had two cesarean deliveries. D) My mother died of breast cancer in her sixties.

A) I don't generally have problems with pain.

25. What statement by a middle-aged adult would most clearly suggest successful achievement of Erikson's central task during this stage of development? A) I'm doing a lot of volunteering in order to give back to the community. B) I've started to exercise more regularly so that I don't put on extra weight. C) I socialize with my coworkers a lot more than I did when I was younger. D) Overall, my marriage is likely stronger than it was when we first got married.

A) I'm doing a lot of volunteering in order to give back to the community.

26. A nurse has assessed a client who was admitted to the medical unit to treat acute complications of type 1 diabetes. During the assessment, the client admitted that his blood sugar monitoring when he is at home is a bit sporadic. How should the nurse best respond to this assessment finding? A) Identify a nursing diagnosis of Ineffective Health Maintenance. B) Identify a collaborative problem that should involve the occupational therapist. C) Make a referral to the unit's social work department. D) Reassess the client's blood glucose level.

A) Identify a nursing diagnosis of Ineffective Health Maintenance.

3. A client who is at 23 weeks' gestation tells the nurse, "I just burn up all the time. I can't even sleep with any covers on me!" The nurse explains to the client that heat intolerance during pregnancy is primarily due to which physiologic change? A) Increased basal metabolic rate B) Decreased sweat gland activity C) Increase in maternal blood volume D) Stretching of abdominal muscles

A) Increased basal metabolic rate

6. The nurse is assessing a client who is in uncompensated right-sided heart failure. What assessment finding should the nurse anticipate? A) Increased jugular venous pressure B) Bradycardia C) Decreased blood pressure D) Dysrhythmias

A) Increased jugular venous pressure

13. The nurse has completed a focused ear and hearing assessment and gathered the following data: the client speaks very softly, denies hearing loss, and has never had and cannot afford additional hearing tests; the client fails the whisper test. Which nursing diagnosis would be most appropriate? A) Ineffective health maintenance related to denial of hearing problem and inadequate resources for additional testing B) Impaired social interaction, related to decreased ability to maintain contact with friends C) Impaired verbal communication, related to lack of understanding of hearing deficit D) Readiness for enhanced communication related to auditory integrity and need for hearing therapy

A) Ineffective health maintenance related to denial of hearing problem and inadequate resources for additional testing

21. A workplace injury has caused a man to be on long-term disability compensation. His wife has returned to paid employment after many years absence in order to make ends meet. The clients both agree that this family transition has been challenging, with the wife stating, He says that he feels he's unproductive, even though there's nothing that can be done about it. What nursing diagnosis may possibly apply to the husband? A) Ineffective role performance related to loss of employment B) Impaired Home Maintenance related to loss of employment C) Caregiver role strain related to injury D) Risk for Impaired Parenting related to family changes

A) Ineffective role performance related to loss of employment

23. A nurse is completing an assessment that will involve gathering subjective and objective data. Which of the following assessment techniques will best allow the nurse to collect objective data? A) Inspection B) Therapeutic communication C) Interviewing D) Active listening

A) Inspection

26. A nurse is reviewing the four basic physical examination techniques and their sequence prior to receiving a new client from postanesthetic recovery. The nurse should plan to perform which technique first? A) Inspection B) Palpation C) Percussion D) Auscultation

A) Inspection

20. While interviewing a family, the mother says that she does the cooking and takes the children to and from school and to their after-school activities during the week. The father reports that he drives the children to their activities and does the cooking on the weekends. The nurse interprets this information as reflecting which family function? A) Instrumental B) Affective C) Expressive D) Health care

A) Instrumental

20. A group of students is preparing for a quiz on breast assessment and the assessment findings that are associated with breast cancer. The students demonstrate understanding of the material when they identify which of the following? Select all that apply. A) Irregular, firm lumps B) Elastic, tender, mobile lumps C) Dimpling and nipple retraction D) Orange peel-like appearance E) Redness and warmth with smooth texture F) Breast fullness and pain

A) Irregular, firm lumps C) Dimpling and nipple retraction D) Orange peel-like appearance

11. A nurse assesses a client's spirituality and religious practices. During the assessment, the nurse notes that the client is very quiet and rarely asks any questions of the health care workers. The nurse recognizes that this behavior may be associated with which religion? A) Islam B) Buddhism C) Hinduism D) Christianity

A) Islam

3. While interviewing a teenage male client, the nurse reviews the various structures of the male genitalia. The client asks, So what does this epididymis do? Which of the following would the nurse include in the response? A) It allows sperm to mature. B) It transports sperm away from the testes. C) It separates the testes from the scrotal wall. D) It produces sperm and male sex hormones.

A) It allows sperm to mature.

12. A group of nursing students is reviewing the purposes of assessment documentation in preparation for a class discussion. The students demonstrate understanding of the information when they identify which of the following as one of the primary purposes? A) It provides a chronologic source of client assessment data. B) It creates a database for care that was not rendered to the client. C) It replaces the client acuity classification system. D) It directly formulates the nursing diagnoses.

A) It provides a chronologic source of client assessment data.

1. A nurse is working with a client who has received a terminal diagnosis. To help the nurse identify the client's possible coping responses, which of the following would be most important for the nurse to understand about spirituality? A) It varies in different situations. B) It increases in significance with illness. C) It decreases in importance with age. D) It remains static throughout the lifespan.

A) It varies in different situations.

9. The nurse is caring for the family of a client who has just died. The family requests that the client's arms not be crossed and that any of the clothing and dressings containing blood be left and be prepared for burial with the client. The nurse understands this family's request as indicative of what religious beliefs? A) Judaism B) Buddhism C) Hinduism D) Christianity

A) Judaism

5. A nurse has completed a client's initial assessment and is preparing to identify abnormal data and the client's strengths. Successful completion of this phase of the nursing process most requires which of the following? A) Knowledge of anatomy and physiology B) Awareness of the client's medical prognosis C) Inferences about the client D) Knowledge about the referral process

A) Knowledge of anatomy and physiology

3. A nurse is planning a spiritual assessment of a client who is experiencing intractable losses in function as a result of disease. Which of the following principles should inform the nurse's assessment? A) Knowledge of the most common spiritual practices in the community is a priority. B) Reviewing all religious denominations before approaching a client is important. C) It is of little importance for a nurse to understand his or her own spirituality. D) Spirituality is a complex phenomenon that is not normally describable.

A) Knowledge of the most common spiritual practices in the community is a priority.

19. After teaching a group of students about the external and internal structures of the eye, the instructor determines that the teaching was successful when the students identify which of the following as external structures? Select all that apply. A) Lacrimal apparatus B) Conjunctiva C) Lens D) Iris E) Sclera F) Caruncle

A) Lacrimal apparatus B) Conjunctiva F) Caruncle

5. The nurse is applying Piaget's theory of development to a client's health history. This approach to analysis will prioritize what activity on the part of the client? A) Learning B) Imitating C) Indulging D) Desiring

A) Learning

11. The nurse is preparing to auscultate the client's thorax. Which of the following actions is the priority during this component of assessment? A) Listen at each site for at least one complete respiratory cycle. B) Have the client breathe deeply through his or her nose. C) Encourage the client to cough before auscultating each site. D) Have the client hold the breath for a few seconds after auscultating each site.

A) Listen at each site for at least one complete respiratory cycle.

26. A nurse is providing care for a client who has longstanding type 2 diabetes. In recent days, the client's blood glucose levels have been higher and more volatile than usual. After drawing this inference, the nurse should take what action? A) Make appropriate referrals B) Assess the client more frequently C) Document the medical diagnosis of hyperglycemia D) Beginning collecting subjective data

A) Make appropriate referrals

30. A teenage boy has been diagnosed with orchitis. When reviewing the child's health history, the nurse should expect that the client may have recently been treated for what health problem? A) Measles B) Varicella C) Phimosis D) Influenza A

A) Measles

18. A nurse is providing a client with instructions on how to perform self-examination of the skin. The nurse would encourage the client to perform this examination at which frequency? A) Monthly B) Bimonthly C) Quarterly D) Yearly

A) Monthly

14. An instructor is reviewing the evolution of the nurse's role in health assessment. The instructor determines that the teaching was successful when the students identify which of the following as the major method used by nurses early in the history of the profession? A) Natural senses B) Biomedical knowledge C) Simple technology D) Critical pathways

A) Natural senses

15. When assessing pain in an older adult client who is alert and oriented, which assessment tool would be most appropriate to use? A) Numerical rating scale B) Faces Pain Scale-Revised C) FLACC Scale D) Graphic rating scale

A) Numerical rating scale

28. A 20-year-old female client has presented to the clinic, and the nurse is preparing to perform a comprehensive assessment. The client states, I'd really like to have my mom in the room. That's okay, isn't it? How should the nurse best respond to the client's request? A) Of course. There's a chair in the exam room where she can sit. B) That's no problem. I'll just have to get you to sign a privacy waiver first. C) That's fine, but be aware that some of the examinations might be embarrassing for you or her. D) It's best to undergo the examination alone in order to make sure I get accurate data, but if you really want her present, we can do that.

A) Of course. There's a chair in the exam room where she can sit.

21. The nurse's assessment of an older adult client's ears and hearing suggests the possible presence of conductive hearing loss. Which of the following is the most likely etiology of this abnormal assessment finding? A) Otitis media B) Cranial nerve VIII damage C) Trauma to the temporal lobe D) Age-related hearing changes

A) Otitis media

19. When reviewing the neural pathways, a group of students is identifying sensations that travel via the spinothalamic tract. Select all the sensations that are carried by this tract. A) Pain B) Temperature C) Position D) Vibration E) Light touch

A) Pain B) Temperature E) Light touch

20. During a health history, a client reports complaints of headaches. Which of the following would lead the nurse to suspect that the client is experiencing cluster headaches? A) Pain radiating from eye to temporal region B) Throbbing and severe pain C) Report of ringing in the ears prior to headache D) Complaint of sensitivity to light

A) Pain radiating from eye to temporal region

16. The nurse is preparing to assess a client's carotid arteries. Which of the following actions would be most appropriate? A) Palpate each artery individually to compare. B) Palpate the arteries before auscultating them. C) Use the diaphragm of the stethoscope. D) Ask the client to breathe in and out deeply.

A) Palpate each artery individually to compare.

27. An older adult client has received a diagnosis of stress incontinence, and the nurse is planning the client's subsequent care. What health education is most relevant to this client's needs? A) Pelvic floor strength training and activity management B) Appropriate use of incontinence pads and dietary modifications C) Management of fluid and electrolyte intake D) Aseptic technique for intermittent catheterization and fluid restriction

A) Pelvic floor strength training and activity management

10. When describing circular communication to a group of students, which of the following would the instructor explain as the basis for the circular communication feedback loop? A) Perceptions and reactions B) Symmetry and complementarily C) Content and relationships D) Listening and observation

A) Perceptions and reactions

1. While auscultating the client's heart at the third intercostal space and on the left sternal border, the nurse notes a high-pitched, scratchy sound that increases with exhalation with the client leaning forward. The nurse should document which of the following? A) Pericardial friction rub B) Midsystolic click C) Summation gallop D) Aortic ejection click

A) Pericardial friction rub

18. A nurse is assessing an Asian client and observes several reddened and bruised areas on the skin. Further assessment reveals that the client was using cupping to treat back pain. The nurse understands this as which of the following? A) Placing heated glass jars on the skin that are allowed to cool B) Rubbing ointment into the skin with a spoon C) Attaching smoldering herbs to acupuncture needles D) Placing warm burning herbs directly on the skin

A) Placing heated glass jars on the skin that are allowed to cool

25. The nurse is assessing an older adult client's vaccination history. This aspect of the client's history will have a significant bearing on her risk for what health problem? A) Pneumonia B) Urinary tract infections C) Cellulitis D) Tuberculosis

A) Pneumonia

19. A client comes to the emergency department complaining of pain in the right lower quadrant. Rebound tenderness is present, and the nurse assesses the client for referred rebound experiences. The client experiences pain the right lower quadrant. The nurse should document which of the following? A) Positive Rovsing's sign B) Psoas sign present C) Obturator sign positive D) Positive skin hypersensitivity test

A) Positive Rovsing's sign

7. A nurse is eliciting a client's health history and the client asks, "Can I take the herb ginkgo biloba with my other medications?" What action would be best if the nurse is unsure of the answer? A) Promise to find out the information for the client. B) Change the subject and return to this topic later. C) Teach the client to only take prescribed medications. D) Encourage the client to ask the pharmacist or primary care provider.

A) Promise to find out the information for the client.

22. An adult client has sought care because he has a two-day history of stool that is black like road tar. How should the nurse best respond to this aspect of the client's history? A) Promptly refer the client for treatment of a possible gastrointestinal bleed. B) Refer the client to a dietitian for treatment of a possible vitamin deficiency. C) Encourage the client to increase his intake of fluids and soluble fiber. D) Encourage the client to use an over-the-counter laxative for the next 2 to 3 days.

A) Promptly refer the client for treatment of a possible gastrointestinal bleed.

7. The nurse is assessing a client who has been admitted with signs and symptoms that are consistent with malnutrition. Which of the following physiological phenomena would the nurse recognize as an early indicator of malnutrition? A) Protein stores are lower than normal B) Bone is metabolized to compensate for missing nutrients C) Calcium levels decrease D) Hemoglobin levels decrease

A) Protein stores are lower than normal

13. A nurse is assessing the eyes of a 3-year-old child. Which finding would the nurse document as normal? A) Pseudostrabismus B) Tropia C) Nystagmus D) Exotropia

A) Pseudostrabismus

23. An adult client states that his mother has been living with peptic ulcer disease, and he is motivated to ensure that he does not develop the disease as he ages. What health promotion advice should the nurse provide? A) Quit smoking as soon as possible. B) Exercise for at least 30 minutes, three times per week. C) Eat several small meals a day rather than three larger meals. D) Attend screening clinics at least twice per year.

A) Quit smoking as soon as possible.

22. A surgical client's pain has become increasingly severe overnight, and she has received her maximum current doses of analgesics. The nurse has consequently phoned the surgeon to obtain a new order for analgesia. After the surgeon tells the nurse the new order, how should the nurse best validate this information? A) Read the order back to the surgeon for confirmation. B) Compare the order with the standard timing and dosage of the analgesic. C) Compare the order to the client's existing medication administration record (MAR). D) Have another nurse read the order that the nurse has transcribed.

A) Read the order back to the surgeon for confirmation.

9. The nurse has identified that a female client desires to leave her abusive husband and move with her children to her parents' house. Which of the following would be the most appropriate nursing diagnosis? A) Readiness for enhanced family processes B) Grieving related to loss of ideal relationship C) Disturbed personal identity related to moving to parent's house D) Impaired parenting related to loss of relationship

A) Readiness for enhanced family processes

9. Based on the analysis of assessment data from a client with pain, the nurse writes a health promotion diagnosis. Which of the following diagnoses would be most appropriate? A) Readiness for enhanced spiritual well-being related to coping with prolonged physical pain B) Risk for activity intolerance related to chronic pain and immobility C) Bathing self-care deficit related to severe pain D) Chronic pain related to chronic inflammatory process of rheumatoid arthritis

A) Readiness for enhanced spiritual well-being related to coping with prolonged physical pain

8. The nurse has completed the comprehensive health assessment of a client who has been admitted for the treatment of community-acquired pneumonia. Following the completion of this assessment, the nurse periodically performs a partial assessment primarily for which reason? A) Reassess previously detected problems B) Provide information for the client's record C) Address areas previously omitted D) Determine the need for crisis intervention

A) Reassess previously detected problems

28. The emergency department nurse's rapid assessment of a young adult client admitted unresponsive reveals fixed, constricted pupils bilaterally. The nurse should consider what possible cause for this assessment finding? A) Recent narcotic use B) Hemorrhagic stroke C) Recent seizure activity D) Cerebellar lesion

A) Recent narcotic use

3. The nurse utilizes the Depression Questionnaire on a client who has recently moved to a long-term care facility. The total score is 22. Which of the following would be most appropriate for the nurse to do next? A) Refer for further evaluation. B) Evaluate benefits versus risks of a mental health label. C) Assess further for dementia. D) Document this as a normal score.

A) Refer for further evaluation.

10. A woman reports a sudden onset of spontaneous nipple discharge. Which of the following would be the nurse's most appropriate action? A) Refer the client for cytologic study of the discharge. B) Observe the breast for eversion of the nipples. C) Reassure the woman that this is a result of hormonal fluctuations. D) Collect a sample for culture and sensitivity testing.

A) Refer the client for cytologic study of the discharge.

9. The emergency department nurse notes a clear, watery discharge from the client's ear following a bicycle accident. Which of the following actions should the nurse do next? A) Refer the client immediately for further evaluation. B) Assess for foreign body impaction. C) Examine for postauricular cysts. D) Position the patient to facilitate drainage.

A) Refer the client immediately for further evaluation.

23. A nurse is providing care at an inner-city shelter, and a man who frequents the shelter presents with a significant frontal growth that is located midline at the base of his neck. The nurse should recognize the need for what referral? A) Referral for further assessment of thyroid function B) Referral for assessment of cranial nerve function C) Referral for assessment of lymphatic system function D) Referral for further assessment of swallowing ability

A) Referral for further assessment of thyroid function

10. When assessing a client's mental status, which of the following would the nurse assess? Select all that apply. A) Remote memory B) Coping skills C) Speech D) Abstract reasoning E) Judgment

A) Remote memory C) Speech D) Abstract reasoning E) Judgment

8. When assessing a client for possible abuse, which of the following would most likely suggest that the client is a victim of abuse? A) Repeated emergency department visits for injuries B) Poor or no prenatal care C) History of alcohol abuse D) Failure to gain weight in pregnancy

A) Repeated emergency department visits for injuries

29. The nurse's assessment of a hospital client's spirituality reveals that the client will accept very few of the standard treatments for her health problems. How should the nurse follow up this assessment finding? A) Report the finding to the appropriate supervisors. B) Prioritize complementary interventions in the client's care. C) Consult the client's clergy to weigh options. D) Document the client's nonadherence to treatment.

A) Report the finding to the appropriate supervisors.

4. The nurse who provides care at an ambulatory clinic is preparing to meet a client and perform a comprehensive health assessment. Which of the following actions should the nurse perform first? A) Review the client's medical record. B) Obtain basic biographic data. C) Consult clinical resources explaining the client's diagnosis. D) Validate information with the client.

A) Review the client's medical record.

1. Assessment of a client reveals a history of insulin-dependent diabetes mellitus, weight loss, polyuria, poor skin turgor, nausea, loss of appetite, and a blood glucose level measured by finger stick of 348 mg/dL. Which of the following nursing diagnoses would be the nurse's priority? A) Risk for imbalanced fluid volume related to inadequate oral intake and frequent urination B) Imbalanced nutrition: more than body requirements related to diabetes C) Potential complication: hypertension D) Powerlessness related to diabetes self-care and management

A) Risk for imbalanced fluid volume related to inadequate oral intake and frequent urination

29. The nurse is planning the care of a 77-year-old woman who has recently been diagnosed with osteoporosis. What nursing diagnoses should the nurse address in the client's plan of care? Select all that apply. A) Risk for injury related to osteoporosis B) Risk for infection related to osteoporosis C) Activity intolerance related to osteoporosis D) Impaired physical mobility related to osteoporosis E) Disturbed sensory perception related to osteoporosis

A) Risk for injury related to osteoporosis C) Activity intolerance related to osteoporosis D) Impaired physical mobility related to osteoporosis

18. A group of students is reviewing information about the salivary glands and their secretions. The students demonstrate understanding of the information when they identify which of the following as components of saliva? Select all that apply. A) Salts B) Proteins C) Fats D) Mucus E) Amylase

A) Salts D) Mucus E) Amylase

26. A female client with advanced-stage vascular dementia has been showing signs of pain over the past several hours. The nurse is unable to obtain a self-report from the client due to her cognitive impairment. When applying the Hierarchy of Pain Assessment Techniques, how should the nurse proceed with assessment? A) Search for potential causes of pain. B) Ask the client's family if they believe she is in pain. C) Perform interventions as if the client were in pain. D) Use a visual assessment tool rather than a verbal tool.

A) Search for potential causes of pain.

6. The nurse chooses to use a formal assessment technique when doing a client's spiritual assessment. Which of the following techniques would be most appropriate? A) Self-response assessment instrument B) Acronyms related to spirituality C) Open-ended questions D) A systematic guide for question choices

A) Self-response assessment instrument

23. The gerontologic nurse is using the SPICES screening tool to assess an older adult's health status. The nurse will assess for which of the following health problems? Select all that apply. A) Sleep disturbances B) Infection C) Poor nutrition D) Falls E) Pain

A) Sleep disturbances C) Poor nutrition D) Falls

30. The nurse is assessing an older adult client's mental status. Consistently, the client pauses after the nurse poses a question, but then the client provides a response that is correct or appropriate. How should the nurse best interpret this characteristic of the client? A) Slight delays in mental processing are normal in older adults. B) The client may be trying to anticipate the nurse's desired response. C) The client is displaying a sign of early Alzheimer's disease. D) The client may be experiencing an early sign of delirium.

A) Slight delays in mental processing are normal in older adults.

25. The nurse is assessing a 79-year-old client's posterior thorax during a focused respiratory assessment. The nurse should attribute what assessment finding to age- related changes? A) Slight kyphosis B) Inaudible posterior lung sounds C) Audible wheeze D) Asymmetrical chest expansion

A) Slight kyphosis

22. An adult client has asked the nurse about actions that she can take to reduce her future risk of stroke. What health promotion activity should the nurse prioritize? A) Smoking cessation B) Annual MRI screening C) Nutritional supplementation D) Improved coping skills

A) Smoking cessation

30. The nurse is inspecting a pregnant client's cervix during a prenatal clinic visit. What is an expected assessment finding? A) Smooth cervix with a bluish tint B) Slightly rough cervix with dark pink coloration C) Pink or red cervix with small, visible nodes D) Smooth cervix with small amounts of creamy white discharge

A) Smooth cervix with a bluish tint

1. When assessing whispered pectoriloquy, the nurse should instruct a client to do which of the following? A) Softly repeat the words one-two-three. B) Say the number ninety-nine. C) Cough each time the stethoscope is moved. D) Say the letter e until instructed to stop.

A) Softly repeat the words one-two-three.

7. A nurse is preparing to begin work in a diverse, urban community with members of numerous different religious traditions. The nurse should identify which statement as best reflective of Buddhism? A) Some holy days include fasting from dawn to dusk. B) The soul has no beginning or end. C) Outcomes are predetermined. D) Beliefs focus around the Koran.

A) Some holy days include fasting from dawn to dusk.

8. A male client is receiving chemotherapy for the treatment of cancer. Which finding should the nurse anticipate during examination of the client's genitalia? A) Sparse pubic hair B) Hardness along the ventral surface of the penis C) Cyanosis to the glans D) Tenderness on scrotal palpation

A) Sparse pubic hair

4. A nurse states, Hispanic people have no clue about primary prevention of illness. The nurse is demonstrating which of the following? A) Stereotyping B) Ethnicity C) Cultural incompetence D) Prejudice

A) Stereotyping

8. Which test would be most appropriate for the nurse to perform when a client complains of low back pain? A) Straight leg test B) Muscle leg strength C) Lateral bending of cervical spine D) Internal rotation of the shoulders

A) Straight leg test

26. A nurse should conduct an assessment of a client's Risk for Complications after gathering data related to the client's spirituality. When planning the client's care, the nurse should be aware that complications are primarily due to the effect of spirituality on what phenomenon? A) Stress B) Pain C) Worry D) Emotional lability

A) Stress

2. When doing a family assessment, what components are most essential for the nurse to include? A) Structure, development, and function B) Development, boundaries, and culture C) Communication, expectations, and strengths D) Socialization, development, and problems

A) Structure, development, and function

5. A nurse has documented the nursing history and physical examination of a client. This health information is best described as which of the following? A) Subjective data and objective data B) Interpretation and inference C) Observation and inspection D) Data and results

A) Subjective data and objective data

22. Long-term care facilities in a community would be deemed adequate if which of the following situations exists? A) Sufficiently specialized care is available for local residents who cannot live independently. B) Residents with cognitive disorders are admitted to inpatient hospital units during exacerbations of symptoms. C) Older adults are able to live in structured settings without having to pay. D) Older adults are able to live in their own homes regardless of health status.

A) Sufficiently specialized care is available for local residents who cannot live independently.

19. During a prenatal visit, the nurse inspects the skin of the client's abdomen. Which of the following would the nurse identify as an abnormal finding? A) Superficial bruising B) Linea nigra C) Striae D) Darkening of the umbilicus

A) Superficial bruising

4. An infant was removed from her home by social services because of the dangerous and neglectful conditions that existed. According to Erikson, failure of the infant to resolve the central crisis of infancy may lead to what personality characteristics later in life? A) Suspicion and fear B) Aggression and antagonism C) Dependency and relational entanglement D) Depression and introversion

A) Suspicion and fear

2. The nurse's assessment reveals that a client is in a low percentile for midarm muscle circumference (MAMC) and a high percentile for triceps skin fold (TSF) thickness. Which of the following would be appropriate? A) Teaching the client muscle-building exercises B) Discussing ways to increase body fat stores C) Assisting client in reducing the amount of fluid build-up D) Encouraging the use of a multivitamin supplement

A) Teaching the client muscle-building exercises

3. The nurse is preparing to assess a client's vital signs. Which vital sign should the nurse assess first? A) Temperature B) Pulse C) Respiration D) Blood pressure

A) Temperature

4. While the nurse is interviewing a client who is a victim of abuse, the client states that she blames herself for not satisfying her husband's demands. Applying Walker's Cycle of Violence theory, the client would be in which of the following phase? A) Tension-building phase B) Acute battering phase C) Honeymoon phase D) Reconciliation phase

A) Tension-building phase

25. The nurse is completing an abbreviated head-to-toe assessment of a client. Which of the following should the nurse perform when assessing the client's eyes? A) Test the client's pupillary response to light. B) Test the client's visual fields. C) Perform the cover test. D) Test the client's vision.

A) Test the client's pupillary response to light.

17. A nurse is interviewing an elderly client and begins the interview by evaluating the client's mental status. The nurse does this based on an understanding of which of the following? A) The aging brain is more easily affected by pathology. B) Older clients have decreased intellectual capacity. C) The brain is the last organ to experience an age-related decline. D) The client is always the most reliable person to provide the data.

A) The aging brain is more easily affected by pathology.

21. A pediatric nurse is assessing a 7-year-old boy who is suspected of being the victim of psychological abuse by his stepfather. What criterion would the nurse use to determine whether the stepfather's actions constitute abuse? A) The behavior is a threat to the child's well-being. B) The child expresses a dislike for his stepfather. C) The behavior is not aimed at fostering the boy's growth and development. D) The child states that he gets punished if he does not behave appropriately.

A) The behavior is a threat to the child's well-being.

18. What assessment finding would most clearly suggest to the nurse that a young adult client has failed to attain normal development within Piaget's framework? A) The client has difficulty understanding abstract reasoning in written form. B) The client has a recent history of tumultuous interpersonal relationships. C) The client is often defiant toward authority figures. D) The client is unwilling to accept responsibilities in the workplace.

A) The client has difficulty understanding abstract reasoning in written form.

2. When preparing to do a comprehensive health assessment, the nurse obtains the client's permission based on an understanding of which of the following principles? A) The client has the right to refuse the assessment. B) Obtaining permission enhances therapeutic rapport. C) The client will be more willing to disclose after giving permission. D) The client's level of comfort will be increased by granting explicit consent.

A) The client has the right to refuse the assessment.

29. The nurse is completing a client's ear assessment. What assessment finding would indicate the need to perform Weber's test? A) The client has unilateral hearing loss. B) The client has suspected otitis externa. C) The client is older than age 65. D) The client has a history of stroke.

A) The client has unilateral hearing loss.

26. The nurse is assessing the gastrointestinal system of an 81-year-old client. What age- related change should the nurse consider when collecting and analyzing assessment data? A) The client is more vulnerable to impaired nutrition due to decreased appetite. B) The client derives less nutritional value from food because of decreased enzyme production. C) The client's liver will be significantly larger than that of a younger client. D) The client will have greater bowel motility than a younger adult.

A) The client is more vulnerable to impaired nutrition due to decreased appetite.

27. A nurse in the emergency department is utilizing the SAD PERSONAS assessment guide during the mental status assessment of a client. What is the most likely rationale for the nurse's choice of this assessment tool? A) The client may have a high risk for suicide. B) The client may have major depression. C) The client may have schizophrenia or psychosis. D) The client may be using alcohol excessively.

A) The client may have a high risk for suicide.

27. The nurse is performing the Romberg test as part of a client's focused neurological assessment. What finding would constitute a positive Romberg test? A) The client moves her feet apart to prevent herself from falling. B) The client is unable to consistently touch her finger to her nose while her eyes are close. C) The client experiences pain during neck flexion and extension. D) The client experiences pain when clenching her teeth.

A) The client moves her feet apart to prevent herself from falling.

11. The nurse is planning to assess a client for graphesthesia. How will the nurse perform this phase of assessment? A) The client will close the eyes and identify what number the nurse writes in the palm of the client's hand with a blunt-ended object. B) The client is asked to identify the number of points felt when the nurse touches the client with the ends of two applicators at the same time. C) The nurse will simultaneously touch the client in the same area on both sides of the body, and the client will identify where the touch occurred. D) The nurse will briefly touch the client, and the client will identify where the touch occurred.

A) The client will close the eyes and identify what number the nurse writes in the palm of the client's hand with a blunt-ended object.

4. A client has sustained a brain stem injury and is being treated in the intensive care unit. Which of the following would the nurse need to consider when assessing this client's respiratory status? A) The client will have a loss of involuntary respiratory control. B) The client will respond negatively to increased stimuli. C) The client will have greatly increased respiratory effort. D) The client will exhibit Cheyne-Stokes respirations.

A) The client will have a loss of involuntary respiratory control.

5. When analyzing data related to a client's behavior, the nurse should compare the observations with which of the following? A) The client's developmental stage B) The client's motivation for change C) The client's body mass index D) The client's vital signs

A) The client's developmental stage

19. A community health nurse is assessing an older adult client in the client's home. When the nurse is gathering subjective data, which of the following would the nurse identify? A) The client's feelings of happiness B) The client's posture C) The client's affect D) The client's behavior

A) The client's feelings of happiness

6. The nurse is analyzing the data obtained from a client interview. When applying the principles of Kohlberg's theory of development, the nurse should prioritize data related to what domain? A) The client's moral behavior B) The client's relationships C) The client's health D) The client's sexual identity

A) The client's moral behavior

27. The nurse is utilizing the Health Belief Model in the care of a client whose type 1 diabetes is inadequately controlled. When implementing this model, the nurse should begin by assessing which of the following? A) The client's motivation for change B) The client's medical comorbidities C) The client's learning style D) The client's prognosis for recovery

A) The client's motivation for change

28. The nurse is using the COLDSPA mnemonic during the client's head-to-toe assessment. This tool will allow the nurse to address what component of assessment? A) The client's present health concern B) The review of the client's body systems C) The client's personal health history D) The client's health practices profile

A) The client's present health concern

8. The nurse is preparing to assess the mental status of a 90-year-old client who is being admitted to the hospital from a long-term care facility. Which of the following should the nurse assess first? A) The client's sensory abilities B) The client's general intelligence C) The presence of any phobias D) The client's judgment and insight

A) The client's sensory abilities

28. The nurse's auscultation of the client's heart sounds reveals the presence of a split S1. What conclusion should the nurse draw from this assessment finding? A) The client's ventricles are not contracting simultaneously. B) The client's aortic valve is incompetent. C) The client has left ventricular hypertrophy. D) The client's atria are not synchronized with the ventricles.

A) The client's ventricles are not contracting simultaneously.

23. The nurse has begun the objective assessment of a client's heart and neck vessels and is assessing the client's jugular veins. What finding would the nurse consider to be normal in a healthy client? A) The jugular venous pulse is not visible when the client is sitting upright. B) The jugular veins are fully distended when the client is in a high Fowler's position. C) The jugular veins are distended when the client sits at 45 degrees. D) The jugular venous pulse is visible when the client lies supine.

A) The jugular venous pulse is not visible when the client is sitting upright.

13. The nurse is assessing a client who is suspected of having an incarcerated scrotal hernia. Which finding would help confirm this suspicion? A) The mass cannot be pushed up into the abdomen. B) The area around the hernia is ecchymotic. C) The client complains of tenderness and nausea. D) A scrotal bulge disappears when the client lies down.

A) The mass cannot be pushed up into the abdomen.

21. The nurse is preparing to perform a focused respiratory assessment on a client. The nurse should be cognizant of what anatomical characteristic of the lungs? A) The right lung has three lobes, while the left lung has two lobes. B) The lungs are structurally symmetrical but functionally differently. C) The right lung is approximately one-third larger than the left lung. D) The lower lobes of both lungs are primarily located toward the anterior chest wall.

A) The right lung has three lobes, while the left lung has two lobes.

1. A nurse is performing an assessment within the legal parameters of assessment and diagnosis. These legal guidelines would be specified in which of the following? A) The state's Nurse Practice Act B) The client's informed consent documents C) The nurse's terms of license D) The institution's policies and procedures guidelines

A) The state's Nurse Practice Act

25. A nurse is providing care for a client who has decreased mobility secondary to a recent stroke. Which of the following assessment findings would be indicative of a stage I pressure ulcer? A) There is a nonblanching reddened area on the client's coccyx region. B) There is scant, frank blood present on the skin surfaces surrounding the client's coccyx. C) There is noticeable bruising on and around the client's coccyx region. D) There is a generalized rash on the client's lower back and buttocks.

A) There is a nonblanching reddened area on the client's coccyx region.

19. A community nurse's assessment of a rural community reveals the presence of more cultural diversity than was previously thought. The presence of differences between cultures may have what effect on the health of the community? A) There may be a potential for conflicting views. B) There may be an unwillingness to change. C) There may be a lack of health services. D) There may be worse health outcomes than in a homogenous community.

A) There may be a potential for conflicting views.

29. A medical nurse is preparing to administer a topical antifungal medication to a client who has just been diagnosed with an oral candida infection (thrush). On inspection of the patient's tongue, the nurse should anticipate what appearance? A) Thick, white plaques on the tongue surface B) Dry appearance with fissures present C) Diffuse reddened lesions that bleed easily D) Firm, raised nodules that are pink or red

A) Thick, white plaques on the tongue surface

2. A new mother asks the nurse, What are those small white spots on my baby's nose? Which response by the nurse would be most appropriate? A) Those are small glands that look like whiteheads but will disappear soon. B) Those white spots are lesions containing pus and are caused by a minor skin infection. C) Newborns retain sweat, which causes those white bumps on their skin. D) Often newborns have a rash of this type, which fades in a few days.

A) Those are small glands that look like whiteheads but will disappear soon.

17. The nurse is assessing a newborn's rooting reflex. What action should the nurse perform during this assessment? A) Touch the infant's lip or cheek with a gloved finger. B) Place a gloved finger in the newborn's mouth. C) Touch the ball of the newborn's foot. D) Hit the surface near where the newborn is lying.

A) Touch the infant's lip or cheek with a gloved finger.

24. A client has just received a positive pregnancy test and is now discussing health promotion activities with the nurse. The client states, "I know I'm supposed to start gaining weight, but how much is healthy?" The nurse should tell the client that she should aim to gain how much weight during the first trimester? A) Two to four pounds B) Six to ten pounds C) Eight to twelve pounds D) Five percent of her normal body weight

A) Two to four pounds

27. A community health nurse is conducting a home visit to a client who requires wound care. The nurse observes that the client is diaphoretic and wishes to measure the client's temperature. The nurse asks if the client has a thermometer in her home, and she states that she owns an ear thermometer. What principle should guide the nurse's use of a tympanic thermometer? A) Tympanic temperature is slightly higher than oral temperature. B) Tympanic temperature is only used if all other methods are unavailable. C) Tympanic temperature varies more widely than oral, rectal, and axillary temperatures. D) In adults, tympanic temperature is equal to axillary temperature.

A) Tympanic temperature is slightly higher than oral temperature.

26. The nurse is performing a peripheral vascular assessment of an adult client. The nurse is palpating the client's peripheral pulses but knows that some are not palpable, even in healthy clients. What pulse is not palpable in a large proportion of healthy clients? A) Ulnar B) Radial C) Brachial D) Femoral

A) Ulnar

5. A postmenopausal woman tells the nurse that she experiences discomfort during sexual intercourse. Which of the following should the nurse suggest? A) Use of a lubricant B) Abstinence from intercourse C) Use of a condom by the partner D) Kegel exercises

A) Use of a lubricant

13. The nurse is preparing to palpate the breasts of a female client. Which technique should the nurse utilize during this aspect of assessment? A) Use the flat pads of three fingers. B) Use the fingertips of both hands. C) Gently pinch the skin between two fingers. D) Use the palm of one hand.

A) Use the flat pads of three fingers.

4. The emergency department has collected extensive data from a client who has presented with a new onset of severe abdominal pain. What nursing action should the nurse perform before proceeding with data analysis? A) Validate the collected data. B) Formulate a nursing diagnosis. C) Make inferences about the data. D) Identify the client's strengths.

A) Validate the collected data.

24. The nurse is conducting an assessment of an adult client who describes herself as being in good health. Inspection of the client's nail beds reveals the presence of a bluish tone. The nurse should recognize that this finding is most likely attributable to what phenomenon? A) Vasoconstriction B) Hyperglycemia C) Hypoxemia D) Cardiopulmonary insufficiency

A) Vasoconstriction

28. The nurse's inspection of a Caucasian client's lower extremities reveals a brownish coloration to the client's ankles and shins. The nurse should perform further assessments that address what health problem? A) Venous insufficiency B) Peripheral edema C) Coronary artery disease D) Raynaud's phenomenon

A) Venous insufficiency

5. The nurse refers an older adult client for further evaluation after the nurse assesses warm skin and brown pigmentation around the ankles. The nurse should note the possibility of what health problem when making the referral? A) Venous insufficiency B) Stasis ulceration C) Arterial occlusion D) Dependent edema

A) Venous insufficiency

2. When reviewing the medications currently taken by a 50-year-old client who is complaining of constipation, teaching is indicated when the nurse notes which medication? A) Vitamin supplement with iron B) Nonsteroidal anti-inflammatory drug C) Antidepressant D) Hormone replacement

A) Vitamin supplement with iron

28. The nurse is assessing a dark-skinned client whose forearms are hands have distinct regions of depigmentation. The nurse should document the presence of what health problem? A) Vitiligo B) Striae C) Angiomas D) Albinism

A) Vitiligo

11. The nurse is performing a head-to-toe assessment of a client. Which of the following would be an example of information obtained during the review of the client's body systems? A) Wears dentures; denies problems with eating, chewing, and swallowing. B) States her father died of a heart attack at age 70. C) Uses over-the-counter antacid for occasional heartburn. D) Vaginal delivery of two children without complications.

A) Wears dentures; denies problems with eating, chewing, and swallowing.

23. The nurse is assessing a family's function within the domain of health care. What assessment question best addresses this area of function? A) What do you think would best improve your health? B) Who is your regular family doctor? C) Is anyone in your family a health professional? D) When was your father first diagnosed with heart disease?

A) What do you think would best improve your health?

13. The nurse is assessing a client's pain. Which question would be most appropriate to ask the client when the goal is to identify precipitating factors that might have exacerbated the pain? A) What were you doing when the pain first stated? B) Do concurrent symptoms accompany the pain? C) When did the pain start? D) Is the pain continuous or intermittent?

A) What were you doing when the pain first stated?

29. The nurse is assessing a client's judgment during a comprehensive head-to-toe assessment. How can the nurse best appraise this aspect of cognitive function? A) What would you do if you found a stamped, addressed envelope on the ground? B) What kinds of activities do you do to improve your health? C) Who is the most important person in your life, and why? D) What is your idea of the ideal vacation?

A) What would you do if you found a stamped, addressed envelope on the ground?

1. A palliative care nurse is explaining the basis of pain to a group of nurses who provide care on a general medical unit. Which of the following factors would the nurse include? Select all that apply. A) Physiologic B) Psychosocial C) Cutaneous D) Somatic E) Visceral

A, B A) Physiologic B) Psychosocial

27. The nurse is examining a 4-year-old girl who is being treated for a burn. When determining whether the burn may be the result of abuse, what assessment parameters should the nurse consider? Select all that apply. A) The presence of other scars on the child's skin B) The pattern or shape of the burn C) The location of the burn D) The child's explanation of how she got the burn E) The child's prognosis for recovering from the burn

A, B, C, D A) The presence of other scars on the child's skin B) The pattern or shape of the burn C) The location of the burn D) The child's explanation of how she got the burn

14. A nurse is preparing an in-service education program for a group of staff nurses about documentation, including documentation of assessment data. The nurse demonstrates understanding of the significance of documentation by including a discussion of which of the following as playing a role in this area? Select all that apply. A) Joint Commission B) State nurse practice act C) Medicare D) Local or city government E) Institutional agency

A, B, C, E A) Joint Commission B) State nurse practice act C) Medicare E) Institutional agency

16. The nurse is assessing the diet and nutritional status of a client from a different culture. Which of the following questions would be appropriate for the nurse to ask? Select all that apply. A) What foods do you commonly eat? B) Do you have any special routines for eating? C) Are there any foods that you can't eat? D) Do you eat three meals a day? E) Do you have certain foods to keep you healthy?

A, B, C, E A) What foods do you commonly eat? B) Do you have any special routines for eating? C) Are there any foods that you can't eat? E) Do you have certain foods to keep you healthy?

1. A nurse has completed the general survey of a client who has been transferred to the unit. The information gathered during the general survey primarily provides the nurse with which of the following? Select all that apply. A) An indication of the level of physical distress experienced by the client B) Clues about the overall health of the client C) A direct link to the client's medical diagnosis D) Indications about normal variations in the status of body systems E) Data relating to the patient's level of social support

A, B, D A) An indication of the level of physical distress experienced by the client B) Clues about the overall health of the client D) Indications about normal variations in the status of body systems

17. A nurse is preparing an in-service program about spirituality and religion for a group of colleagues. When describing the effects on clients of religion and spirituality, which of the following should the nurse include? Select all that apply. A) Improved client sense of well-being B) Enhanced coping with end-of-life issues C) Increased mortality levels D) Increased timely use of health care E) Increased adherence to medical regimens

A, B, E A) Improved client sense of well-being B) Enhanced coping with end-of-life issues E) Increased adherence to medical regimens

13. A woman brings her 69-year-old husband to the clinic for an evaluation because he has become increasingly forgetful. Which of the following would lead the nurse to suspect that the client has Alzheimer's disease? Select all that apply. A) He repeats the same story, word for word, over and over again. B) He took a fall when he was replacing a light bulb last month. C) I have to balance the checkbook now because he just won't do it. D) If I don't tell him when to shower, he won't and will fight me on it. E) He got lost walking to the pharmacy around the corner the other day.

A, C, D, E A) He repeats the same story, word for word, over and over again. C) I have to balance the checkbook now because he just won't do it. D) If I don't tell him when to shower, he won't and will fight me on it. E) He got lost walking to the pharmacy around the corner the other day.

24. The nurse is evaluating the setting prior to beginning a client's physical examination. The nurse should confirm the presence of which of the following? Select all that apply. A) Adequate lighting B) Cool room temperature C) Quiet surroundings D) Soft chair or table E) Table for equipment F) Door or curtain

A, C, E, F A) Adequate lighting C) Quiet surroundings E) Table for equipment F) Door or curtain

21. The nurse is reviewing a client's health history and the results of the most recent physical examination. Which of the following data would the nurse identify as being subjective? Select all that apply. A) I feel so tired sometimes. B) Weight: 145 lbs C) Lungs clear to auscultation D) Client complains of a headache E) My father died of a heart attack. F) Pupils equal, round, and reactive to light

A, D, E

26. The nurse is completing a review of systems for a client. Which of the following information would the nurse document related to the client's musculoskeletal system? Select all that apply. A) Joint stiffness B) Rhinorrhea C) Shortness of breath D) Chest pain E) Muscle strength F) Knee swelling

A, E, F A) Joint stiffness E) Muscle strength F) Knee swelling

7. A 42-year-old female client says she does not perform breast self-examination because she believes that mammograms are more thorough. Which response by the nurse would be most appropriate? A) "You should do the exam. It's the best way to detect breast cancer early." B) "Be sure to have your breasts checked by a doctor and have a mammogram every year." C) "Mammograms don't always detect the lumps that you might feel." D) "Once you hit age 50, you really won't have a choice about doing them."

B) "Be sure to have your breasts checked by a doctor and have a mammogram every year."

21. During the interview, the client states, "Is today the 12th? My wife died 2 months ago today." Which of the following responses would be most appropriate? A) "What was the cause of your wife's death?" B) "How does that make you feel right now?" C) "You probably must be sad." D) "Are you feeling sad, depressed, angry, or upset?"

B) "How does that make you feel right now?"

16. During a prenatal class, a nurse teaches a client how to measure the frequency of contractions. The client demonstrates understanding with which statement? A) "I should time from when I feel the contraction to the end of the contraction." B) "I'll start timing when I feel one starting until I feel another one starting." C) "I should start timing when the contraction is the strongest until it subsides." D) "I should time from when one contraction ends and another one starts."

B) "I'll start timing when I feel one starting until I feel another one starting."

28. A client expresses frustration that the nurse is assessing his spirituality, stating, "I thought I was here to have my tumor removed, not to figure out what I believe or don't believe about God." How should the nurse best justify the need for a spiritual assessment? A) "It's important that we plan to make sure that we don't offend you." B) "Spirituality actually has a significant effect on your overall health." C) "We need to make plans in case there are unexpected outcomes of your surgery." D) "Your beliefs determine whether we will focus more on your body or on your spirit."

B) "Spirituality actually has a significant effect on your overall health."

18. A nurse on a busy acute medical unit asks a clinical educator for suggestions on how to best develop expertise in using diagnostic reasoning skills to arrive at correct conclusions. Which of the following statements would be most appropriate? A) "You need to cluster the data more rapidly." B) "This skill comes with accumulating experience." C) "Try to be more efficient in documenting the data." D) "This is a skill that only comes with an advanced practice designation."

B) "This skill comes with accumulating experience."

28. A 60-year-old woman with a bunion will undergo surgery later today. The client tells the nurse in the surgical daycare admitting department, "I'm sure I've been asked these questions before. Can't we just focus on my foot and not all these other topics?" How should the nurse best explain the rationale for obtaining a health history? A) "In general, it's necessary for us to gather as much information about each client as possible." B) "We want to make sure your nursing care matches your needs as closely as possible." C) "The care team needs to cross-reference your diagnostic testing with the information that I'm asking you about." D) "We don't want to make the mistake of focusing solely on the medical problem that brought you here."

B) "We want to make sure your nursing care matches your needs as closely as possible."

18. A nurse is completing a comprehensive assessment of a client who has been referred to the clinic. Which of the following would be most appropriate for the nurse to ask when beginning to assess the client's spirituality? A) "What religion are you?" B) "What gives you hope or peace?" C) "Do you believe in God?" D) "Would you like to speak to a chaplain?"

B) "What gives you hope or peace?"

28. The nurse is palpating the axillary lymph nodes of a client who has been experiencing recent malaise. The nurse should consider a lymph node to be enlarged if its diameter exceeds what size? A) 0.5 cm B) 1 cm C) 2 cm D) 2.5 cm

B) 1 cm

12. A nurse weighs a client today and finds that the client's weight has increased 2.2 lbs from the previous day. The nurse interprets this finding as suggesting a fluid gain of which amount? A) 0.5 liters B) 1.0 liters C) 1.5 liters D) 2.0 liters

B) 1.0 liters

25. During an initial prenatal visit, the nurse is performing a nutritional assessment of a woman who has just learned that she is pregnant for the first time. The nurse has determined that the client has an average stature and is 5 feet, 3 inches tall. What is this client's ideal body weight? A) 105 lbs. B) 115 lbs. C) 125 lbs. D) 135 lbs.

B) 115 lbs.

27. The clinic nurse is assessing a client who is pregnant at 18 weeks' gestation. The nurse is obtaining a fetal heart rate using Doppler ultrasound. What fetal heart rate represents an expected finding? A) 90 beats per minute B) 130 beats per minute C) 175 beats per minute D) 225 beats per minute

B) 130 beats per minute

18. The nurse assesses a client's carotid pulse and finds it to be of normal amplitude. The nurse would document this as which of the following? A) 1+ B) 2+ C) 3+ D) 4+

B) 2+

17. The nurse assesses the respiratory rate of a 5-year-old boy. Which finding would indicate to the nurse that his rate is within the age-appropriate range for this child? A) 16 breaths/minute B) 24 breaths/minute C) 32 breaths/minute D) 40 breaths/minute

B) 24 breaths/minute

4. A client has received a diagnosis of chronic nonmalignant pain. The nurse who is planning this client's nursing care should understand that this client has experienced this pain for at least how many months? A) 3 B) 6 C) 9 D) 12

B) 6

19. Due to a change in the client's level of consciousness, a nurse is now assessing a client's temperature by the axillary route. Previously, the client had an oral temperature of 98.4∫F. Which finding would the nurse interpret as corresponding most closely to the client's previous temperature? A) 97.0 F B) 97.4 F C) 98.9 F D) 99.4 F

B) 97.4 F

9. The nurse is working in an ambulatory care clinic that is located in a busy, inner-city neighborhood. Which client would the nurse determine to be in most need of an emergency assessment? A) A 14-year-old girl who is crying because she thinks she is pregnant B) A 45-year-old man with chest pain and diaphoresis for 1 hour C) A 3-year-old child with fever, rash, and sore throat D) A 20-year-old man with a 3-inch shallow laceration on his leg

B) A 45-year-old man with chest pain and diaphoresis for 1 hour

13. The nurse at a busy primary care clinic is analyzing the data obtained from the following clients. For which clients would the nurse most likely expect to facilitate a referral? A) An 80-year-old client who lives with her daughter B) A 50-year-old client newly diagnosed with diabetes C) An adult presenting for an influenza vaccination D) A teenager seeking information about contraception

B) A 50-year-old client newly diagnosed with diabetes

30. A nurse practitioner refers clients for osteoporosis screening according to the latest U.S. Preventive Services Task Force (USPSTF) recommendations. According to these recommendations, what client should be screen for osteoporosis? A) A 71-year-old man who has type 2 diabetes B) A 69-year-old woman with no major risk factors for osteoporosis C) A 37-year-old woman who takes oral contraceptives D) A 49-year-old African-American woman who is obese

B) A 69-year-old woman with no major risk factors for osteoporosis

26. In which of the following clients would the nurse consider a bluish tint to the cervix an expected assessment finding? A) A client who is 17 years old and sexually active. B) A client who is 10 weeks' pregnant. C) A 71-year-old multiparous client D) A client who has a 24 pack-year smoking history.

B) A client who is 10 weeks' pregnant.

24. The nurse is performing a community health assessment. When performing data collection, the nurse should seek out what informants? A) A combination of educational leaders and political leaders B) A combination of community leaders and "typical" residents C) The residents who use health services most often D) The residents who work in the health care industry

B) A combination of community leaders and "typical" residents

3. The nurse's assessment reveals that a male client can neither turn his head against resistance nor shrug his shoulders. The nurse should document a potential deficit in the functioning of which cranial nerve? A) Abducens (VI) B) Accessory (XI) C) Hypoglossal (XII) D) Trochlear (IV)

B) Accessory (XI)

30. An older adult client with osteoarthritis has tearfully admitted to the nurse that she is no longer able to climb the stairs to the second floor of her house due to her knee pain. What nursing diagnosis is suggested by this client's statement? A) Ineffective coping related to knee pain B) Activity intolerance related to knee pain C) Ineffective role performance related to osteoarthritis D) Situational low self-esteem related to osteoarthritis

B) Activity intolerance related to knee pain

12. A nurse is preparing to document conclusions after analyzing data, and he or she includes information about related factors and manifestations. The nurse is formulating which of the following? A) Risk nursing diagnosis B) Actual nursing diagnosis C) Collaborative problem D) Problem for referral

B) Actual nursing diagnosis

21. A nurse is providing care for a client who has hepatic encephalopathy secondary to chronic alcohol abuse. The nurse's assessment reveals that the client often provides incorrect answers to assessment questions. As well, the client makes statements that are not grounded in reality. What nursing diagnosis is suggested by these assessment data? A) Impaired Verbal Communication related to hepatic encephalopathy AMB confusion B) Acute Confusion related to hepatic encephalopathy C) Ineffective Health Maintenance related to alcohol abuse AMB decreased cognition D) Ineffective Coping related to alcohol abuse

B) Acute Confusion related to hepatic encephalopathy

20. Assessment of a client's nails reveals the presence of Beau's lines. The nurse interprets this finding as suggestive of which of the following? A) Oxygen deficiency B) Acute illness C) Psoriasis D) Trauma

B) Acute illness

9. Which child should the pediatric nurse suspect of having a developmental delay? A) A 5-month-old who does not sit unsupported B) An 11-month-old who does not pull himself to a standing position C) A 3-month-old who cannot grasp an object voluntarily D) A 12-month-old who cannot build a tower of eight blocks

B) An 11-month-old who does not pull himself to a standing position

20. A community health nurse is aware of the important role of primary care. Which of the following would be an example of a primary care facility? A) A medical laboratory B) An ambulatory clinic C) An obstetrician's office D) An intensive care unit

B) An ambulatory clinic

17. The nurse observes a client's entire body posture to be somewhat stiff, with his shoulders elevated upward toward the ears. The nurse would most likely interpret this to indicate that the client is experiencing which of the following? A) Confusion B) Anxiety C) Powerlessness D) Restlessness

B) Anxiety

14. A nurse should assess the client's epitrochlear lymph nodes when assessing which of the following? A) Neck B) Arms C) Posterior chest D) Sinuses

B) Arms

18. A nurse is preparing to assess the cranial nerves of a client. The nurse is about to test CN I. Which of the following should the nurse do? A) Use a Snellen chart to test visual acuity. B) Ask a client to identify scents. C) Test extraocular eye movements. D) Perform the Weber test.

B) Ask a client to identify scents.

30. A client who is new to the facility has a recent history of chronic pain that is attributed to fibromyalgia. The nurse has reviewed the available health records and suspects that pain management will be a major focus of nursing care. How can the nurse best validate this assumption? A) Review the client's medication administration record for analgesic use. B) Ask the client about the most recent experiences of pain. C) Meet with the client's spouse and daughter to discuss the client's pain. D) Collaborate with the physician who is treating the client.

B) Ask the client about the most recent experiences of pain.

23. The nurse is interviewing an adult client in the context of a focused mouth, nose, sinus, and throat assessment. After asking the client about his history of environmental allergies, the client states, I'm pretty sure that I'm allergic to something, but I'm not exactly sure what triggers my allergies. How can the nurse begin to identify the specific allergens that cause the man's symptoms? A) Ask the client if his allergies respond to OTC antihistamines. B) Ask the client about the timing of his allergy symptoms. C) Perform a detailed inspection of the client's ears and throat using an otoscope. D) Perform transillumination of the client's sinuses.

B) Ask the client about the timing of his allergy symptoms.

17. During an assessment, the nurse determines that a client sees more than one primary care provider and has obtained prescriptions from each provider. Which method would be most appropriate to determine a client's current medication regimen? A) Ask the client to identify which medications taken every day. B) Ask the client to bring all the medications and supplements to an interview. C) Ask the caregiver whether the client is taking prescribed medications. D) Ask the client about the use of any over-the-counter medications.

B) Ask the client to bring all the medications and supplements to an interview.

24. The nurse has completed the assessment of a client's breast and lymphatic system. The nurse has ended the assessment by offering to teach the client how to perform breast self-examination (BSE). The client states, "That's alright. I already know how to do that." What should the nurse do next? A) Encourage the client to perform BSE as often as possible. B) Ask the client to demonstrate BSE. C) Encourage the client to promote BSE to her peers. D) Reiterate the correct technique for BSE.

B) Ask the client to demonstrate BSE.

13. When testing muscle strength, a client has difficulty moving her right arm against resistance. Which of the following should the nurse do next? A) Move the part passively through its range of motion. B) Ask the client to move the part against gravity. C) Inspect by touch for a palpable contraction of the muscle. D) Percuss the client's shoulder joint

B) Ask the client to move the part against gravity.

24. The nurse is performing an abbreviated head-to-toe assessment of a client. When the nurse asks the client about his pain, the client states, My stomach's really killing me right now. How should the nurse first respond to this client's statement? A) Offer analgesia to the client B) Ask the client to rate his pain on a 0-to-10 scale C) Assess the client's level of consciousness D) Assure the client that his pain will be addressed immediately following the assessment

B) Ask the client to rate his pain on a 0-to-10 scale

24. The nurse is attempting to cluster the data that she collected during the initial assessment of an older adult client. The nurse notes that the client had a swollen left knee and complained of "a bit of soreness" in the joint, but the nurse does not have enough data to support a nursing diagnosis of Impaired Physical Mobility. What should the nurse do next? A) Document a suspected nursing diagnosis of Impaired Physical Mobility. B) Assess the client further for evidence of reduced mobility and decreased range of motion. C) Make a referral to the physical therapist. D) Plan interventions that will conservatively manage the client's joint dysfunction.

B) Assess the client further for evidence of reduced mobility and decreased range of motion.

27. A nurse is admitting a client to the postsurgical unit from the postanesthetic care unit. The nurse has transferred the client from the stretcher to a bed and asked the client if he is experiencing pain. The client acknowledges that he is in pain. What should be the nurse's next action? A) Ask the client to briefly explain his cultural background. B) Assess the client's pain according to COLDSPA. C) Assess the client's self-management skills. D) Assess the client's pain by obtaining a set of vital signs.

B) Assess the client's pain according to COLDSPA.

14. A client at 32 weeks' gestation, who has had regular prenatal care, is found to have gained 6 pounds in 1 week. Which of the following would be most appropriate for the nurse to do next? A) Ask for 24-hour diet recall. B) Assess the legs for edema. C) Collect a urine culture. D) Check fundal height.

B) Assess the legs for edema.

1. A client tells the clinic nurse that she has sought care because she has been experiencing excessive tearing of her eyes. Which assessment should the nurse next perform? A) Inspect the palpebral conjunctiva. B) Assess the nasolacrimal sac. C) Perform the eye positions test. D) Test pupillary reaction to light.

B) Assess the nasolacrimal sac.

27. The nurse is planning the comprehensive head-to-toe assessment of a client. What assessment should the nurse usually conduct last? A) Assessment of the abdomen B) Assessment of the genitalia and rectum C) Assessment of the lower extremities D) Assessment of the posterior thorax

B) Assessment of the genitalia and rectum

11. A client has sought care because he is concerned that a mole on his scalp may be evidence of skin cancer. During assessment using the mnemonic ABCDE, which finding would the nurse identify as being most suggestive of melanoma? A) Solid, dark brown color B) Asymmetric, irregular borders C) Diameter of 3 mm D) Flat with silvery scales

B) Asymmetric, irregular borders

14. A client's bladder is found to be distended. At which location should the nurse begin palpating? A) At the umbilicus B) At the symphysis pubis C) In the right lower quadrant D) In the left lower quadrant

B) At the symphysis pubis

9. A school nurse plans to test hearing acuity in students who range between kindergarten and sixth grade. Which of the following would be most appropriate method? A) Loud noise screening B) Audiometry C) Whisper test D) Weber test

B) Audiometry

26. A client has been recovering from surgery in the hospital, and the nurse is beginning a shift by conducting an abbreviated head-to-toe assessment. How should the nurse assess the client's bowel sounds? A) Auscultate for 2 to 3 minutes in the client's right upper abdominal quadrant. B) Auscultate for bowel sounds in each of the client's four abdominal quadrants. C) Auscultate for 5 minutes to confirm the presence of consistent bowel sounds. D) Auscultate to determine which quadrant contains the most active bowel sounds.

B) Auscultate for bowel sounds in each of the client's four abdominal quadrants.

28. A client has sought care because of a sudden increase in the size of his scrotum. The nurse's assessment reveals the presence of a large scrotal mass. How can the nurse best assess for a scrotal hernia? A) Palpate the mass for pain. B) Auscultate the mass for bowel sounds. C) Percuss the mass for dullness. D) See if the mass disappears when the client stands.

B) Auscultate the mass for bowel sounds.

16. Assessment of a client's nails reveals brownish-black discoloration and crumbling of the nail plate. The nurse should suspect which of the following etiologies? A) Fungal infection B) Bacterial infection C) Yeast infection D) Circulatory disorder

B) Bacterial infection

17. When inspecting a client's inguinal area for bulging, which of the following would be most appropriate for the nurse to have the client do? A) Bend forward from the waist B) Bear down as if having a bowel movement C) Hold his breath after exhaling D) Lie supine and draw his knees to his chest

B) Bear down as if having a bowel movement

20. The nurse is preparing to assess the abdomen of a client who is complaining of abdominal pain. Which statement by the nurse would be most appropriate? A) I'm going to examine the area where you're having pain first to get a better picture of what's going on. B) Before I get ready to examine the painful area, I will let you know in plenty of time. C) You don't need to worry about anything. I will make sure to be very gentle during the exam. D) Since you're having pain in a certain area, I won't have to do a very detailed exam there.

B) Before I get ready to examine the painful area, I will let you know in plenty of time.

1. The nurse is assessing a client's psychosocial development in light of Freud's theory. The nurse would interpret the client's status as the outcome of conflict between what variables? A) Cultural norms and personality traits B) Biological desires and social expectations C) Sexual desires and relational desires D) Sociocultural norms and health needs

B) Biological desires and social expectations

7. A nurse practitioner is performing a comprehensive physical examination of a 51-year- old man. After performing a digital-rectal exam for prostate enlargement and tenderness, the nurse checks the fecal material on the gloved finger for the presence of which of the following? A) Parasites B) Blood C) Bacteria D) Fungus

B) Blood

2. When assessing adolescent girls, the nurse should know that which of the following usually appears first? A) Pubic hair B) Breast buds C) Axillary hair D) Menses onset

B) Breast buds

23. During the health interview, the nurse asks a middle-aged client at what age she began menstruating. This question addresses a risk factor for what health problem? A) Mastitis B) Breast cancer C) Benign breast disease D) Paget's disease

B) Breast cancer

18. A group of students is reviewing information about general assessment indicators of nutritional status. The students demonstrate a need for additional review when they identify which of the following as an indicator of adequate nutritional status? A) Flat, firm abdomen B) Brittle hair C) Pink mucous membranes D) Elastic skin

B) Brittle hair

9. A client's recent episode of becoming lost near his home has prompted the nurse to use the Saint Louis University Mental Status (SLUMS) Assessment Tool. The nurse should begin this assessment by asking what question? A) How would you respond if someone said that you might have dementia? B) Can I ask you some questions about your memory? C) Do you generally consider yourself to be an intelligent person? D) I want to ask you some questions to see if you have Alzheimer's.

B) Can I ask you some questions about your memory?

19. An elderly client with a history of sinusitis has been taking antibiotics for this condition. The nurse should assess for what potential adverse effect of treatment? A) Exacerbation of cardiac dysrhythmias B) Candidal infection C) Overdrying of nasal passages D) Exacerbation of hypertension

B) Candidal infection

28. A nurse is collecting subjective data during a client's eye and vision assessment. When asking the question, Do you wear sunglasses during exposure to the sun? the nurse is addressing a known risk factor for what health problem? A) Presbyopia B) Cataracts C) Nystagmus D) Glaucoma

B) Cataracts

17. During a nutritional assessment, the client asks the nurse for suggestions to improve her diet. The nurse identifies a nursing diagnosis of health-seeking behaviors related to desire to improve diet. Which of the following suggestions would be most appropriate? A) The majority of your diet should consist of whole grains. B) Choose low-fat versions of milk products such as yogurt. C) Drink at least 2 to 3 glasses of fruit juices a day. D) Eat fewer orange vegetables and more dark green vegetables daily.

B) Choose low-fat versions of milk products such as yogurt.

24. The nurse's assessment suggests that a 10-year-old has failed to achieve Erikson's central task of this stage of development. What nursing diagnosis should most likely be included in the child's plan of care? A) Risk for injury B) Chronic low self-esteem C) Fear D) Disturbed thought processes

B) Chronic low self-esteem

15. A nurse has assessed an elderly client and is preparing to analyze the assessment data. Which of the following would the nurse need in order to accurately perform data comparison? A) Client's major complaints B) Client's usual daily patterns C) Client's adherence to treatment D) Client's underlying pathology

B) Client's usual daily patterns

29. The nurse is inspecting the dominant hand of an older adult client and notes the presence of irregularly shaped brown lesions on the dorsal surface of the client's hand. What action should the nurse perform next? A) Obtain a tissue sample for pathology B) Compare the appearance of the client's other hand C) Palpate the lesions for tenderness and warmth D) Perform health promotion teaching about sun protection

B) Compare the appearance of the client's other hand

23. A nurse is caring for a 70-year-old client from a different culture whose breast cancer has metastasized. The nurse observes that the client tends to defer responsibility for decision making around treatment options to her eldest son. How should the nurse respond to this? A) Explain the disconnect between the client's practice and the principle of client autonomy. B) Confirm that the client wants her son to make decisions and follow those decisions accordingly. C) Attempt to dialogue with the client when her son is not present. D) Refer the family to social work in order to further explore alternative decision- making practices.

B) Confirm that the client wants her son to make decisions and follow those decisions accordingly.

9. Based on a colleague's feedback, a nurse learns that she is aware of cultural differences in a general way but does not know what the specific differences are or how to communicate with a person of a specific culture. This nurse exhibits which of the following? A) Unconscious incompetence B) Conscious incompetence C) Conscious competence D) Unconscious competence

B) Conscious incompetence

11. A new graduate nurse asks a clinical nurse educator, I know abuse is a problem, but why must we screen all women during routine health contacts for potential abuse? Which response by the more experienced nurse would be most appropriate? A) The Family Violence Prevention Fund mandates it. B) Consistent risk factors for women at risk have not been identified. C) Women who are being abused always try to hide the fact. D) The abuse is typically part of the presenting problem.

B) Consistent risk factors for women at risk have not been identified.

16. A home care nurse is assessing an older adult's functional status. The nurse should identify which of the following as an instrumental activity of daily living? A) Bathing B) Cooking C) Toileting D) Eating

B) Cooking

30. A 12-year-old boy has been brought to the emergency department after being hit in the head with a pitch during a baseball game. The emergency department nurse's comprehensive assessment includes examination of the boy's ears with an otoscope. What assessment finding would suggest trauma to the middle ear or inner ear? A) White spots on the tympanic membrane B) Dark red or bluish tympanic membrane C) Yellowish, bulging tympanic membrane D) Clear tympanic membrane

B) Dark red or bluish tympanic membrane

21. The nurse is conducting a focused musculoskeletal assessment of an older adult client. When analyzing assessment data, the nurse should be aware of what age-related physiological changes? Select all that apply A) Absence of knee flexion B) Decreased bone density C) Decreased joint flexibility D) Joint capsule calcification E) Reduced muscle strength

B) Decreased bone density C) Decreased joint flexibility D) Joint capsule calcification E) Reduced muscle strength

11. The nurse is preparing to assess the size of the client's aorta. The nurse should palpate at which location? A) Midline at the umbilicus B) Deep epigastrium to the left of midline C) Slightly above the suprapubic area D) Between the umbilicus and the symphysis pubis

B) Deep epigastrium to the left of midline

17. The nurse is admitting a client to surgical daycare and is assessing the client's vital signs. When obtaining the client's oral temperature, where should the nurse insert the thermometer? A) At the gum line between the check and tongue B) Deep in the posterior sublingual pocket C) On either side of the frenulum at gingival level D) Just past the teeth, below the tongue

B) Deep in the posterior sublingual pocket

11. The nurse is assessing a client with mitral insufficiency. Which characteristic of the first heart sound should the nurse expect to hear? A) Split B) Diminished C) Accentuated D) Varying

B) Diminished

16. A nurse is assessing a client for possible fluid overload. Which of the following assessment findings is most consistent with this diagnosis? A) Venous filling of 3 seconds B) Distended neck veins with head elevated at 45 degrees C) Moist, plump tongue D) Boggy eyeball

B) Distended neck veins with head elevated at 45 degrees

13. The nurse is applying the principles of Freud's theory of psychosocial development during the health assessment of a young adult client. What assessment question is most likely to elicit data that are meaningful within this theoretical framework? A) Do you have a sufficient number of friends? B) Do you have a satisfying sexual relationship? C) How do you feel about your cultural background? D) Do you consider yourself to be a good person?

B) Do you have a satisfying sexual relationship?

22. A client exhibits many of the most common signs and symptoms of peptic ulcer disease. What interview question addresses the most plausible cause of the client's health problem? A) Do you feel like you're able to adequately address the stress in your life? B) Do you take painkillers like aspirin on a regular basis? C) Do you tend to eat foods that are quite high in fat? D) Are you currently taking vitamin supplements?

B) Do you take painkillers like aspirin on a regular basis?

27. The nurse is performing the Allen test on a client who has a diagnosis of peripheral vascular disease. What action should the nurse take after a positive Allen test? A) Document the absence of dorsalis pedis or posterior tibial pulses. B) Document the lack of patency in the ulnar and/or radial arteries. C) Attempt to palpate the popliteal pulse with the client's leg in a dependent position. D) Corroborate the finding by assessing capillary refill in the client's great toes.

B) Document the lack of patency in the ulnar and/or radial arteries.

26. The nurse is assessing a 6-year-old child. While auscultating the child's apical heart rate, the nurse notes that the child's heart rate increases during inspiration. What is the nurse's most appropriate action? A) Arrange for a STAT electrocardiogram. B) Document this as an expected assessment finding. C) Facilitate a referral for medical assessment. D) Reposition the child and then reassess.

B) Document this as an expected assessment finding.

20. A group of students is reviewing information from class about the purposes of assessment documentation. The students demonstrate understanding of the material when they state which of the following? A) Documentation helps support reimbursement but gives little epidemiologic data. B) Documentation provides a permanent legal record of care given and not given. C) Documentation is a viable means of communication but is repetitious. D) Documentation helps determine client education needs but not staff mix.

B) Documentation provides a permanent legal record of care given and not given.

2. A client has presented to the emergency department (ED) with complaints of abdominal pain. Which member of the care team would most likely be responsible for collecting the subjective data on the client during the initial comprehensive assessment? A) Gastroenterologist B) ED nurse C) Admissions clerk D) Diagnostic technician

B) ED nurse

20. A nurse is preparing a class for a local community group on coronary heart disease. Which of the following recommendations should the nurse include as appropriate for reducing a person's risk? Select all that apply. A) Avoid eating carbohydrates. B) Eat foods low in sodium. C) Walk for at least 30 minutes/day. D) Limit alcohol intake to 3 drinks per day. E) Use relaxation techniques to manage stress.

B) Eat foods low in sodium. C) Walk for at least 30 minutes/day. E) Use relaxation techniques to manage stress.

6. In response to a client's query, the nurse is explaining the differences between the physician's medical exam and the comprehensive health assessment performed by the nurse. The nurse should describe the fact that the nursing assessment focuses on which aspect of the client's situation? A) Current physiologic status B) Effect of health on functional status C) Past medical history D) Motivation for adherence to treatment

B) Effect of health on functional status

13. After assessing a family, the nurse determines that the family has permeable boundaries. The nurse interprets this as leading to what outcome? A) Restricting self-differentiation B) Encouraging emotional development C) Discouraging family cohesiveness D) Inhibiting emotional communication

B) Encouraging emotional development

10. When examining a child who complains of a sore throat, the nurse notes swelling on either side of the child's oropharynx. The nurse would include which of the following when documenting this finding? A) Enlarged pharyngeal tonsils B) Enlarged palatine tonsils C) Enlarged adenoids D) Enlarged lingual tonsils

B) Enlarged palatine tonsils

25. A nurse is aware of the need to protect against false allegations of inappropriate physical touch during a client's genitourinary assessment. How can the nurse best address this risk? A) Thoroughly explain the rationale for each aspect of the assessment. B) Ensure that a chaperone is present in the room during the exam. C) Perform the assessment as quickly and efficiently as possible. D) Ask for the client's permission prior to starting the assessment.

B) Ensure that a chaperone is present in the room during the exam.

14. When preparing to test a client for meningeal irritation, which of the following would be most important for the nurse to do first? A) Check for evidence of fever and chills. B) Ensure there is no injury to the cervical spine. C) Position the client prone. D) Check for a Babinski reflex.

B) Ensure there is no injury to the cervical spine.

7. The nurse is working with an older adult client and is attempting to determine whether the client deems her life to have been meaningful and valuable. As well, the nurse has addressed the client's acceptance of the inevitability of death. This nurse's actions are best understood within the ideas of which theorist? A) Freud B) Erikson C) Piaget D) Kohlberg

B) Erikson

18. An instructor is describing the stages of family growth and development using a two- parent nuclear family as an example. Which tasks would the instructor include as the likely priority for the childbearing family? A) Establishing a mutually satisfying marriage B) Expanding relationships with extended family C) Relating harmoniously to the kin network D) Socializing the children

B) Expanding relationships with extended family

20. A client has just been admitted to the postsurgical unit from postanesthetic recovery, and the nurse is in the introductory phase of the client interview. Which of the following activities should the nurse perform first? A) Collaborate with the client to identify problems. B) Explain the purpose of the interview. C) Determine the client's vital signs. D) Obtain family health history data.

B) Explain the purpose of the interview.

29. The nurse is preparing a client for an assessment of her genitalia and rectum. What action should the nurse perform when preparing the client? A) Assist the client into a prone position. B) Explain the rationale for using foot stirrups. C) Reassure the client that no one other than the nurse will be in the room. D) Obtain written, informed consent for the examination.

B) Explain the rationale for using foot stirrups.

16. A group of students is reviewing material related to the role of religion and spirituality in health care choices. The students demonstrate understanding when they identify which of the following situations as the most prominent ethical dilemma that involves religion? A) Providing life-saving therapy B) Failure to seek timely medical care C) Implementing spiritual care D) Treating clients' psychological needs

B) Failure to seek timely medical care

7. An African-American woman collapses at the funeral of her mother and later states that she could hear everything people were saying to her but, for a brief period, she could not move. The nurse interprets this as which of the following? A) Spell B) Falling out C) Empacho D) Susto

B) Falling out

20. When preparing a discussion about violence, which of the following would the nurse include? A) Abuse is primarily limited to lower socioeconomic groups. B) Family violence is a public health problem. C) Males experiencing intimate partner abuse have more options for help. D) Several states now require the reporting of child abuse.

B) Family violence is a public health problem.

29. A school nurse is working with kindergarten students. Within Kohlberg's framework of moral development, the nurse should recognize that these students' moral reasoning is primarily motivated by which of the following? A) An innate conscience B) Fear of the negative consequences of individual actions C) Motivation to exhibit behaviors that are culturally normalized D) Adherence to basic moral beliefs

B) Fear of the negative consequences of individual actions

20. A nurse is appraising a colleague's assessment technique as part of a continuing education initiative. The nurse demonstrates the proper technique for light palpation by performing which of the following actions? A) Depressing the skin 1 to 2 centimeters with the dominant hand B) Feeling the surface structures using a circular motion C) Placing the nondominant hand on top of the dominant hand D) Using one hand to apply pressure and the other hand to feel the structure

B) Feeling the surface structures using a circular motion

15. While assessing the knee joint of a client, a nurse also explains about the typical motions associated with that joint. Which of the following would the nurse include? A) Circumduction B) Flexion C) Abduction D) Internal rotation

B) Flexion

11. A woman comes to the clinic for an exam and says that she is considering trying to become pregnant in the next few months. Which of the following would the nurse encourage the client to begin taking now? A) Iron B) Folic acid C) Calcium D) Magnesium

B) Folic acid

20. Which of the following, if obtained during the health history, would alert the nurse to a possible risk factor for ear-related problems? A) Frequent use of acetaminophen (Tylenol) B) Frequent use of cotton-tipped applicators inside the ear C) Preference for showers rather than baths D) In adequate hygiene practices

B) Frequent use of cotton-tipped applicators inside the ear

4. The nurse is using a Wood's light for a client who has complaints of itching, burning, and peeling of the skin between his toes. The nurse is assessing for what etiology of the client's symptoms? A) Parasitic infection B) Fungal infection C) Bacterial infection D) Allergic reaction

B) Fungal infection

9. A nurse is planning to assess a male client for urethral discharge. Which technique would be best for the nurse to use? A) Have the client hold the penis while the examiner looks for discharge. B) Gently squeeze the glans between the thumb and index finger. C) Inspect the scrotal skin while holding the penis aside. D) Observe the glans of the penis for signs of abnormal discharge.

B) Gently squeeze the glans between the thumb and index finger.

21. A 49-year-old woman has sought care because of severe perimenopausal symptoms. The client has asked the nurse if she should talk to her doctor about beginning hormone replacement therapy (HRT). How should the nurse best respond? A) The most recent research suggests that the benefits of HRT have been greatly overstated. B) HRT often relieves many of the symptoms of menopause, but it's not without some risks. C) HRT is a good option for many women, mostly because it's a naturally occurring substance. D) Your doctor will likely recommend HRT because you're beginning menopause quite young.

B) HRT often relieves many of the symptoms of menopause, but it's not without some risks.

22. The nurse is assessing an 81-year-old client's peripheral vascular function. What principle should guide the nurse's analysis of assessment data? A) Leg pain that is relieved by rest is the result of normal physiological changes. B) Hair loss on the legs may be an age-related change rather than a sign of arterial insufficiency. C) Venous ulcers and arterial ulcers have a similar appearance and course in older adults. D) Non-palpable peripheral pulses are expected in clients over the age of 80.

B) Hair loss on the legs may be an age-related change rather than a sign of arterial insufficiency.

17. When appraising a young adult's psychosocial development within the framework of Erikson's theory, what question should guide the nurse's data collection and analysis? A) Can the client successfully solve problems? B) Has the client successfully achieved intimacy? C) Has the client learned to trust others? D) Can the client teach life skills to others?

B) Has the client successfully achieved intimacy?

21. A nurse is completing a comprehensive health history of a 69-year-old woman who is a new client of the clinic. Which of the nurse's interview questions most directly addresses the client's risk for developing cataracts? A) Do you exercise regularly? B) Have you ever been tested for diabetes? C) Do you ever take over-the-counter pain medications? D) At what age did you first start wearing glasses?

B) Have you ever been tested for diabetes?

6. When performing a client's head-to-toe assessment, during which part would the nurse assess the motor function of cranial nerve VII? A) Mental status examination B) Head and face assessment C) Ears assessment D) Examination of mouth and throat

B) Head and face assessment

21. A client has sought care with complaints of increasing swelling in her feet and ankles, and the nurse's assessment confirms the presence of bilateral edema. The nurse's subsequent assessments should focus on the signs and symptoms of what health problem? A) Myocardial infarction B) Heart failure C) Atherosclerosis D) Heart block

B) Heart failure

13. The nurse is completing the general survey of a client and determines that the client's temperature is 102∞F. Which of the following would the nurse also expect to find? A) Weak, thready pulse B) Heart rate greater than 100 bpm C) Respiratory rate between 12 and 20 breaths/minute D) Diastolic blood pressure 10 mm Hg greater than normal

B) Heart rate greater than 100 bpm

27. During a client's spiritual assessment, the client explains that the ultimate purpose of her existence is to achieve a state that she describes as nirvana. The nurse should recognize that this client ascribes to what religion? A) Islam B) Hinduism C) Buddhism D) Judaism

B) Hinduism

23. During a health screening event, the nurse is assessing a client's risk factors for lung cancer. When addressing the most significant risk factor for lung cancer, the nurse should question the client about which of the following? A) Childhood exposure to air pollution B) History of tobacco use C) History of working in a factory or smelter D) History of recurrent lung infections

B) History of tobacco use

22. A nurse is completing a detailed assessment of a family who is receiving care. When assessing the context of the family, the nurse should include what assessment questions? Select all that apply. A) Are there any ways that you think your family could be happier? B) How would you describe your family's ethnicity? C) Would you characterize your family as being religious? D) When were each of you born? E) How do you interact within the neighborhood around you?

B) How would you describe your family's ethnicity? C) Would you characterize your family as being religious? E) How do you interact within the neighborhood around you?

10. A nurse is preparing a health education session for a local community group. When addressing the relationship between coronary artery disease (CAD) and culture, which information would the nurse include? A) Caucasians usually possess greater lifestyle risks for CAD than African Americans. B) Hypertension is more prevalent in African Americans than among Caucasians. C) Hypertension is seen more in white women than in African-American women. D) Hispanic Americans have a higher rate of CAD than white Americans.

B) Hypertension is more prevalent in African Americans than among Caucasians.

18. A client's electronic health record reveals that he had surgery as an infant to correct the fact that his urethra was located on the ventral side of his penis. The nurse should recognize that this client had which of the following? A) Epispadias B) Hypospadias C) Paraphimosis D) Phimosis

B) Hypospadias

7. When documenting a comprehensive assessment, which statement would the nurse record as the reason for seeking health care? A) I try not to let the pain affect my life. B) I haven't had a checkup in over 5 years. C) I had my appendix removed when I was 14 years old. D) I have an aunt who had breast cancer.

B) I haven't had a checkup in over 5 years.

21. A nurse is providing a verbal update to a client's primary care provider because of the client's worsening nausea. When using an SBAR format to provide a report, the nurse should complete the report with which of the following statements? A) What would you like to do to address this client's nausea? B) I think this client would benefit from an antiemetic. C) This client has no recent history of any nausea or vomiting. D) This client rates his nausea as seven out of ten.

B) I think this client would benefit from an antiemetic.

26. A nurse is working with a client who has a history of headaches. When preparing to assess the client's temporomandibular joint (TMJ), the nurse should provide what instruction? A) I'm going to press on several different places below and in front of your ear. B) I'm going to put my fingers in front of your ears and ask you to open your mouth wide. C) Turn so I can see the side of your face and then open your mouth wide like you're yawning. D) When I place my hands on your cheeks, clench your teeth and then relax them.

B) I'm going to put my fingers in front of your ears and ask you to open your mouth wide.

13. A nurse is preparing to conduct a community assessment. Upon completing the assessment, which of the following should the nurse expect as the primary outcome? A) Development of a common bond with community members B) Identification of health-related concerns in the community C) Creation of a partnership with local health care providers D) Increase in the number of health services provided

B) Identification of health-related concerns in the community

12. The nurse is assessing a client who has a complex cardiac history. The nurse has asked the client to lean forward while in a sitting position. This position will allow the nurse to do which of the following? A) Assess the client's heart sounds while preventing shortness of breath. B) Identify heart sounds that may be inaudible in other positions. C) Assess the impact of the client's heart disease on his mobility. D) Differentiate heart sounds from breath sounds.

B) Identify heart sounds that may be inaudible in other positions.

1. A nurse is preparing to assess a family. The nurse best adopts the view of the family unit as being a system by using which approach? A) Focusing on each individual member's health problems B) Identifying strengths and problem areas within the family structure C) Educating the sick client about methods to maintain independence D) Fostering emotional support for each family member

B) Identifying strengths and problem areas within the family structure

29. A nurse is relying heavily on gestures and simplified language during the assessment of a client from another culture who speaks minimal English. During the lengthy assessment, the nurse asks the client if she is okay by making a circle with his thumb and forefinger. The nurse should be aware of which of the following? A) In some cultures, this gesture denotes confusion. B) In some cultures, this gesture is offensive. C) This gesture has meaning only in American cultures. D) In some cultures, this gesture denotes pain.

B) In some cultures, this gesture is offensive.

16. A nurse is performing an otoscopic exam of a client's right tympanic membrane. At which location would the nurse document seeing the cone of light if it were in the appropriate place? A) In the center of the membrane B) In the 5 o'clock position C) In the 7 o'clock position D) In the upper left quadrant

B) In the 5 o'clock position

9. A client has presented with signs and symptoms that are suggestive of Bell's palsy. What assessment finding is most consistent with this diagnosis? A) Inability to detect sharp and dull stimuli B) Inability to wrinkle the forehead C) Closure of the affected eye from swelling D) Muscle spasm of the lower face on the affected side

B) Inability to wrinkle the forehead

19. A nurse is teaching an older adult client about breast self-examination. The nurse includes teaching on expected changes in the client's breasts due to aging. Which of the following would the nurse include? A) Increase in glandular tissue B) Increase in fatty tissue C) Larger nipple area D) Less "granular" in texture

B) Increase in fatty tissue

14. A client's history reveals that he has been taking oral steroid therapy for several years for the treatment of an autoimmune disorder. During assessment, the nurse would expect the client's skin to have what characteristic? A) Increased thickness and hair loss B) Increased thinness C) Pallor D) Erythema

B) Increased thinness

27. A nurse has learned that the community's real estate prices have risen sharply over the past several years. The nurse should anticipate what effect from this phenomenon? A) Increases in substance abuse B) Increases in homelessness C) Decreased access to primary care D) Decreased numbers of older adults

B) Increases in homelessness

19. Assessment of an older adult client suggests that the client does not possess formal operational thinking. Within Piaget's framework of development, what nursing diagnosis is the most likely consequence of this developmental deficit? A) Spiritual distress B) Ineffective health maintenance C) Ineffective sexuality pattern D) Risk for suicide

B) Ineffective health maintenance

12. During the first prenatal examination, a woman is found to have inverted nipples. Which suggestion would be most appropriate if the woman desires to breast-feed her infant? A) Wear a supportive nursing bra during the pregnancy. B) Insert breast shields in the bra during the third trimester. C) Apply a lanolin cream to the nipples twice daily. D) Rub the nipples frequently with a rough towel.

B) Insert breast shields in the bra during the third trimester.

14. The nurse is performing an ear assessment of an adult client. Which of the following actions constitutes the correct procedure for using an otoscope when examining the client's ears? A) Keeping the dominant hand away from the client's head B) Inserting the speculum down and forward into the ear canal C) Using the smallest speculum on the otoscope head D) Holding the otoscope in the nondominant hand

B) Inserting the speculum down and forward into the ear canal

6. A nurse is creating a family attachment diagram. The nurse is assessing which area of family structure and function? A) Composition B) Interaction patterns C) Power structure D) Socialization

B) Interaction patterns

24. The nurse is performing a health assessment on a community-dwelling client who is recovering from hip replacement surgery. Which of the following actions should the nurse prioritize during assessment? A) Focus the assessment on the client as a member of her age group. B) Interpret the information about the client in context. C) Corroborate the client's statements with trusted sources. D) Gather information from a variety of sources.

B) Interpret the information about the client in context.

6. During an integumentary assessment, the nurse notes that the client's fingernails are very thin and concave. The nurse knows the client needs medical follow-up for further assessment to rule out which condition? A) Diabetes mellitus B) Iron deficiency anemia C) Vitamin A deficiency D) Peripheral vascular disease

B) Iron deficiency anemia

20. A nurse is presenting a class to a local community group about vision and eye health. As part of the presentation, the nurse explains how visual perception occurs. Which of the following would the nurse include in the explanation? A) It refers to a client's subjective appraisal of his or her vision. B) It begins with light rays striking the retina. C) It primarily involves the lens of the eye. D) It allows the eyes to focus on near objects.

B) It begins with light rays striking the retina.

2. A client asks why cerumen is important, stating, It just clogs up the ears anyway. How should the nurse best describe the purpose of cerumen? A) It helps protect the delicate ear drum from loud noise that may be damaging. B) It helps to keep the ear drum soft and functioning well. C) It helps to amplify the sound waves through the inner ear. D) It helps create the translucent, pearly color of the ear drum.

B) It helps to keep the ear drum soft and functioning well.

3. The nurse has completed an initial assessment of a newly admitted client and is applying the nursing process to plan the client's care. What principle should the nurse apply when using the nursing process? A) Each step is independent of the others. B) It is ongoing and continuous. C) It is used primarily in acute care settings. D) It involves independent nursing actions.

B) It is ongoing and continuous.

3. A nurse educator is leading a group of nurses in exercises aimed at improving cultural competence. Which of the following would the educator use to best describe an aspect of the term culture? A) Transmission occurs to another generation through genetics. B) It is shared through norms for behaviors, values, and beliefs. C) It is adapted to a specific environment. D) It is experienced by all people even without human contact.

B) It is shared through norms for behaviors, values, and beliefs.

26. A client tearfully admits to the nurse that her husband beats her when he drinks alcohol excessively. How should the nurse best respond to the client's statement? A) Thank you for implicating your husband in this illegal behavior. B) It took a great deal of courage for you to tell me that. C) I will take action to make sure that this never happens again. D) Now that this is out in the open, you can begin to rebuild your life.

B) It took a great deal of courage for you to tell me that.

30. An obese teenage boy from a culture that values increased body mass has been referred to the clinic. The nurse is assessing him for malnutrition based on his electronic health record and current health complaints. His mother questions the nurse's rationale, stating, Anyone can see he's not malnourished. Just look at the size of him! How should the nurse best respond? A) People sometimes become obese because their bodies are storing up nutrients that they often lack. B) It's actually very possible for a person to be overweight but have inadequate nutrition. C) Assessment for malnutrition is a standard component of a larger nutritional assessment, which is very important for your son's health. D) Actually, there's very little relationship between body mass and nutritional state.

B) It's actually very possible for a person to be overweight but have inadequate nutrition.

19. A nurse educator is reviewing the unit's resources about religious groups and their views about blood and blood products, organ donation, and autopsy. A member of which group is most likely to refuse a blood transfusion? A) Christian Scientists B) Jehovah's Witnesses C) Orthodox Jews D) Roman Catholics

B) Jehovah's Witnesses

8. The nurse is examining an older adult client and using a goniometer. Which of the following would the nurse be assessing? A) Extremity edema B) Joint flexion/extension C) Two-point discrimination D) Vibratory sensation

B) Joint flexion/extension

20. A nurse asks a client the following question: What do you do if you have pain? The nurse is assessing which of the following aspects of cognitive function? A) Orientation B) Judgment C) Abstract reasoning D) Memory

B) Judgment

1. The nurse is reviewing a client's electronic health record before assessing her mouth. Which of the following diagnoses would the nurse recognize as an indication for immediate medical follow-up? A) Thrush B) Leukoplakia C) Gingivitis D) Canker sore

B) Leukoplakia

28. The nurse is conducting an assessment of a rural community. Which of the following community assessment findings is most characteristic of rural communities? A) Apathy regarding health and health care B) Limited access to specialized health care C) Increased proportions of children and young adults D) Reduced levels of safety and security

B) Limited access to specialized health care

16. A nurse is using a nursing minimum data set to document findings following the assessment of a client. This nurse is most likely providing care in which setting? A) Acute care facility B) Long-term care facility C) Urgent care center D) Health clinic

B) Long-term care facility

30. A nurse is conscientious in adhering to the requirements of the Health Insurance Portability and Accountability Act (HIPAA) when providing care for clients. What action best meets these legal requirements for care? A) Having a colleague audit the nurse's documentation to ensure objectivity B) Maintaining the privacy and confidentiality of clients' medical records C) Using electronic records whenever possible, rather than paper-based records D) Collaborating with the client and his or her family prior to documenting

B) Maintaining the privacy and confidentiality of clients' medical records

28. A nurse is preparing a female client for a genitourinary examination that has been scheduled for later in the week. What anticipatory guidance should the nurse provide to the client? A) Stop taking any antibiotics for 24 hours before your examination. B) Make sure not to douche for 48 hours before the examination. C) Don't bathe or shower on the morning of the appointment. D) Drink at least 48 ounces of fluid the morning before the appointment.

B) Make sure not to douche for 48 hours before the examination.

22. A nurse is conducting a focused head and neck assessment of a client. When preparing to assess the client's thyroid gland, the nurse should be aware of which of the following principles? A) The thyroid gland is not normally palpable in female clients. B) Many clients have an additional (third) thyroid lobe. C) The thyroid gland is not normally palpable until clients are in their thirties or forties. D) Palpation creates a risk of rupturing the thyroid gland in some older adult clients.

B) Many clients have an additional (third) thyroid lobe.

2. A nurse is planning care that is grounded in the fact that clients are holistic beings. Which of the following lists of components constitute the view of clients as holistic beings? A) Physical identity, psychosocial identity, religious identity B) Mind, body, spirit C) Id, ego, superego D) Spiritual identity, egocentric nature, naïve identity

B) Mind, body, spirit

16. A nurse in the emergency department assesses a client's pupillary reaction and observes pinpoint pupils. The nurse interprets this finding as suggesting which of the following? A) Recent eye trauma B) Narcotic use C) Macular degeneration D) Recent peripheral nervous system injury

B) Narcotic use

15. The nurse inspects a client's mouth and notes the presence of a bifid uvula. The nurse understands that this finding is most common in which ethnic group? A) Italian Americans B) Native Americans C) African Americans D) Non-Hispanic Americans

B) Native Americans

7. A nursing student has been assigned to the care of a client whose history suggests the need for a mental status assessment. This client most likely has a history of health problems affecting what body system? A) Respiratory B) Neurologic C) Cardiovascular D) Renal

B) Neurologic

8. The nurse assesses chest expansion in a 30-year-old man and finds it to be 8 cm. The nurse should document this as which of the following? A) Limited expansion B) Normal expansion C) Hypoexpansion D) Hyperexpansion

B) Normal expansion

20. When attempting to assess a client's pain, which of the following actions should the nurse perform first? A) Observe behaviors in the client. B) Obtain a client self-report. C) Search for possible causes of pain. D) Ask family members about the client's pain.

B) Obtain a client self-report.

26. Palpation of a male client's urethra produces a yellowish-white discharge. What is the nurse's best action? A) Obtain a urine sample for culture and sensitivity testing. B) Obtain a sample of the discharge for culture. C) Ask the client to void and then repeat palpation of the client's urethra. D) Palpate the client's scrotum and testes for the presence of fluid.

B) Obtain a sample of the discharge for culture.

11. The nurse notes a malodorous, yellow discharge upon inserting the speculum into the client's vagina. Which of the following should the nurse do next? A) Obtain a urine specimen. B) Obtain a wet mount slide. C) Procure a Papanicolaou (Pap) smear. D) Perform a bimanual exam.

B) Obtain a wet mount slide.

22. An older adult client has been admitted to the intensive care unit after experiencing a serious decline in health due to influenza. The client's family is surprised that influenza could have such serious health consequences. When educating the family about this phenomenon, what should the nurse describe? A) Older adults' immune systems cannot produce new antibodies. B) Older adults have a diminished physiologic reserve. C) Older adults lack resistance to many common viruses. D) Older adults cannot tolerate antibiotics used to treat influenza.

B) Older adults have a diminished physiologic reserve.

1. The nurse is preparing to palpate a client's temporal artery. The nurse would place the hands at which location? A) On each side of the client's face, anterior and inferior to the ears B) On each side between the top of the ear and the eye C) Bilaterally, parallel to and anterior to the sternomastoid muscle D) Inferior to the lower jaw beneath the client's tongue

B) On each side between the top of the ear and the eye

18. The nurse is preparing to perform the Rinne test on a client. The nurse should place the tuning fork at which location first? A) Center of the client's forehead B) On the client's mastoid process C) In front of the client's external auditory canal D) At the base of the client's skull

B) On the client's mastoid process

2. When performing a client's ophthalmoscopic exam, the nurse observes a round shape with distinct margins. The nurse would document this as which of the following? A) Physiologic cup B) Optic disc C) Retinal vessels D) Fovea

B) Optic disc

8. A nurse palpates a client's ear and finds that the tragus is exquisitely tender. The nurse should suspect which of the following health problems? A) Otitis media B) Otitis externa C) Ruptured tympanic membrane D) Mastoiditis

B) Otitis externa

24. A client has sought care because of chronic constipation. During the health history interview, the nurse should address what potential contributing factor? A) Excessive fat and sugar intake B) Overuse of laxatives C) Obesity D) Inadequate abdominal muscle tone

B) Overuse of laxatives

21. A client has a family history of prostate cancer and is committed to regular screening. What should the nurse teach the client about prostate-specific antigen (PSA) blood testing? A) Annual PSA blood testing should begin at age 50. B) PSA blood testing is not recommended for most clients. C) PSA blood testing should only be performed on men who reject digital rectal exams. D) PSA blood tests should be performed biannually between ages 45 and 60 and then annually thereafter.

B) PSA blood testing is not recommended for most clients.

19. A nurse is having difficulty identifying a client's heart sounds, specifically S1 and S2. Which of the following would be most appropriate for the nurse to do? A) Use the bell of the stethoscope to help distinguish the sounds. B) Palpate the carotid pulse while auscultating the heart. C) Determine the pulse deficit. D) Palpate the apical impulse.

B) Palpate the carotid pulse while auscultating the heart.

15. When the nurse palpates the abdomen of a preschool boy, he begins to giggle and draw his legs up onto his abdomen. Which of the following would be most appropriate for the nurse to do? A) Omit the entire abdominal exam. B) Palpate with the child's hand under the nurse's hand. C) Ask the parent to discipline the child. D) Explain the purpose of the exam to the child.

B) Palpate with the child's hand under the nurse's hand.

26. A client has presented with a terrible head cold, and the nurse is assessing for signs and symptoms of sinusitis. The nurse should utilize what assessment techniques? Select all that apply. A) Inspection B) Palpation C) Auscultation D) Percussion E) Transillumination

B) Palpation D) Percussion E) Transillumination

9. A 15-year-old boy shows the school nurse a bump on his neck. The nurse observes a raised, erythematous, solid, 0.3-cm by 0.2-cm mass. The nurse would document the presence of which of the following? A) Macule B) Papule C) Nodule D) Pustule

B) Papule

30. Examination of a client's gait reveals that the client is stooped over when walking and that he slowly shuffles. As well, the client maintains a stiff posture when walking. The nurse should document what type of gait? A) Scissors gait B) Parkinsonian gait C) Spastic hemiparesis D) Footdrop

B) Parkinsonian gait

17. A woman and her partner come to the emergency department. The woman has bruising on both upper extremities and a fracture of the left arm. The client states that she fell down the stairs. Which of the following would lead the nurse to suspect that the client is a victim of violence? A) Client freely answers questions asked about the injury B) Partner states that client is very clumsy and accident prone C) Partner offers to leave the examination room D) Client holds partner's hand when arm is being examined

B) Partner states that client is very clumsy and accident prone

20. After inspecting an adolescent male's genitalia, the nurse documents the findings as Tanner stage 3. Which of the following findings would be most likely? A) Scrotum and testes slightly enlarged; sparse, long, downy pubic hair B) Penis elongated; pubic hair sparse over pubis, coarse and curly C) Penis increased in width; abundant pubic hair not extending to thighs D) Penis of adult size; dark curly abundant pubic hair to thighs

B) Penis elongated; pubic hair sparse over pubis, coarse and curly

20. A nurse in the surgical daycare department has called a client in from the waiting room and is meeting the client for the first time. The nurse immediately observes that the client has a noticeably stooped posture. How should the nurse best follow up this abnormal assessment finding? A) Facilitate a referral to the hospital's rheumatology department B) Perform a focused assessment of the client's musculoskeletal system C) Obtained a detailed family health history from the client D) Document the assessment finding and inform the anesthesiologist

B) Perform a focused assessment of the client's musculoskeletal system

3. A 45-year-old African-American client comes to the clinic complaining of fatigue, thirst, and frequent urination. During the exam, the nurse notices areas of hyperpigmentation around the neck and in the axillae. Which of the following should the nurse do next? A) Document the benign findings. B) Perform a random blood sugar test. C) Ask the client about a family history of cancer. D) Refer the client for medical follow-up.

B) Perform a random blood sugar test.

30. The nurse should ensure that a Doppler ultrasound is available when performing which of the following assessments? A) Respiratory assessment B) Peripheral vascular assessment C) Abdominal assessment D) Musculoskeletal assessment

B) Peripheral vascular assessment

10. While inspecting the skin of an older adult client, the nurse notes multiple small, flat, reddish-purple macules. The nurse should recognize the presence of which of the following? A) Purpura B) Petechiae C) Ecchymosis D) Cherry angioma

B) Petechiae

28. A nurse is conducting a focused ear and hearing assessment of an adult client who has a history of mild hearing loss. When performing the whisper test, what instruction should the nurse begin with? A) Please close your eyes and listen carefully to what I say. B) Please cover your ear that has the weakest hearing. C) Please tell me when you can hear me talking. D) Please repeat the words that I say.

B) Please cover your ear that has the weakest hearing.

25. A nurse at an ambulatory clinic is preparing to begin the collection of objective assessment data from a female client. After meeting the client and bringing her into the examination room, what instruction should the nurse provide? A) I'll get you to lay down flat on the exam table, please. B) Please have a seat on the edge of the exam table. C) I'll start the assessment with you standing up and then help you onto the table. D) Where would you like me to conduct your health assessment?

B) Please have a seat on the edge of the exam table.

14. A nurse is modifying an Asian client's diet to accommodate the concept of hot and cold. The nurse demonstrates an understanding of this concept when identifying which of the following as a cold condition? A) Diabetes B) Pneumonia C) Sore throat D) Hypertension

B) Pneumonia

27. The nurse is using an ophthalmoscope to examine a client's inner eye structures. What action should the nurse perform in order to accurately examine the client's optic disc? A) Slowly approach the client's eye from a 90-degree angle, maintaining a focus on the pupil. B) Position the scope close to the client's eye and look through the pupil at a 15- degree angle. C) From a distance of 3 to 5 cm, examine the pupil from a 45- to 50-degree angle. D) While looking through the ophthalmoscope, approach the client's eye slowly from the side.

B) Position the scope close to the client's eye and look through the pupil at a 15- degree angle.

9. A nurse assesses a client's blood pressure and the findings suggest orthostatic hypotension. Which area should the nurse emphasize during client education? A) Daily exercise routine B) Prevention of falls C) Diet high in iron D) Vitamin supplementation

B) Prevention of falls

20. A client comes to the prenatal clinic for a follow-up examination. When assessing the client's breasts, which of the following should the nurse expect to find? Select all that apply. A) Pallor of the areolae B) Prominent veins C) Nodular breasts D) Warmth E) Increased sensitivity

B) Prominent veins C) Nodular breasts E) Increased sensitivity

6. A client has presented to the emergency department and is having difficulty describing her vague sensation of physical discomfort and unease. How can the nurse best elicit meaningful assessment data about the nature of the client's complaint? A) Ignore the complaint for now and return to it later in the assessment. B) Provide a laundry list of descriptive words. C) Restate the question using simpler terms. D) Wait in silence until the client can determine the correct words.

B) Provide a laundry list of descriptive words.

9. A nurse has finished examining a client's nose and sinuses and is about to examine the client's mouth and throat. Which of the following would be most important for the nurse to do? A) Warm the hands B) Put on gloves C) Obtain a tuning fork D) Collect a saliva specimen

B) Put on gloves

2. After teaching a group of students about the bones and their functions, the instructor determines that the teaching was successful when the students state that blood cells are produced in which of the following? A) Compact bone B) Red marrow C) Yellow marrow D) Spongy bone

B) Red marrow

22. A 52-year-old woman's current medication regimen includes estrogen-progestin therapy (EPT). In addition to reduced symptoms of menopause, the nurse should be aware that this therapy confers what secondary benefit? A) Weight loss B) Reduced risk of colorectal cancer C) Protection against stroke D) Increased libido

B) Reduced risk of colorectal cancer

7. When asked to touch her ear to her shoulder, a client reports pain. Which of the following should the nurse do next? A) Perform muscle strength against resistance. B) Refer the client for further evaluation. C) Flex and then hyperextend the neck. D) Palpate the paravertebral muscles for pain.

B) Refer the client for further evaluation.

14. The nurse assesses thick, white plaques on a client's tongue and hard palate. Which of the following nursing actions should the nurse do next? A) Facilitate blood testing for human immunodeficiency virus (HIV). B) Refer the client to a primary care provider for medication. C) Asses the client's laboratory values for zinc deficiency. D) Assess the client for signs of jaundice.

B) Refer the client to a primary care provider for medication.

5. When examining the mouth of an adult client with recent cognitive changes, the nurse notes a distinct bluish-black line along the client's gum line. Which action should be the nurse's priority? A) Determining whether the client is receiving phenytoin therapy B) Referring the client for further evaluation C) Encouraging the client to enroll in a smoking cessation program D) Providing the client with information on proper mouth care

B) Referring the client for further evaluation

14. A nurse is reviewing an infant's Apgar score. Which of the following areas was assessed during the calculation of the score? A) Temperature B) Reflex irritability C) Head circumference D) Weight

B) Reflex irritability

21. A nurse is completing an admission assessment of an adult client, during which the client states, "I've never been a religious man, but I'm definitely spiritual." How should the nurse best understand an aspect of the relationship between spirituality and religion? A) Spirituality is an Eastern concept, whereas religion is associated with Western cultures. B) Religion consists of the spiritually focused rituals and practices of a group. C) Spirituality is the codification of principles that are based on religion. D) Religion is the state of spiritual certainty that results from cultural influences.

B) Religion consists of the spiritually focused rituals and practices of a group.

27. The nurse is percussing the area over the client's lungs and hears a loud, low-pitched, hollow sound. The nurse documents this finding as which of the following? A) Flatness B) Resonance C) Tympany D) Dullness

B) Resonance

18. When evaluating a community's education, which of the following would the nurse use to identify the effectiveness of the community's school system? A) Number of libraries B) Scores on standardized tests C) School health programs D) Extracurricular activities

B) Scores on standardized tests

19. The nurse is assessing the client's perception of pain and the client's description of its intensity and quality. Which dimension of pain is the nurse evaluating? A) Physical B) Sensory C) Behavioral D) Cognitive

B) Sensory

16. The nurse is inspecting the cervix of a client who has two children. The nurse would expect the cervical os to appear as which of the following? A) Round B) Slit-like C) Transverse D) Stellate

B) Slit-like

21. The nurse is preparing to examine an older adult client. Which of the following would be most appropriate for the nurse to do during the examination? A) Complete the examination as quickly as possible. B) Speak clearly and slowly when explaining a procedure. C) Begin the examination with auscultation instead of inspection. D) Maintain the supine position for each part of the examination.

B) Speak clearly and slowly when explaining a procedure.

13. When obtaining a cervical specimen for a Neisseria gonorrhoeae culture, which of the following would be most appropriate? A) Wipe the cotton-tipped applicator onto a slide. B) Spread the specimen in a Z pattern on a special culture plate. C) Immerse the swab in a liquid medium and refrigerate. D) Roll the endocervical brush onto a slide.

B) Spread the specimen in a Z pattern on a special culture plate.

8. A nurse has been asked to assess an older adult resident of a long-term care facility. During assessment of the resident's skin, the nurse notes a break in the skin, erythema, and a small amount of serosanguineous drainage over the resident's sacrum. Inspection reveals that the area appears blister-like. The nurse should interpret this finding as indicating which stage of pressure ulcer? A) Stage I B) Stage II C) Stage III D) Stage IV

B) Stage II

17. The nurse is assessing a client who is in liver failure and who has developed ascites. When measuring the client's abdominal girth, the nurse should place the client in which position? A) Sitting B) Standing C) Supine D) Prone

B) Standing

12. A nurse is palpating the position of the client's trachea. At which anatomic site would the nurse first position a finger for palpation? A) Sternocleidomastoid muscle B) Sternal notch C) Submental space D) Supraclavicular space

B) Sternal notch

17. The nurse can best palpate the superficial cervical nodes, the deep cervical chain, and the supraclavicular nodes by first locating which muscle? A) Infraspinous B) Sternomastoid C) Trapezius D) Platysma

B) Sternomastoid

4. An older adult client states, Sometimes when I sneeze, I notice that I wet my pants. The nurse interprets this as which of the following? A) Reflex incontinence B) Stress incontinence C) Urge incontinence D) Total incontinence

B) Stress incontinence

16. A nurse instructor is observing a nursing student assess a client's capillary refill. Which action by the student indicates the proper technique? A) Student gently compresses the wrist area on the side of the thumb. B) Student compresses the client's nail bed until it blanches. C) Student applies firm pressure to the hand, noting any indentation. D) Student asks client to turn hands slowly over and back.

B) Student compresses the client's nail bed until it blanches.

18. A nurse is assessing a 9-month-old infant. Which reflexes would the nurse expect to assess? Select all that apply. A) Rooting B) Sucking C) Tonic neck D) Moro E) Palmar grasp F) Babinski

B) Sucking F) Babinski

11. A family has just admitted their grandmother to a long-term care facility after caring for her in the home for several years. Which statement by the nurse would best demonstrate purposeful therapeutic conversation? A) If possible, it would be best to visit her every day. B) Tell me about her morning routine at home. C) How do you plan to finance her medications? D) How independent is she with dressing and bathing?

B) Tell me about her morning routine at home.

19. The nurse is assessing the genitalia of an older adult client. Which of the following would the nurse document as a normal finding? A) Decrease in size of the testes B) Testes hanging lower in the scrotum C) Abundant pubic hair D) Bulging in the inguinal area

B) Testes hanging lower in the scrotum

1. A nurse is preparing a presentation for a local community group on family violence and child abuse. Which of the following would be most appropriate for the nurse to research when defining child abuse in legal terms? A) The Prevent Child Abuse America group B) The Child Abuse Prevention and Treatment Act C) The Child Abuse Prevention Network D) The Child Well-being and Domestic Violence Project

B) The Child Abuse Prevention and Treatment Act

27. The nurse is assessing an older adult client's heart and neck vessels. When attempting to palpate the client's apical impulse, what principle should guide the nurse's actions? A) The apical impulse will be irregular due to normal, age-related physiological changes. B) The apical impulse may be more difficult to palpate than in a younger client. C) The apical impulse will be found in a more medial location than in a younger client. D) The apical impulse will be easier to palpate if the client is in a standing position.

B) The apical impulse may be more difficult to palpate than in a younger client.

24. The nurse is completing a client's genitourinary assessment and is preparing to assess the client's cervix. What finding would most clearly warrant referral? A) The cervix is firm on palpation. B) The cervix is immobile on palpation. C) The cervix is smooth and pink on inspection. D) The cervix projects 2 cm into the client's vagina.

B) The cervix is immobile on palpation.

22. The nurse has assessed the head circumference (HC) of an 18-month-old during a regular checkup. The nurse should compare the percentile of the child's HC to which of the following? A) The child's body mass index B) The child's height and weight percentiles C) The child's chest circumference percentile D) The child's developmental stage

B) The child's height and weight percentiles

14. The nurse is assessing a young adult client in light of Erikson's theory of psychosocial development. During this life stage, what assessment finding would most clearly suggest a lack of successful development? A) The client is dissatisfied with her current job. B) The client describes herself as lonely and isolated. C) The client has been diagnosed with bipolar disorder. D) The client had a child when she was in her late teens.

B) The client describes herself as lonely and isolated.

4. When taking a health history for a female client, which factor should the nurse identify as placing the client at increased risk for breast cancer? A) The client smokes six to eight cigarettes per day B) The client had her first child at age 38 C) The client breast-fed her child for a full year D) The client has a low body mass index

B) The client had her first child at age 38

29. A nurse is assessing an older adult client's risk for pressure ulcers using the Braden Scale for Predicting Pressure Sore Risk. Which aspect of the client's current health status would be reflected in her score on this scale? A) The client has a full-time caregiver. B) The client is consistently incontinent of urine. C) The client has a surgical diagnosis. D) The client adheres to a vegetarian diet.

B) The client is consistently incontinent of urine.

7. The nurse is assessing a client with a cardiac condition who complains of not sleeping well and of having to get up frequently at night to urinate. The nurse should recognize what implication of this statement? A) The client may have developed a cardiac conduction problem. B) The client may be experiencing symptoms of heart failure. C) The client's cardiac problem is being adequately compensated for. D) The client may be at increased risk for myocardial infarction.

B) The client may be experiencing symptoms of heart failure.

25. A nurse is attempting to apply the principles of cultural competency in the care of a 72- year-old Asian-American woman who has a spinal cord compression. Which of the following statements should guide the nurse's care? A) The client may view pain as a sign of weak character. B) The client may be reluctant to accept opioids. C) The client may tend to overreport her pain. D) The client may be unable to understand quantitative assessment scales.

B) The client may be reluctant to accept opioids.

24. A clinic nurse is conducting a comprehensive assessment of a 70-year-old male client of Native American ethnicity. The nurse observes that the client rarely makes eye contact and holds his head low during the assessment. How should the nurse best interpret this practice? A) The client may not understand the purpose of the assessment. B) The client may be showing the nurse respect. C) The client may be a victim of intimate partner violence. D) The client may not trust the nurse's expertise.

B) The client may be showing the nurse respect.

24. The nurse's assessment of an 81-year-old client's hearing has corroborated her recent history of hearing loss. The nurse questions the client about her use of hearing aids, to which the client responds, I've got enough frustration in my life without having to fiddle with those. The nurse should suspect which of the following? A) The client may misunderstand the factors underlying her hearing loss. B) The client may have had a negative experience with hearing aids in the past. C) The client may be unable to afford the cost of hearing aids. D) The client may be unwilling to adhere to treatment regimens.

B) The client may have had a negative experience with hearing aids in the past.

30. An older adult client has been admitted for assessment related to decreased cognition. What assessment finding is most suggestive of delirium as the cause of the client's cognitive changes? A) The client has a family history of cognitive disorders. B) The client recently began a new medication regimen. C) The client has been under significant psychosocial stress. D) The client's cognition has declined over several months.

B) The client recently began a new medication regimen.

21. A nurse is utilizing the Braden Scale for Predicting Pressure Sore Risk during the admission assessment of an older adult client. What assessment parameter will the nurse evaluate when using this scale? A) The client's current medication regimen B) The client's ability to change position C) The pigmentation of the client's skin D) The client's history of integumentary disorders

B) The client's ability to change position

30. The nurse is assessing a client's differentiation of self within the context of a broader family assessment. When assessing the client's differentiation of self, the nurse must determine which of the following? A) The client's rank order and communication style B) The client's emotional function and intellectual function C) The client's self-esteem and family responsibilities D) The client's strengths and weaknesses

B) The client's emotional function and intellectual function

29. The nurse is providing care for a client with a history of chronic heart failure. The client is in bed with the head of her bed at 45 degrees, and the nurse is assessing the client's neck veins. What assessment finding would be most consistent with a nursing diagnosis of fluid volume excess related to chronic heart failure? A) The client's carotid arteries are not palpable. B) The client's jugular veins are clearly visible and firm to palpation. C) The client's carotid pulses are asymmetrical and difficult to palpate. D) The client's carotid pulses are easier to palpate than the jugular pulses.

B) The client's jugular veins are clearly visible and firm to palpation.

30. The presence of faint pedal pulses in a client has prompted the nurse to perform a position change test for arterial insufficiency. What finding would suggest that the client may have arterial insufficiency? A) The client's legs are tender on palpation when in a dependent position. B) The client's legs are visibly pale when elevated above the examination table. C) The client's legs return to a pink color in 5 seconds. D) The client's legs develop pitting edema when he or she dangles them over the bedside.

B) The client's legs are visibly pale when elevated above the examination table.

30. The nurse is assessing the characteristics and positioning of the client's uvula, which deviates asymmetrically when the nurse has the client say aaah. This finding should prompt the nurse to focus on which of the following during subsequent assessment? A) The client's nutritional status B) The client's neurological status C) The client's immune function D) The client's respiratory function

B) The client's neurological status

4. A nurse is reviewing a colleague's documentation of a client assessment. The nurse reads that the client's radial pulse was 2+. How should the nurse interpret this assessment finding? A) The client's radial pulse occluded easily. B) The client's radial pulse occluded with moderate pressure. C) The client's radial pulse occluded with very firm pressure. D) The client's radial pulse could not be manually occluded.

B) The client's radial pulse occluded with moderate pressure.

2. During the health history, a young male client asks the nurse why his scrotum rises and relaxes. The nurse would incorporate knowledge of which of the following when responding to this client? A) When the temperature is warm, the scrotum rises. B) The cremasteric reflex controls the rise and relaxation of the scrotum. C) When the scrotum relaxes, it has many rugae. D) If the temperature is colder, the scrotum relaxes.

B) The cremasteric reflex controls the rise and relaxation of the scrotum.

8. The nurse has begun a client's assessment and is applying the blood pressure cuff on a client's arm. Which action would be most appropriate? A) The cuff is wrapped loosely around the arm. B) The cuff is placed about 1 inch above the antecubital area. C) The bladder inside the cuff encircles 50% of the arm circumference. D) The nurse can fit three to four fingers under the inflated cuff.

B) The cuff is placed about 1 inch above the antecubital area.

19. A nurse is preparing to perform the physical examination of an adult client who has presented to the clinic for the first time. Which of the following statements should guide the nurse's use of a stethoscope during this phase of assessment? A) Auscultation can be performed through clothing. B) The diaphragm should be held firmly against the body part. C) The bell of the stethoscope can best detect bowel sounds. D) Use of the bell is reserved for advanced practice nurses.

B) The diaphragm should be held firmly against the body part.

8. The nurse begins the physical examination of a newly admitted client by assessing the client's mental status. What is the nurse's best rationale for performing the mental status exam early in the assessment? A) The client will be less anxious early, providing the nurse with more accurate and reliable data. B) The exam can provide clues about the validity of the client's responses now and throughout. C) The exam provides data about mental health problems that the client may be afraid to report. D) The client's fears about having a serious illness may be alleviated by the results of the exam.

B) The exam can provide clues about the validity of the client's responses now and throughout.

27. The nurse is reviewing and analyzing data from the initial assessment of a newly admitted client who is a 79-year-old man. What assessment finding most clearly indicates a need for further data? A) The man has male pattern baldness. B) The man has a diffuse rash on his torso. C) The man's heart rate is 63 beats per minute. D) The man had an inguinal hernia repair in 2008.

B) The man has a diffuse rash on his torso.

24. A hospital nurse is performing a nutritional assessment of a 39-year-old obese client who has been recently diagnosed with type 2 diabetes. The nurse has completed the collection of subjective data and is preparing to proceed with objective data collection. Which principle should guide the nurse's subsequent actions? A) There are likely to be inconsistencies between subjective data and objective data. B) The nurse should be aware that the client may find assessment embarrassing. C) The nurse should avoid performing anthropometric measurements due to the client's obesity. D) The assessment should be performed over a series of brief sessions rather than one continuous assessment.

B) The nurse should be aware that the client may find assessment embarrassing.

29. The nurse reads in a client's electronic health record that her most recent ankle-brachial index (ABI) was 0.42. How should this assessment finding inform the nurse's care? A) The nurse should inspect the client's feet and ankles for venous ulcers once per shift. B) The nurse should implement interventions to address severe arterial insufficiency. C) The nurse should assess the client's extremities for pitting edema at least once per shift. D) The nurse should position the client to promote venous return.

B) The nurse should implement interventions to address severe arterial insufficiency.

28. The nurse has completed an assessment of a 36-year-old woman who was assaulted by her boyfriend. The nurse should ensure that documentation is particularly detailed and accurate for what reason? A) The nurse's scope of practice requires extensive documentation. B) The nurse's documentation may be used as legal evidence. C) The documentation must be validated with the client. D) The nurse's documentation must be notarized once it is complete.

B) The nurse's documentation may be used as legal evidence.

26. A nurse is consciously implementing the principles of therapeutic conversation during interactions with a family. Which of the following should characterize the nurse's communication? A) The nurse should improve the family's structure and function. B) The nurse's statements should be purposeful. C) The nurse should gather as much information as possible. D) The nurse should be motivated by a need to educate.

B) The nurse's statements should be purposeful.

19. A nurse is preparing a teaching session for a group of new parents about ear infections and measures to prevent them. The nurse is planning to address the reasons why children are more susceptible to these infections than adults. Which of the following would the nurse describe? A) Young children have a tendency to stick objects into their ear canal. B) The size and shape of children's eustachian tubes makes them vulnerable. C) Children's immune systems lack the maturity to fight infections. D) Children generally have poorer hygiene than adults.

B) The size and shape of children's eustachian tubes makes them vulnerable.

30. The school nurse has learned that a 14-year-old student is having social difficulties. According to Erikson, what is the most likely source of this child's stress? A) The student is experiencing moral dilemmas. B) The student is having difficulty creating an identity. C) The student is experiencing a sexual crisis. D) The student having difficulty understanding the viewpoints of others.

B) The student is having difficulty creating an identity.

2. A client admits to the nurse that she feels guilty for not providing more direct care for her ill mother. According to Freud, the moral component of this client's feelings results from which of the following? A) Defense mechanisms B) The superego C) The id D) The ego

B) The superego

29. The nurse is measuring a pregnant client's fundal height during a scheduled prenatal visit. The nurse should measure with reference to what anatomical landmarks? A) The edge of the fundus and the umbilicus B) The symphysis pubis and the fundus C) The fundus and the abdomen D) The xiphoid process and the symphysis pubis

B) The symphysis pubis and the fundus

1. A preadolescent girl comes to the clinic for a sports physical exam. The nurse notes beginning breast development and documents which of the following? A) Gynecomastia B) Thelarche C) Menarche D) Adolescence

B) Thelarche

14. After teaching a group of students about the traditional areas of auscultation of heart sounds, the instructor determines that the teaching was successful when the students identify which of the following as Erb's point? A) Fifth intercostal space near the left midclavicular line B) Third to fifth intercostal space at the left sternal border C) Second intercostal space at the right sternal border D) Second or third intercostal space at the left sternal border

B) Third to fifth intercostal space at the left sternal border

24. A new mother rings her call bell after giving birth to a healthy infant 18 hours earlier. The client states that her infant looks like she has milk coming out of her nipples. How should the nurse best interpret this phenomenon? A) The infant is showing signs of postnatal mastitis. B) This is a normal finding that results from hormonal stimulation. C) This is an expected finding in female infants but an unexpected finding in male infants. D) The nurse should plan to manually express the liquid from the infant's breasts.

B) This is a normal finding that results from hormonal stimulation.

24. The nurse is conducting a focused neurological assessment of an 81-year-old client. When analyzing the assessment data, the nurse should be aware of what age-related neurological change? A) Impaired judgment B) Tremors accompanying intentional movements C) Loss of remote memory D) Loss of sensation in distal extremities

B) Tremors accompanying intentional movements

20. A nurse is testing a client's deep tendon reflex. The nurse taps the tendon above the olecranon process. The nurse is assessing which reflex? A) Brachioradialis B) Triceps C) Biceps D) Achilles

B) Triceps

12. The nurse is preparing to perform the physical examination of an older adult client who will begin rehabilitation from an ischemic stroke. Which of the following actions would be most appropriate? A) Omit intrusive parts of the exam. B) Try to minimize position changes. C) Allow client to remain dressed. D) Dim the room light to ensure privacy.

B) Try to minimize position changes.

13. A nurse is performing a head-to-toe assessment and is preparing to examine the client's ears. Which equipment would the nurse need to have readily available? A) Ophthalmoscope B) Tuning fork C) Facial tissues D) Stethoscope

B) Tuning fork

14. A nurse is interviewing a child who is suspected of being abused. Which of the following would be most appropriate? A) Ask questions that are highly detailed. B) Use direct, nonleading questions. C) Offer the child a reward for answering questions. D) Use simple yes and no questions regardless of the child's age.

B) Use direct, nonleading questions.

15. A nurse is obtaining a client's radial pulse. Which of the following actions demonstrates correct technique for this assessment? A) Application of firm pressure on the wrist area along the side of the fifth digit B) Use of two middle fingers lightly applied to wrist area along the thumb side C) Use of the thumb and index finger applied to obliterate the wrist area along the thumb side D) Application of the bell of the stethoscope to the antecubital area of the upper extremity

B) Use of two middle fingers lightly applied to wrist area along the thumb side

4. A nurse is interviewing a 22-year-old client of the campus medical clinic. Which nonverbal behavior should the nurse adopt to best facilitate communication during this phase of assessment? A) Standing while the client is seated B) Using a moderate amount of eye contact C) Sitting across the room from the client D) Minimizing facial expressions

B) Using a moderate amount of eye contact

2. A nurse assesses the skin of an older adult's forearms and observes purpura. The nurse interprets this finding as indicative of which of the following? A) Elder abuse B) Vascular fragility C) Poor circulation D) Herpes zoster

B) Vascular fragility

10. A nurse is analyzing the assessment data of a client who has been admitted with exacerbation of heart failure. The nurse has determined that the cue clusters meet the defining characteristics of specific nursing diagnoses. Which of the following would the nurse do next? A) Explain the client's problems to the client and his or her family. B) Verify it with the client and with other health care professionals. C) Validate the diagnosis with the physician. D) Work with the client to begin planning interventions.

B) Verify it with the client and with other health care professionals.

17. A group of students is reviewing the vertical reference lines of the thorax. They demonstrate understanding when they identify which line as a reference line for the posterior thorax? A) Midaxillary line B) Vertebral line C) Right midclavicular line D) Sternal line

B) Vertebral line

12. An older adult client reports that he is experiencing severe trunk pain and is concerned that he might have shingles. Which type of lesion would the nurse most likely assess? A) Papule B) Vesicle C) Bulla D) Crust

B) Vesicle

28. A nurse will be working in a clinic in South Asia for several weeks, where the majority of residents have darkly pigmented skin. The nurse should expect a higher-than-average incidence of what integumentary health problem? A) Contact dermatitis B) Vitiligo C) Psoriasis D) Eczema

B) Vitiligo

22. A client has sought care at the clinic, telling the nurse, This ringing in my ears has gone on for weeks, and it's driving me crazy. The patient denies exposure to excessive noise levels. The nurse recognizes the likely presence of tinnitus and should follow up with which of the following questions? A) Did your parents even complain of something similar? B) What medications are you currently taking? C) How would you describe your overall level of health? D) How do you usually clean your ears?

B) What medications are you currently taking?

23. A client has admitted to the nurse that he has been having difficulty obtaining and maintaining erections for many months. Which of the nurse's assessment questions most clearly addresses a potential cause for the client's problem? A) How would you describe a typical day's food intake? B) What medications are you currently taking? C) Have you ever been screened for prostate cancer? D) Do you ever experience pain when you urinate?

B) What medications are you currently taking?

26. The nurse is assessing an adult client's self-image during the health history interview. What assessment question is most likely to elicit meaningful data? A) What are the activities that give you the most joy? B) What would you describe as your main strengths and weaknesses? C) Do you consider yourself to be a particularly religious person? D) What actions are you taking to improve your life?

B) What would you describe as your main strengths and weaknesses?

1. The nurse is preparing to assess the remote memory of a client who has a diagnosis of early stage Alzheimer's disease. Which question would be most appropriate for the nurse to use? A) Can you tell me what you have eaten in the last 24 hours? B) When did you get your first job? C) What did you do last evening? D) How are an apple and orange the same?

B) When did you get your first job?

27. A nurse is beginning a scheduled follow-up meeting with a family by providing a commendation. What is the best example of a commendation? A) Everyone in your family is kind and considerate. B) Your family shows great strength by caring for your grandmother in her own home. C) Your family is much more compassionate than most of the families with which I have contact. D) Your family definitely chose the best plan to improve your grandmother's health outcomes.

B) Your family shows great strength by caring for your grandmother in her own home.

3. The nurse has asked a female client if she has noticed any lumps or swelling in her breasts. After the client responds "yes," which question should the nurse ask next? A) "Have any of the other women in your family had this happen?" B) "Has there been any corresponding change in your breast size?" C) "Does the lump change over the course of your menstrual cycle?" D) "What do you think is causing this change?"

C) "Does the lump change over the course of your menstrual cycle?"

16. A client's elevated body mass index (BMI) has prompted the nurse to assess the client's activity and exercise level. Which statement would indicate to the nurse that the client is getting the recommended amount of exercise? A) "I walk briskly on the treadmill once or twice a week." B) "I play basketball with a team every Friday night without fail." C) "I go to a step class for an hour three times a week." D) "I swim for at least half an hour each Saturday morning."

C) "I go to a step class for an hour three times a week."

23. The nurse is obtaining information about a client's past health history. Which client statement would best reflect this component of assessment? A) "My mom's still alive, but my dad died 10 years ago of heart failure." B) "I have a brother with leukemia and a sister with hypertension." C) "I had surgery 5 years ago to repair an inguinal hernia." D) "I have been having some pain when I urinate for the last several days."

C) "I had surgery 5 years ago to repair an inguinal hernia."

12. A clinic nurse has reviewed a new client's available health record and will now begin taking the client's health history. Which of the following questions should the nurse ask first when obtaining the health history? A) "Do you have adequate health insurance coverage?" B) "Are you generally fairly healthy?" C) "What is your major health concern at this time?" D) "Did you bring all your medications with you?"

C) "What is your major health concern at this time?"

3. A 71-year-old woman has been admitted to the hospital for a vaginal hysterectomy, and the nurse is collecting subjective data prior to surgery. Which statement by the nurse could be construed as judgmental? A) "How often do your adult children typically visit you?" B) "Your husband's death must have been very difficult for you." C) "You must quit smoking because it affects others, not only you." D) "How would you describe your feelings about getting older?"

C) "You must quit smoking because it affects others, not only you."

14. A woman appears restless and is wringing her hands prior to having a clinical breast examination performed. Which statement by the nurse would be most appropriate? A) "I know you are worried, but your risk for cancer is low." B) "You need to pay attention to these instructions so we can finish as quickly as possible." C) "You seem to be anxious. Can you tell me what you are thinking?" D) "You appear restless but I can assure you that your doctor is very good."

C) "You seem to be anxious. Can you tell me what you are thinking?"

17. A nurse is determining a client's ankle-brachial index. Which result would indicate to the nurse that the client's circulation is normal and free of arterial occlusion? A) 0.5 B) 0.8 C) 1.1 D) 1.4

C) 1.1

21. A clinic nurse is assessing a 6-month-old infant prior to the administration of scheduled immunizations. The nurse should anticipate that the infant's resting heart rate will be nearest to what value? A) 80 beats per minute B) 100 beats per minute C) 120 beats per minute D) 140 beats per minute

C) 120 beats per minute

7. The nurse assesses the uterine fundus and finds it to be halfway between the symphysis pubis and the umbilicus. The nurse knows that this is an expected finding at how many gestational weeks? A) 6 B) 12 C) 16 D) 20

C) 16

8. The nurse is inspecting a client's tonsils and notes that they make contact with the client's uvula. The nurse would document this finding as which of the following? A) 1+ B) 2+ C) 3+ D) 4+

C) 3+

12. A mother voices concern about the amount of time her school-age child sleeps. When responding to the mother, the nurse understands that this age group sleeps an average of how many hours each night? A) 11 to 12 B) 9 to 10 C) 8 to 9.5 D) 7 to 8

C) 8 to 9.5

29. During the mental status assessment of a new client, the nurse has asked the client to describe some of the similarities and differences between a tennis ball and a soccer ball. Despite adequate time and cuing, the client is unable to state any similarities or differences. The nurse should document what assessment finding? A) A deficit in practical intelligence B) An inability to follow directions accurately C) A deficit in abstract reasoning D) A lack of spatial orientation

C) A deficit in abstract reasoning

29. A client's recent weight loss and diarrhea has been attributed to hyperthyroidism. When auscultating the client's thyroid gland, what assessment finding is most consistent with this diagnosis? A) Audible referred breath sounds at the site of the thyroid B) An audible S3 sound at the site of the thyroid C) A sound of turbulent blood flow in the thyroid D) Irregular S1 and S2 rhythms in the thyroid

C) A sound of turbulent blood flow in the thyroid

21. The nurse is caring for a client who has been diagnosed with colon cancer. When planning the client's care, the nurse should be aware of what function of the colon? A) Absorbing electrolytes B) Secreting digestive enzymes C) Absorbing large amounts of water D) Secreting bile

C) Absorbing large amounts of water

12. Which of the following principles should inform the nurse's practice when assessing a client for possible physical abuse? A) The abuse is usually part of the client's presenting health problem. B) Physical abuse is thought to occur in about 5% of women. C) Abuse may start at any time during a relationship. D) Abuse rarely occurs in women younger than age 25 years.

C) Abuse may start at any time during a relationship.

10. A nurse is planning a program to address measures to reduce the leading cause of mortality in children age 1 to 14 years. Which of the following topics would best address this cause of death? A) Gun safety B) Prenatal care C) Accident prevention D) Substance abuse prevention

C) Accident prevention

15. When reviewing cultural differences that relate to the incidence and prevalence of disease among various cultural groups, the nurse would expect to see the highest prevalence of asthma in which group? A) Non-Hispanic blacks B) Caucasians C) African Americans D) Southeast Asians

C) African Americans

3. When describing cultural differences related to tooth eruption, the nurse explains that permanent teeth typically appear earlier in which group? A) Caucasians B) Hispanics C) African Americans D) Native Americans

C) African Americans

8. The clinic nurse is reviewing the medication history of a 39-year-old woman. Which medication would the nurse identify as a potential risk factor for thrombophlebitis? A) A beta-adrenergic blocker B) A selective serotonin reuptake inhibitor (SSRI) C) An oral contraceptive D) An antilipid agent

C) An oral contraceptive

11. The nurse is collecting data from a client who has recently been diagnosed with type 1 diabetes and who will begin an educational program. The nurse is collecting subjective and objective data. Which of the following would the nurse categorize as objective data? A) Family history B) Occupation C) Appearance D) History of present health concern

C) Appearance

29. While assisting an older adult with morning hygiene, the nurse notes a lesion on the client's coccyx region. How should the nurse best document this objective assessment finding? A) Possible pressure ulcer observed over client's coccyx region. B) Reddened area noted on skin surface superficial to client's coccyx. C) Area of nonblanching erythema noted over client's coccyx, 2 cm x 2 cm. D) Impaired Skin Integrity related to decreased mobility.

C) Area of nonblanching erythema noted over client's coccyx, 2 cm ◊ 2 cm.

20. When assessing a client for possible oral cancer, the nurse should most closely inspect which area? A) Buccal mucosa B) Hard palate C) Area under the tongue D) Along the gum line

C) Area under the tongue

11. Which of the following would the nurse expect to assess when examining the eyes of a client who reports a history of severe allergies? A) Generalized redness B) Pinguecula C) Areas of dryness D) Nodular appearance

C) Areas of dryness

8. The nurse would test for stereognosis during which part of the comprehensive exam? A) Posterior and lateral chest B) Nose and sinuses C) Arms, hands, and fingers D) Legs, feet, and toes

C) Arms, hands, and fingers

21. A client has presented for care to the clinic, stating, "I'm pretty sure that I feel a new lump in my breast." After confirming the presence of a lump, what action should the nurse take? A) Arrange for the client to be brought to the hospital emergency department immediately. B) Tell the client to monitor the lump for the next three weeks and seek care if it increases in size. C) Arrange for a prompt referral to her primary care provider. D) Facilitate a referral to an oncologist if more lumps emerge in the coming weeks.

C) Arrange for a prompt referral to her primary care provider.

23. A factory worker has presented to the occupational health nurse with a small wood splinter in his left eye. The nurse has assessed the affected eye and irrigated with warm tap water, but the splinter remains in place. What should the nurse do next? A) Attempt to remove the splinter using sterile forceps. B) Irrigate the eye with dilute hydrogen peroxide. C) Arrange for worker to be promptly assessed by an eye specialist. D) Encourage the worker to see an optometrist as soon as possible.

C) Arrange for worker to be promptly assessed by an eye specialist.

16. The nurse is assessing the skin condition and color of an African-American client. Which of the following would the nurse document as an abnormal finding? A) Evenly distributed color B) Light to medium dark brown skin C) Ashen gray skin color D) Lack of visible pores

C) Ashen gray skin color

10. While using an otoscope to assess the ears of an 8-year-old boy, the nurse observes white spots on the boy's tympanic membrane. The nurse also observes that no redness is present. Which action would be most appropriate? A) Assess the boy for previous trauma to his skull. B) Determine whether impacted cerumen is present. C) Ask the mother whether the child has had numerous ear infections. D) Assess the child for further symptoms of acute otitis media.

C) Ask the mother whether the child has had numerous ear infections.

26. An older adult client who enjoys good overall health has sought care because of a recent onset of weakness and fatigue. The client is unaware of any precipitating events. How should the nurse proceed with assessment? A) Perform a focused respiratory assessment. B) Obtain the client's vaccination history. C) Assess the client for signs and symptoms of anemia. D) Assess the client for evidence of chronic heart failure.

C) Assess the client for signs and symptoms of anemia.

28. The nurse palpates a client's pulse and notes that the rate is 61 beats per minute, with an amplitude that is weak and thready. How should the nurse respond to this assessment finding? A) Call a code blue from the bedside and prepare for resuscitation. B) Assess the client's jugular venous pressure. C) Assess the client's pulse at the carotid site. D) Palpate the client's femoral pulse.

C) Assess the client's pulse at the carotid site.

2. When assessing the mental status of a 67-year-old woman, the nurse detects some difficulty with free-flow of thought and the woman's ability to follow directions. Which of the following would the nurse do first? A) Use a Geriatric Depression Scale. B) Refer for further medical evaluation. C) Assess the client's vision and hearing. D) Refer the client to social services for home assistance.

C) Assess the client's vision and hearing.

17. A nurse has documented the findings of a comprehensive assessment of a new client. What is the primary rationale that the nurse should identify for accurate and thorough documentation? A) Guaranteeing a continual assessment process B) Identifying abnormal data C) Assuring valid conclusions from analyzed data D) Allowing for drawing inferences and identifying problems

C) Assuring valid conclusions from analyzed data

15. The nurse is preparing to measure the chest circumference of a 2-day-old newborn. The nurse would place the tape measure at which area? A) High up under the axillary area B) At the level of the umbilicus C) At the level of the nipple line D) Midway between the nipple line and umbilicus

C) At the level of the nipple line

28. The nurse is assessing the head and neck of a 51-year-old male client. Following inspection and palpation of the client's thyroid gland, the nurse determines that the gland is enlarged. What is the next action that the nurse should perform? A) Obtain a full set of vital signs. B) Percuss the client's thyroid. C) Auscultate the client's thyroid. D) Perform a swallowing assessment.

C) Auscultate the client's thyroid.

10. A nurse has completed gathering some basic data about a client who has multiple health problems that stem from heavy alcohol use. The nurse has then reflected on her personal feelings about the client and his circumstances. The nurse does this primarily to accomplish which of the following? A) Determine if pertinent data has been omitted B) Identify the need for referral C) Avoid biases and judgments D) Construct a plan of care

C) Avoid biases and judgments

5. During an assessment of an elderly client, the nurse notes a decrease in pupil size and a slowed reaction of the pupil to light. Accommodation and convergence are normal. Based on these findings, which of the following should the nurse emphasize with client education? A) Use drops to prevent dryness B) Wear sunglasses outdoors C) Avoid driving at night D) Obtain an eye examination

C) Avoid driving at night

20. While inspecting the vagina, the nurse observes a thin, grayish-white vaginal discharge with a fishy odor. Which of the following would the nurse suspect? A) Moniliasis B) Trichomoniasis C) Bacterial vaginosis D) Atrophic vaginitis

C) Bacterial vaginosis

19. Inspection of a client's knee reveals swelling, and the nurse suspects that there is significant fluid in the knee. Which of the following would the nurse use to confirm the suspicion? A) Phalen's test B) Tinel's test C) Ballottement test D) Leg raising test

C) Ballottement test

1. To examine the Bartholin's glands of a female client, the nurse would palpate at which anatomic location? A) On both sides of the clitoris B) Just inside the urethral orifice C) Between the vaginal opening and labia minora D) Inside the vaginal orifice

C) Between the vaginal opening and labia minora

29. The nurse is auscultating the bowels of an infant who was born 10 hours ago. What principle should guide the nurse's assessment and data analysis? A) Bowel sounds are not normally audible until 48 to 72 hours postpartum. B) Bowel sounds are not normally audible until 24 to 48 hours postpartum. C) Bowel sounds should be audible every 10 to 30 seconds. D) Bowel sounds should be absent at rest and audible after palpation.

C) Bowel sounds should be audible every 10 to 30 seconds.

5. The nurse is completing a head-to-toe assessment of a pregnant client. What anatomic area should be examined when assessing the Montgomery tubercles? A) Thorax B) Abdomen C) Breasts D) Perineum

C) Breasts

17. The nurse is preparing to perform Leopold's maneuvers. During the first maneuver, the nurse palpates a soft mass in the upper quadrant of the abdomen. The nurse interprets this as which fetal part? A) Back B) Head C) Buttocks D) Feet

C) Buttocks

21. A nurse is assessing the head and neck of an adult client. Which vertebra should the nurse identify as a landmark in order to locate the client's other vertebrae? A) C3 B) C5 C) C7 D) T2

C) C7

23. The nurse is performing an abbreviated head-to-toe assessment of a hospital client. What question should the nurse ask when assessing the client's level of consciousness? A) If there were a fire in your house, what would you do? B) How would you describe your overall level of stress? C) Can you tell me the current month and year? D) Can you tell me what you ate for breakfast this morning?

C) Can you tell me the current month and year?

9. The nurse is preparing to perform a nutritional assessment of a newly admitted client. Which of the following questions would be most appropriate to use when initiating the assessment? A) Did you eat breakfast today? B) How many meals do you eat each day? C) Can you tell me what you've eaten in the last 24 hours? D) How often do you eat out?

C) Can you tell me what you've eaten in the last 24 hours?

25. The nurse has had a client place the backs of both her hands against each other while flexing her wrists 90 degrees with fingers pointed downward and wrists dangling. The presence of pain or tingling during this test suggests what health problem to the nurse? A) Osteoarthritis B) Diabetic neuropathy C) Carpal tunnel syndrome D) Gouty arthritis

C) Carpal tunnel syndrome

6. A nurse who provides care in a busy, inner-city clinic performs physical examinations on clients of various cultures. In a client from which group would the nurse expect to find the greatest amount of body odor from perspiration? A) Inuit B) Asian C) Caucasian D) Native American

C) Caucasian

9. A nurse has assessed a client and identified data that are associated with the diagnoses of Impaired Physical Mobility and Activity Intolerance. How can the nurse best determine which nursing diagnosis is most applicable to the client? A) Document preliminary conclusions. B) Identify abnormal data. C) Check the defining characteristics of the diagnoses. D) Test the nursing diagnoses clinically.

C) Check the defining characteristics of the diagnoses.

12. During the health interview, the nurse notes that a client is a mouth breather. The client denies nasal congestion and has a healthy body mass index. Which of the following would be most important for the nurse to assess? A) Asking if the client experiences dry mouth often B) Inspecting for inflammation of the tonsils C) Checking for a deviated nasal septum D) Performing a focused respiratory assessment

C) Checking for a deviated nasal septum

18. A nurse is providing an in-service presentation to a group of new pediatric nurses and reviewing differences in assessment of children and adults. When describing the heart sound typically auscultated in children in comparison to an adult, which characteristic would the nurse describe? A) Children typically have softer heart sounds. B) Children typically have less harsh heart sounds. C) Children typically have higher pitched heart sounds. D) Children typically have heart sounds of longer duration.

C) Children typically have higher pitched heart sounds.

7. Upon entering the examination room, the nurse observes that the client is leaning forward with his arms supporting his body weight. The nurse would recognize this as a tripod position and suspect the presence of which of the following medical problems? A) Pleural effusion B) Heart failure C) Chronic obstructive pulmonary disease D) Pneumonia

C) Chronic obstructive pulmonary disease

17. A nurse is constructing a genogram of a family. Assessment reveals that the maternal grandmother died at age 69. The nurse would depict this person on the genogram using which symbol? A) Triangle with a line through it B) Square with a line surrounding it C) Circle with an x through it D) Connecting line with two small lines through it

C) Circle with an x through it

14. When counseling a family regarding verbal communication patterns, the nurse should encourage which type of messages to foster open communication? A) Metaphorical B) Displaced C) Clear D) Obtainable

C) Clear

14. While inspecting the penis of a client, the nurse suspects herpes progenitalis based on which assessment finding? A) Red, oval ulcerations B) Hardened nodules on the glans C) Clear vesicles that erupt D) Painless, fleshy papules

C) Clear vesicles that erupt

19. The nurse is documenting findings of a comprehensive assessment. Which statement would be categorized as part of the general survey? A) Hair neat and clean with white and gray streaks; no scalp lesions noted B) Sclera white; conjunctiva slightly reddened without lesions C) Client alert and cooperative; sitting comfortably on chair with hands in lap D) Head symmetrically round; neck nontender with full range of motion

C) Client alert and cooperative; sitting comfortably on chair with hands in lap

8. A 54-year-old client is receiving a follow-up assessment in a clinic, following abnormal findings on her recent mammogram. Which of the following statements best reflects appropriate documentation by the nurse? A) Client depressed because of fear of breast biopsy B) Client with lower back pain C) Client has unkempt appearance and avoids eye contact D) Client has good lung sounds in right and left lungs

C) Client has unkempt appearance and avoids eye contact

19. When assessing a client's spirituality, the nurse has the client complete a Brief Religious Coping Questionnaire. When reviewing the completed questionnaire, the nurse identifies which of the following as indicating positive religious coping? A) Client feels stress is something from God that is to be endured. B) Client wonders if God has abandoned him or her. C) Client looks to God for support in a crisis. D) Client decides what to do without relying on God.

C) Client looks to God for support in a crisis.

11. After having a client perform a Romberg test, which of the following would indicate to the nurse that the test is negative? A) Client moves the feet apart during the test B) Client sways slightly during the test C) Client maintains the position during the test D) Client keeps his or her eyes close during the test

C) Client maintains the position during the test

15. The nurse is preparing to perform a speculum examination on a client. The nurse lubricates the speculum with which of the following? A) Petroleum jelly B) Water-soluble lubricant C) Client's vaginal secretions D) Antimicrobial ointment

C) Client's vaginal secretions

24. A female client with bone cancer is experiencing pain that has become more severe over the past several days. When modifying the client's plan of care, the nurse identifies a need to assess the affective dimension of the client's pain. How can the nurse best accomplish this goal? A) Document the ways that the client's pain affects her activities of daily living. B) Determine whether the client is able to independently treat her pain. C) Closely monitor the effects of the client's pain on her emotions. D) Ask the client to rate her pain during every physiological assessment.

C) Closely monitor the effects of the client's pain on her emotions.

25. The nurse recognizes the need to screen a middle-aged client for intimate partner violence, but the client's partner is remaining close to the client. As a result, the nurse is unable to screen the client in privacy. What action should the nurse next take? A) Document the fact that the client is suspected of being a victim of IPV. B) Prioritize the results of the physical assessment. C) Closely observe the client's interactions with the partner. D) Assess the client's psychosocial status and mental status.

C) Closely observe the client's interactions with the partner.

6. A nurse assesses the pulses of an infant and notes that the femoral pulses are weak. Which of the following health problems should the nurse suspect? A) Right ventricular enlargement B) Sinus arrhythmia C) Coarctation of the aorta D) Patent ductus arteriosus

C) Coarctation of the aorta

18. A nurse has received a report on a client who will soon be admitted to the medical unit from the emergency department. When preparing for the assessment phase of the nursing process, which of the following should the nurse do first? A) Collect objective data. B) Validate important data. C) Collect subjective data. D) Document the data.

C) Collect subjective data.

10. Assessment of a client who has suffered a recent stroke reveals that he is unresponsive to all stimuli and his eyes remain closed. The nurse documents the client's level of consciousness as which of the following? A) Obtunded B) Stupor C) Coma D) Lethargy

C) Coma

21. An experienced medical-surgical nurse has identified critical thinking as an integral component of diagnostic reasoning. How can the relationship between these two concepts be best described? A) Critical thinking is the practical application of diagnostic reasoning skills. B) Critical thinking and diagnostic reasoning are synonymous. C) Critical thinking is the foundation of the process of diagnostic reasoning. D) Critical thinking is the domain of the novice nurse, whereas diagnostic reasoning is present in experts.

C) Critical thinking is the foundation of the process of diagnostic reasoning.

10. A task force has been established at a hospital with the aim of overhauling the assessment forms that are used throughout the facility. Which of the following options is most likely to help standardize the process of data collection? A) Open-ended form B) Integrated cued checklist form C) Cued or checklist form D) Nursing minimum data set

C) Cued or checklist form

30. A nurse has completed the assessment of an older adult client's head and neck and is now analyzing the assessment findings. Which of the following findings should the nurse attribute to age-related physiological changes? A) Increased size of a single thyroid nodule B) A nonpalpable carotid pulse C) Decreased strength of temporal artery pulsations D) Tenderness of lymph nodes on palpation

C) Decreased strength of temporal artery pulsations

12. The advanced practice nurse is preparing to perform a pelvic examination on an elderly female client. Which of the following would the nurse expect to find? A) Elongation of the vagina B) Thick, pale epithelium C) Decreased vaginal secretions D) Palpable ovaries

C) Decreased vaginal secretions

18. A client who has fractured her arm is describing her pain as excruciating. The nurse determines that the client is most likely experiencing what type of pain? A) Cutaneous B) Visceral C) Deep somatic D) Radiating

C) Deep somatic

12. A nurse is preparing to complete a comprehensive health assessment on a female client. Prior to beginning the assessment, the client states, I'm really having a good deal of pain in my hip now. Which of the following would be most appropriate for the nurse to do? A) Begin the comprehensive assessment and aim to complete it efficiently. B) Explain the reason for the client's assessment. C) Delay the full exam until the client's pain has been addressed. D) Provide education on pain control.

C) Delay the full exam until the client's pain has been addressed.

30. In preparation for discharge, the nurse is assessing a newborn infant's hearing acuity. How should the nurse best perform this assessment? A) Determine whether the infant turns his or her head toward verbal stimuli. B) Determine whether the infant makes eye contact in response to a loud voice. C) Determine whether a loud noise near the infant evokes a startle response. D) Determine whether the infant appears to recognize the mother's voice.

C) Determine whether a loud noise near the infant evokes a startle response.

3. Which of the following would the nurse most likely find when assessing a client diagnosed with a frontal lobe contusion following a motor vehicle accident? A) Inability to hear high-pitched sounds B) Loss of tactile sensation C) Difficulty speaking D) Blurred vision

C) Difficulty speaking

30. A young man has presented to the clinic with a 2-week history of head congestion, fever, and malaise. What assessment technique should the nurse utilize to assess for sinus tenderness? A) Light palpation B) Deep palpation C) Direct percussion D) Blunt percussion

C) Direct percussion

6. The nurse has identified a need to discuss sexuality with a 15-year-old client. How should the nurse best plan this aspect of the health interview? A) Obtain informed consent for the health interview. B) Begin by explaining appropriate and acceptable sexual behavior. C) Discuss the matter when a parent is not present. D) Ensure that a chaperone is in the room during the interview.

C) Discuss the matter when a parent is not present.

2. The nurse in a prenatal clinic is performing an assessment on a pregnant client. When it is noted that clumps of hair are missing from the client's scalp, the nurse should ask what assessment question? A) Have you ever been the victim of a crime? B) What do you know about the problem of domestic violence? C) Do you feel safe in your home setting? D) Can you tell me if anyone recent attacked you?

C) Do you feel safe in your home setting?

21. A nurse is integrating health promotion education into the assessment of a client's mouth, nose, and throat. What interview question is most likely to identify a risk factor for oral cancer? A) Would you say that you're prone to getting mouth ulcers? B) Do you brush and floss daily? C) Do you use tobacco, whether smoking or chewing? D) How often do you usually go to the dentist in a year?

C) Do you use tobacco, whether smoking or chewing?

24. The nurse has attempted to palpate the client's popliteal pulses but is unable to feel them, despite confirming appropriate landmarking and client positioning. What is the nurse's best response? A) Advocate for a referral to a vascular surgeon. B) Have the client perform light physical activity to promote circulation and then reattempt. C) Document the finding and proceed with the assessment. D) Palpate the client's brachial pulse.

C) Document the finding and proceed with the assessment.

7. The nurse wants to assess a family's interactions with the systems outside of the family. Which tool would be most appropriate to use? A) Genogram B) Family attachment diagram C) Ecomap D) Calgary Family Assessment Model

C) Ecomap

28. The nurse is palpating a pregnant client's left and right adnexa. The presence of a palpable mass should prompt the nurse to refer the client promptly for what problem? A) Abruptio placentae B) Placenta previa C) Ectopic pregnancy D) Incompetent cervix

C) Ectopic pregnancy

30. The nurse has assessed a client's neck vessels and is now preparing to auscultate the client's heart sounds. What action should the nurse perform during this phase of assessment? A) Rapidly auscultate all areas of the precordium and then repeat the assessments in greater detail. B) Stand on the client's left side, nearest the heart. C) Elevate the head of the client's bed to 30 degrees. D) Begin by auscultating the entire precordium with the bell of the stethoscope.

C) Elevate the head of the client's bed to 30 degrees.

22. An older adult client has a body mass index of 15.5 and is consequently considered to be underweight. The client lives alone and states that she has never been a heavy eater. How can the nurse most accurately assess the client's nutritional habits? A) Assess the client's waist circumference and waist-to-hip ratio. B) Measure the client's mid-arm circumference. C) Elicit the client's 24-hour food recall. D) Have the client describe an ideal meal.

C) Elicit the client's 24-hour food recall.

17. After teaching a group of students about the brain and spinal cord, the instructor determines that the students demonstrate the need for additional teaching when they identify which of the following as being controlled by the brain stem? A) Respiratory function B) Heart rate C) Equilibrium D) Reflex actions

C) Equilibrium

4. A novice nurse is practicing how to complete a comprehensive assessment to gain confidence and skill. Which of the following would be most important for the nurse to remember? A) Always gather objective data before subjective data. B) Intersperse the physical exam with the history. C) Establish a routine for the assessment. D) Allow the client a break between the two parts of the history/exam.

C) Establish a routine for the assessment.

12. A nurse assesses a client's epitrochlear nodes and finds them to be enlarged and tender. Which of the following would the nurse do next? A) Ask the client about any recent ear and throat infections. B) Carefully assess the cervical lymph nodes for enlargement. C) Examine the lower arm and hand for infection sites. D) Assess both legs for Homans' sign.

C) Examine the lower arm and hand for infection sites.

16. A nurse is applying the diagnostic reasoning process in the care of a client with a number of comorbidities. Which of the following descriptions best characterizes Step Two, Clustering Data? A) Hypothesizing of any potentially applicable health promotion diagnoses, risk diagnoses, and actual diagnoses B) Documentation of all professional judgments along with any data that support those judgments C) Examining identified abnormal findings and strengths for cues that are related D) Evaluation of both subjective and objective data to identify strengths and abnormal findings

C) Examining identified abnormal findings and strengths for cues that are related

17. The nurse is preparing to examine a client's skin. Which of the following actions would be most important for the nurse to do? A) Ensure that the room is hot to prevent chilling. B) Wear gloves when preparing to inspect the skin and nails. C) Expose only the body part that is being examined. D) Have the client remove clothing from the upper body.

C) Expose only the body part that is being examined.

7. A nurse is preparing a presentation for a local community group about preventing traumatic brain injury. The nurse would discuss which measure as prevention of the leading cause? A) Safe use of firearms B) Safe use of machinery C) Falls prevention D) Domestic violence prevention

C) Falls prevention

16. When percussing the scapula of a client, which of the following would the nurse expect to hear? A) Resonance B) Dullness C) Flatness D) Hyperresonance

C) Flatness

5. Which of the following would the nurse expect to find when examining a client with a herniated lumbar disc? A) Rounded thoracic convexity B) Lumbar lordosis C) Flattened lumbar curve D) Lateral curvature of the spine

C) Flattened lumbar curve

25. A client comes to the health care provider's office for a visit. The client has been seen in this office on occasion for the past 5 years and arrives today complaining of a fever and sore throat. Which type of assessment would the nurse most likely perform? A) Comprehensive assessment B) Ongoing assessment C) Focused assessment D) Emergency assessment

C) Focused assessment

20. After teaching a group of students about the second phase of the nursing process, the instructor determines that additional teaching is needed when the students identify which of the following as a component? A) Organizing data B) Clustering data C) Formulating a medical diagnosis D) Generating hypotheses

C) Formulating a medical diagnosis

12. A client has large, pendulous breasts. Which of the following would be most appropriate to ensure better access while examining the client's breasts for retraction and dimpling? A) Have the client stand and lean forward B) Have the client lie on her side C) Have the client sit and then lean forward D) Have the client lie flat on her back

C) Have the client sit and then lean forward

6. Which question would be most important to ask when obtaining the nursing health history of a male client with extensive peripheral vascular disease? A) What dietary supplements do you take? B) When was your last prostate exam for cancer? C) Have you experienced a change in your usual sexual activity? D) Have you had an electrocardiogram recently?

C) Have you experienced a change in your usual sexual activity?

18. An instructor is describing various ways that a nurse can validate data to a group of nursing students. The instructor determines that additional teaching is necessary when the students identify which of the following as a reliable method? A) Repeating the assessment B) Asking additional questions C) Having the client repeat what was said D) Checking findings with another health care professional

C) Having the client repeat what was said

5. When assessing a community, the nurse is reviewing statistics related to adult mortality in clients who are 65 years of age and older. The nurse would assess the community for health programs to address which of the following as the major cause of death in this age group? A) Unintentional injuries B) Chronic lower respiratory diseases C) Heart disease D) Diabetes

C) Heart disease

8. The nurse collects vital signs on a hospital client who has recently been experiencing pain. Which of the following would suggest most strongly to the nurse that the client is experiencing pain? A) Respiratory rate of 18 breaths per minute B) Temperature of 99.1∞F C) Heart rate of 110 beats per minute D) Blood pressure of 120/70 mm Hg

C) Heart rate of 110 beats per minute

11. A nurse is preparing a program on osteoporosis for a local women's group. Which of the following should the nurse cite as a risk factor? A) Obesity B) Multiparity (multiple pregnancies) C) History of smoking D) African-American ethnicity

C) History of smoking

17. When palpating a female client's axillae, which of the following actions is most appropriate? A) Have the client hold the arm of the side being examined slightly away from the body. B) Tell the client to raise her arm on the side being examined up over her head. C) Hold the client's elbow of the side being examined with one hand. D) Have the client lean forward from the waist with arms outstretched.

C) Hold the client's elbow of the side being examined with one hand.

25. The nurse is using Doppler ultrasound to auscultate the peripheral pulses of a client with peripheral vascular disease. What action should the nurse perform during this assessment? A) Gently cool the client's extremities to aid auscultation. B) Apply a small amount of petroleum gel to the Doppler probe. C) Hold the probe at a 60- to 90-degree angle to the client's skin. D) Push the probe firmly against the skin to enhance audibility.

C) Hold the probe at a 60- to 90-degree angle to the client's skin.

22. The nurse is conducting the health interview of an adult client who has sought care because of a wicked cough leading to dyspnea. When trying to differentiate between pathologic lung changes and an infection as the etiology of the client's cough and resultant dyspnea, what interview question should the nurse ask? A) Does your cough often cause you to be short of breath? B) Do you experience chest pain when you cough? C) How long have you been experiencing your cough? D) Are you now or have you ever been a smoker?

C) How long have you been experiencing your cough?

9. When assessing the vaginal orifice of a young female client who has never been sexually active, the nurse notes a fold of fibrous tissue at the introitus. The nurse recognizes this as which structure? A) Labia B) Urethra C) Hymen D) Clitoris

C) Hymen

17. A nurse is testing the range of motion of the thoracic and lumbar spine. Which of the following would the nurse document as an abnormal finding? A) Flexion of 80 degrees B) Lateral bending of 35 degrees C) Hyperextension of 15 degrees D) Rotation of 30 degrees

C) Hyperextension of 15 degrees

8. During the health history, the nurse teaches a client about toxic shock syndrome and ways to reduce her risks. The nurse determines that the teaching was successful when the client states which of the following? A) I will get a Pap smear regularly. B) It is important to use latex condoms. C) I should change tampons at least every 4 to 6 hours. D) I should stop using oral contraceptives.

C) I should change tampons at least every 4 to 6 hours.

21. A hospital's protocols for assessment have been modified in light of standards established by the Joint Commission. What change would bring practice into alignment with these standards? A) Teaching all new clients about the basic pathophysiology of pain B) Assessing clients' pain objectively rather than subjectively C) Identifying pain as the fifth vital sign and assessing clients accordingly D) Triaging clients according to the type of pain that they are experiencing

C) Identifying pain as the fifth vital sign and assessing clients accordingly

18. The nurse is assessing a female client's genitourinary system. Which of the following findings would lead the nurse to suspect a problem with the ovaries during palpation? A) Slight tenderness on palpation B) Walnut-sized ovaries C) Immobile ovaries D) Smooth ovarian surface

C) Immobile ovaries

2. In the course of the nurse's health interview, a client reports an occasional blockage in the upper portion of his nasal passage. What is the most pronounced effect that this will have on the client? A) Decreased sense of taste B) Difficulty hearing C) Impaired sense of smell D) Occasional dizziness

C) Impaired sense of smell

28. There has been some resistance to the planned transition to electronic health records (EHRs) in a hospital system, with many caregivers questioning the rationale for this change in practice. What potential advantage of EHRs should administrators cite? A) Increased influence for the nursing profession B) Elimination of documentation C) Improved continuity of care D) Reduced nursing workload

C) Improved continuity of care

17. An experienced nurse is teaching a recently graduated colleague about common pitfalls encountered in the diagnostic reasoning process. The experienced nurse should identify a need for further teaching if the new graduate identifies which of the following as a pitfall? A) View of things as either right or wrong B) Overemphasis on details C) Inclusion of valid data D) Clustering of unrelated cues

C) Inclusion of valid data

6. The pregnant client tells the nurse she has a history of mitral valve stenosis as a sequela of rheumatic fever. The nurse plans to closely monitor the client based on the understanding that which physiologic change in pregnancy increases this client's risk for complications? A) Physiologic anemia B) Altered carbohydrate metabolism C) Increased blood volume D) Hormonal changes

C) Increased blood volume

12. A client rates his pain as 9 on a scale of 1 to 10. The nurse would expect to assess which of the following? A) Constricted pupils B) Hypotension C) Increased serum glucose D) Flaccid muscles

C) Increased serum glucose

28. A nurse is caring for a patient whose diagnosis of cystic fibrosis results in the production of large amounts of sticky mucus. The client has a history of repeated hospital admissions for complications of his disease and receives daily treatments to mobilize the secretions. When planning the care of this client, what nursing diagnosis is most plausible? A) Readiness for Enhanced Breathing Patterns B) Risk for Impaired Oral Mucous Membranes related to mouth breathing C) Ineffective Airway Clearance related to respiratory secretions D) Ineffective Breathing Pattern: Hyperventilation related to cystic fibrosis

C) Ineffective Airway Clearance related to respiratory secretions

10. While assessing a client's abdomen, the nurse observes involuntary reflex guarding on expiration. The nurse would interpret this as most likely indicating which of the following? A) Hernia B) Malignancy C) Infection D) Aneurysm

C) Infection

25. The nurse inspects a 10-day-old infant's umbilicus and notes that it is reddened with the presence of slight discharge. What nursing diagnosis is suggested by these data? A) Risk for contamination B) Ineffective peripheral tissue perfusion C) Infection D) Risk for injury

C) Infection

9. Which of the following would be most important for the nurse to do when assessing a client's blood pressure? A) Palpate the pulsations of the ulnar artery. B) Hold the client's arm slightly flexed with palm down. C) Inflate the cuff 30 mm Hg above where the radial pulse disappears. D) Deflate the cuff about 5 mm Hg per second.

C) Inflate the cuff 30 mm Hg above where the radial pulse disappears.

3. A client's electronic health record states that he has been diagnosed with sensorineural hearing loss. Which condition should the nurse most likely identify as a cause? A) Perforated eardrum B) Otosclerosis C) Inner ear problem D) Otitis media

C) Inner ear problem

5. A client presents to an ambulatory clinic with purulent, bloody drainage of the ear. Which of the following should the nurse assess first? A) Assess the client's tympanic membrane. B) Palpate the client's tragus. C) Inspect the client's external ear canal. D) Perform hearing assessments.

C) Inspect the client's external ear canal.

3. A group of students is preparing for their clinical experience, during which they are required to demonstrate the techniques for assessing the abdomen. The students demonstrate understanding of the proper sequence when they demonstrate the techniques in which order? A) Palpate, percuss, inspect, auscultate B) Auscultate, inspect, palpate, percuss C) Inspect, auscultate, percuss, palpate D) Percuss, inspect, auscultate, palpate

C) Inspect, auscultate, percuss, palpate

13. A nurse is interviewing a client who is a suspected victim of abuse. Which of the following practices should the nurse avoid during this phase of assessment? A) Using direct questions about being injured B) Displaying a concerned, empathetic approach C) Interjecting often to clarify information D) Emphasizing the nurse's availability to talk

C) Interjecting often to clarify information

23. An audit of a hospital unit's incident reports reveals that several errors have resulted from incomplete or inaccurate information during change-of-shift handoff. In order to prevent such errors, what practice should be encouraged on the unit? A) Delegate handoff reports to unlicensed care providers who have fewer demands on their time. B) Use an intermediary to receive report from the first nurse and then provide the handoff report to the second nurse. C) Involve as few people as possible in the verbal report. D) Encourage nurses to perform handoff as quickly as possible.

C) Involve as few people as possible in the verbal report.

21. The nurse is interviewing an 82-year-old client who is accompanied by her daughter. The daughter states that her mother is unable to hold her urine, and the client attests that this is true. What question should the nurse prioritize when assessing the client's urinary incontinence? A) Did you deliver your children vaginally or by cesarean section? B) Have you been prone to urinary tract infections in the past? C) Is this something that has begun to happen just recently? D) Have you noticed any change in your bowel function?

C) Is this something that has begun to happen just recently?

10. While interviewing a hospitalized client, he states, "The holy days of Ramadan are coming soon. I am not to have any food or drink from sunrise to sunset during this time." Further assessment reveals that the client's request is associated with which religion? A) Christianity B) Judaism C) Islam D) Hinduism

C) Islam

14. A client has questioned why the nurse asked him how his family members usually treat their pain. Which of the following would be the most appropriate response by the nurse? A) It is just a way for me to more fully understand you and your upbringing. B) It helps me to direct interventions toward your cultural history. C) It helps me to determine how the family understands and perceives pain. D) It will allow me to see if you are more likely to react to pain in a negative manner.

C) It helps me to determine how the family understands and perceives pain.

3. A nurse is assessing the pain of a client who has had major surgery. The client also has been experiencing depression. Which of the following principles should guide the nurse's assessment of a client's pain? A) The client is likely experiencing less pain than he is reporting. B) The client's depression exists independently of the level of pain. C) It is likely that the client's pain rating will be influences by his emotional state. D) The degree of surgery will be the key indicator for level of pain experienced.

C) It is likely that the client's pain rating will be influences by his emotional state.

2. A new nursing graduate recently made an oversight during the analysis of a client's assessment data that resulted in a postoperative complication. What characteristic of data analysis makes it a challenging aspect of nursing practice? A) Abnormal data must be identified. B) It requires the prior identification of nursing diagnoses. C) It requires sophisticated diagnostic reasoning skills. D) Conclusions must be clearly and accurately documented.

C) It requires sophisticated diagnostic reasoning skills.

18. A nurse is reviewing a depression questionnaire completed by a client. Which of the following would the nurse interpret as being suggestive of depression? A) Occasionally I feel like my attention wanders. B) I haven't noticed any change in my appetite. C) It usually takes me over an hour to fall asleep. D) I might wake up once during the night but not often.

C) It usually takes me over an hour to fall asleep.

29. The nurse is assessing the means of communication that exist in an urban community. What barrier to communication is the nurse most likely to identify? A) Lack of access to written communication B) Illiteracy C) Language barriers D) Lack of technological literacy

C) Language barriers

3. The nurse completes the initial newborn assessment and notes the presence of fine, downy hair on the infant's shoulders and back. The nurse documents the presence of which of the following? A) Vernix B) Milia C) Lanugo D) Nevi

C) Lanugo

13. A nurse is attempting to auscultate fetal heart tones after determining that the fetus is in a longitudinal lie, cephalic presentation, and left occiput anterior position. The nurse would auscultate them at which area? A) Left upper quadrant B) Right upper quadrant C) Left lower quadrant D) Right lower quadrant

C) Left lower quadrant

1. A client has suffered a suspected a rotator cuff tear. Which of the following would the nurse expect to find? A) Limitation of all shoulder motion B) Chronic pain C) Limited abduction D) Sharp catches of pain with movement

C) Limited abduction

19. A victim of intimate partner violence tells a nurse, I don't know how I'd live if I left my husband. And what about my children? I have no skills and haven't worked since I was a teenager. When developing the plan of care for this client, which nursing diagnosis would most likely apply? A) Anxiety related to the physical escalation of the violence B) Impaired parenting related to family violence C) Low self-esteem related to lack of confidence in ability D) Risk for violence related to inadequate coping abilities

C) Low self-esteem related to lack of confidence in ability

19. A nurse has identified a goal of developing his critical thinking skills. In order to facilitate this goal, what action should the nurse prioritize? A) Applying quick decision-making B) Seeking new experiences C) Maintaining an open mind D) Maintaining a stable and static knowledge base

C) Maintaining an open mind

6. While assessing a woman's breasts, the nurse notes a pronounced and asymmetric pattern of veins on the client's breasts. Follow-up care is ordered because the nurse should suspect which of the following? A) Pregnancy B) Fibrocystic changes C) Malignancy D) A low platelet count

C) Malignancy

19. The nurse is assessing the face of a client with a diagnosis of Parkinson's disease. Which of the following would the nurse most likely assess? A) Sunken face B) Drooping of one side C) Masklike expression D) Asymmetry of earlobes

C) Masklike expression

20. The nurse needs to obtain the height of a client who is unable to stand. Which of the following would the nurse do? A) Estimate the height while the client is lying in bed. B) Measure the distance from the top of the client's head to his ankles. C) Measure from client's arm span using one of his arms outstretched. D) Extend a ruler from the forehead to the tip of the client's toes.

C) Measure from client's arm span using one of his arms outstretched.

12. Which of the following would be most appropriate when the nurse notes limitation in active range of motion of a client's right shoulder? A) Test muscle strength. B) Perform passive range of motion test. C) Measure range of motion with a goniometer. D) Ask the client which is the dominant side.

C) Measure range of motion with a goniometer.

10. A nurse is assessing a client's skeletal muscle mass in the context of a comprehensive nutritional assessment. Which measurement would yield the most valid and reliable data? A) Body mass index B) Triceps skin fold measurement C) Mid-arm circumference D) Waist circumference

C) Mid-arm circumference

19. A gerontologic nurse is assessing the speech of an older adult client. Which of the following would the nurse characterize as an expected assessment finding? A) Repetition B) Rapid speech C) Moderate pace D) Loud tone

C) Moderate pace

20. A nurse teaches a male client how to perform testicular self-examination when the client's history reveals that he does not do it. The nurse should instruct the client to perform the self-examination at which frequency? A) Weekly B) Bimonthly C) Monthly D) Quarterly

C) Monthly

23. The nurse is assessing a newborn infant who currently has nasal congestion and rhinorrhea (runny nose). When analyzing these data, the nurse should consider which of the following? A) Nasal congestion in an infant is indicative of infection. B) Nasal mucus in infants should be treated with an inhaled vasoconstrictor. C) Nasal congestion can impair oxygenation because infants are nose breathers. D) Nasal congestion in infants is an expected finding for the first 6 weeks of life.

C) Nasal congestion can impair oxygenation because infants are nose breathers.

16. The nurse is palpating the client's tonsillar, submandibular, and submental lymph nodes. The nurse is most likely examining which area during a comprehensive assessment? A) Nose and sinuses B) Abdomen C) Neck D) Face

C) Neck

17. A nurse is creating a concept map of the pathophysiology of pain. The nurse should identify which of the following as being responsible for transmitting pain sensations to the central nervous system? A) Transduction B) Modulation C) Nociceptors D) Cytokines

C) Nociceptors

5. During a health history, a 62-year-old male client reveals that he occasionally sees spots before his eyes. The nurse interprets this finding as the result of which of the following? A) Increased ocular pressure B) Vitamin A deficiency C) Normal findings for client's age D) Vascular spasm

C) Normal findings for client's age

17. While inspecting the client's tympanic membrane, the nurse notes a pearly gray and shiny appearance. The nurse would interpret this finding as which of the following? A) Scarring from previous infections B) Otitis media C) Normal tympanic membrane D) Otitis externa

C) Normal tympanic membrane

18. An older adult client has been admitted to the medical unit after suffering an exacerbation of chronic obstructive pulmonary disease (COPD). Which of the following should the nurse do to assess the depth of the client's respirations? A) Count the respirations for 30 seconds and multiply by 2. B) Place the client's arm across the chest while palpating the pulse. C) Observe the client's chest expansion bilaterally. D) Percuss the client's posterior thorax

C) Observe the client's chest expansion bilaterally.

10. When palpating the Bartholin's glands, the nurse expresses a purulent discharge. Which of the following would be most appropriate for the nurse to do next? A) Recommend sitz baths. B) Palpate the uterus. C) Obtain a culture. D) Perform a rectal exam.

C) Obtain a culture.

17. When assessing the cervix of an older postmenopausal woman, which of the following would the nurse document as a normal finding? A) Bluish color B) Bright red C) Pale pink D) White patches

C) Pale pink

8. Which of the following tests would be most appropriate for the nurse to use when assessing motor function of a client's trigeminal nerve? A) Ask client to differentiate sharp and dull sensations on the face. B) Have the client smile, frown, and wrinkle the forehead. C) Palpate temporal and masseter muscles while client clenches the teeth. D) Assess dilatation of the client's pupils with direct light.

C) Palpate temporal and masseter muscles while client clenches the teeth.

11. A nurse is unable to palpate a client's radial and ulnar pulses. What is the nurse's most appropriate action? A) Refer the client for medical follow-up. B) Document the finding and proceed with the assessment. C) Palpate the brachial pulse. D) Auscultate the apical pulse.

C) Palpate the brachial pulse.

11. During the health interview of a new client, the nurse has explored the client's decision- making strategies. These data are most essential to the developmental theory of which theorist? A) Freud B) Kohlberg C) Piaget D) Erikson

C) Piaget

6. A client is diagnosed with pulmonary edema, and the nurse is performing a rapid assessment prior to treatment. The nurse would be most concerned about which of the following assessment findings related to the client's sputum? A) White or cream-colored B) Yellowish and foul-smelling C) Pink and frothy D) Rust-tinged

C) Pink and frothy

4. To promote relaxation of the client's abdominal muscles, which of the following would be most appropriate for the nurse to do? A) Encourage the client to hold his or her breath. B) Cover the client in a warm blanket. C) Place a pillow under both of the client's knees. D) Assure the client that painful areas will not be examined.

C) Place a pillow under both of the client's knees.

6. A nurse is preparing a health education class for a group of older adult clients at a local senior center. The nurse is focusing on health promotion and disease prevention. Which condition would the nurse cite as a common cause of infection-related deaths in the elderly? A) Pyelonephritis B) Cellulitis C) Pneumonia D) Meningitis

C) Pneumonia

6. The nurse is caring for a client with influenza symptoms and is documenting the initial and ongoing assessment database. Which of the following would the nurse emphasize as the major rationale for this action? A) Reducing the fragmentation of care B) Maximizing the efficiency of care C) Promoting communication between disciplines D) Facilitating achievement of professional standards

C) Promoting communication between disciplines

4. An adult male client reports hesitancy when urinating. The nurse would further assess this client for which of the following? A) Scrotal hernia B) Sexually transmitted infection C) Prostate enlargement D) Testicular tumor

C) Prostate enlargement

24. A nurse is teaching a recent nursing graduate about the significance of verbal and nonverbal communication during client care. The new graduate demonstrates an understanding of these techniques by citing what example of verbal communication? A) Maintaining an open attitude B) Using silence appropriately C) Providing a laundry list of descriptors when needed D) Maintaining an open and encouraging facial expression

C) Providing a laundry list of descriptors when needed

9. The nurse is conducting a health interview and is addressing the client's current stressors. What is the primary rationale for including stress as a focus of psychosocial assessment? A) Stress provides the main impetus for psychosocial development and adaptation. B) Psychosocial development cannot progress normally in the presence of stress. C) Psychosocial stress has a major influence on health in many domains. D) The results of the health interview are distorted when the client is experiencing stress.

C) Psychosocial stress has a major influence on health in many domains.

25. The nurse is using her fingerpads to palpate a client's body part during the physical examination. Which of the following would the nurse best be able to detect? A) Temperature B) Vibrations C) Pulses D) Fremitus

C) Pulses

19. A nurse has completed an assessment of a school-age child. The nurse has identified several soft signs of potential neurologic impairment. How should the nurse best interpret these findings? A) Recognize that the findings are related to developmental tasks rather than neurologic pathology B) Recognize the need for an emergency neurological assessment C) Recognize that the findings may or may not indicate the presence of a neurologic problem D) Recognize that the findings need to be interpreting in light of the child's education level

C) Recognize that the findings may or may not indicate the presence of a neurologic problem

10. The children of an elderly client tell the nurse, He has lost his appetite. He eats very small amounts, and only twice a day. Which suggestion would be most appropriate? A) Inform them that he will eat when he is hungry. B) Counsel them to weigh him daily. C) Recommend nutrient-dense foods. D) Advise them to restrict fluid intake.

C) Recommend nutrient-dense foods.

15. Assessment findings reveal that a client has herpes progenitalis. Which of the following would be most important to include in the teaching related to after the initial lesions disappear? A) The disease will spontaneously regress. B) The client is at increased risk for cancer of the glans. C) Recurrence can happen with varying frequency. D) The next outbreak will include moist, fleshy papules.

C) Recurrence can happen with varying frequency.

25. The nurse is completing the assessment of a client who is 26 weeks pregnant. Assessment reveals a fundal height of 21 cm. How should the nurse follow up this assessment finding? A) Have the client reassessed for gestational diabetes. B) Obtain a 24-hour food recall. C) Refer the client due to possible intrauterine growth retardation. D) Order a repeat ultrasound due to possible multiple gestation.

C) Refer the client due to possible intrauterine growth retardation.

4. A client complains of headaches each morning that resolve after getting out of bed. Which of the following would be most appropriate for the nurse to do? A) Assess the client's level of consciousness. B) Assess the client's deep tendon reflexes. C) Refer the client for immediate medical follow-up. D) Refer the client for physical therapy and occupational therapy.

C) Refer the client for immediate medical follow-up.

23. The nurse is obtaining the health history of a young adult client. During the interview, the client tells the nurse, I banged my head pretty good when I was snowboarding last weekend. The client states that he did not subsequently seek care. What is the nurse's most appropriate action? A) Promptly assess the client's balance and coordination. B) Teach the client about the warning signs of increased intracranial pressure. C) Refer the client for medical assessment and possible treatment. D) Teach the client about the importance of wearing head protection during sports.

C) Refer the client for medical assessment and possible treatment.

25. Assessment of a client's mouth reveals a lesion on the client's buccal membrane that is approximately 0.5 cm in diameter. On further questioning, the client states that the lesion has been present for 3 months and that it bleeds intermittently. How should the nurse follow up this assessment finding? A) Swab the lesion to obtain a sample for culture and sensitivity testing. B) Recommend that the client gargle with saltwater twice daily for several days. C) Refer the client to her primary care provider promptly. D) Determine whether the lesion can be removed with a sterile cotton-tipped applicator.

C) Refer the client to her primary care provider promptly.

30. A middle-aged client with a periorbital hematoma admits to the nurse that the injury occurred when her husband punched her in a rage. Following assessment and treatment, the client expresses her intention to return home to her husband. How should the nurse best respond? A) Remind the client that she waives her right to press charges if she willingly returns home. B) Inform the client that she will not be permitted to return to a known unsafe setting. C) Remind the client of the threat to her safety and of her options for shelter. D) Inform the client that a social worker must grant permission for her to return home.

C) Remind the client of the threat to her safety and of her options for shelter.

1. The nurse is interviewing a female Hispanic client who is scheduled for a cardiovascular education program. The client states, I can't eat and I don't sleep because my daughter left to return to Mexico. I am sad and nervous. I need rest. The nurse suspects that she is suffering from susto. Which action by the nurse would be best? A) Give her a multivitamin supplement. B) Encourage her to exercise. C) Reschedule the education program. D) Refer her to a counselor.

C) Reschedule the education program.

15. The nurse is evaluating a new nursing graduate's ability to perform a rebound tenderness test for suspected appendicitis. The nurse identifies correct technique when the new graduate is observed pressing deeply at which abdominal location? A) Right upper quadrant B) Left upper quadrant C) Right lower quadrant D) Left lower quadrant

C) Right lower quadrant

20. While assessing an infant's abdomen, which finding would the nurse interpret as necessitating immediate evaluation and treatment? A) Palpable mass B) Tenderness C) Rigidity D) Gurgling sounds

C) Rigidity

28. The nurse is caring for a client who has been experiencing dysphagia secondary to a stroke. What risk nursing diagnosis should the nurse associate with this health problem? A) Risk for injury related to potential esophageal trauma B) Risk for oral infection related to dysphagia C) Risk for aspiration related to decreased swallowing ability D) Risk for excess fluid volume related to decreased peristalsis

C) Risk for aspiration related to decreased swallowing ability

11. A nurse determines that a community lacks adequate recreational activities and facilities. Which nursing diagnosis would the nurse most likely identify? A) Ineffective role performance B) Risk for other-directed violence C) Risk for social isolation D) Ineffective community coping

C) Risk for social isolation

11. A nurse's data analysis has led to the formulation of a risk nursing diagnosis. Which of the following best demonstrates accurate documentation of a risk nursing diagnosis? A) Risk for fatigue related to increased job demands, as manifested by feelings of exhaustion and frequent naps B) Risk for infection, as manifested by lack of client knowledge of wound care C) Risk for violence related to history of overt, aggressive acts D) Risk for altered respiratory function related to environmental allergens, as manifested by asthma

C) Risk for violence related to history of overt, aggressive acts

4. A nurse is reviewing the electrical conduction system of the heart in preparation for assessing a client with a conduction problem. The nurse should be aware that the electrical signal originates in which of the following locations? A) Bundle of His B) Purkinje fibers C) Sinoatrial node D) AV node

C) Sinoatrial node

5. A nurse has gathered the necessary equipment for the physical assessment of an adult client. For which of the following assessments would it be most appropriate for a nurse to use a centimeter-scale ruler for measurement? A) Mid-arm circumference B) Client's height C) Skin lesion size D) Pupillary size

C) Skin lesion size

6. Which factor, if present in a client's lifestyle and health practices assessment, would alert the nurse to the need for performing a more thorough head and neck assessment? A) Alcohol abuse B) Recreational drug use C) Smokeless tobacco use D) Multiple sex partners

C) Smokeless tobacco use

12. An adult client has been diagnosed with bronchitis. Which of the following would the nurse most likely hear on auscultation? A) Sibilant wheezes B) Fine crackles C) Sonorous wheezes D) Coarse crackles

C) Sonorous wheezes

24. The intensive care nurse is working with a client who has increased intracranial pressure secondary to a traumatic brain injury. The nurse is performing the hourly assessment of the client's level of consciousness and observes that the client's eyes are closed. How should the nurse first stimulate the client to assess for arousability? A) Gently shake the client's right shoulder and then his left shoulder. B) Rub the client's sternum with the knuckles. C) Speak to the client clearly from a close distance. D) Press down on one of the client's nail beds.

C) Speak to the client clearly from a close distance.

7. The nurse is beginning the physical exam of a male client's genitals. The nurse is sitting on a stool in front of the client. In which position would be best to place the client? A) Lying supine B) Kneeling C) Standing D) Sitting

C) Standing

14. The nurse has collected objective and subjective data during the assessment of a client who has been admitted for the treatment of an exacerbation of chronic obstructive pulmonary disease (COPD). During the current phase of the diagnostic reasoning process, the nurse is writing down thoughts about each cue cluster of data that was collected. The nurse is involved in which step of the diagnostic reasoning process? A) Step One: Identify Abnormal Data and Strengths B) Step Two: Cluster Data C) Step Three: Draw Inferences D) Step Four: Propose Possible Nursing Diagnoses Page 3

C) Step Three: Draw Inferences

2. When preparing to assess a client's thoracic cage, the nurse should locate which landmark when determining where to begin the assessment of the ribs and intercostal spaces? A) Scapula B) Suprasternal notch C) Sternal angle D) Sternal border

C) Sternal angle

2. Which of the following would be the best way for the nurse to implement the process of participant observation when conducting a community assessment? A) Interview the key leaders of the community. B) Review the census data and health records. C) Take part in the daily life of the community. D) Conduct a detailed survey of the community's health problems.

C) Take part in the daily life of the community.

6. The nurse is assessing the economic stability of a community. The nurse should first address which of the following? A) Mortality statistics B) Hospital facilities C) Tax base D) Banks

C) Tax base

18. A nurse has completed an assessment of a client's lymph nodes. Which of the following data would the nurse document as an abnormal finding? A) Diameter: 0.75 cm B) Mobile C) Tender D) Discrete

C) Tender

1. When assessing the client's ear, which finding should the nurse identify as indicating a need for further assessment and possible treatment? A) Darwin tubercle B) Red, flaky cerumen C) Tender tragus D) Pearly gray tympanic membrane

C) Tender tragus

29. The nurse is performing blunt percussion of a client's kidneys. For what abnormal finding is the nurse primarily assessing? A) Dullness B) Tympany C) Tenderness D) Hyperresonance

C) Tenderness

22. A nurse's colleague states, "I think Mrs. Nguyen in room 412 is a Buddhist, so she'll definitely be a vegetarian." The nurse should understand what principle of religion and spirituality when planning clients' care? A) Decisions around a religious client's care should be deferred to the clergy of that religion. B) Clients who claim to be spiritual generally oppose meat consumption. C) The beliefs of members of a particular religion are not necessarily homogeneous. D) Nurses should avoid planning care on the basis of religion.

C) The beliefs of members of a particular religion are not necessarily homogeneous.

25. A hospital nurse is admitting a client with a documented history of acute pancreatitis, liver cirrhosis, malnutrition, and frequent traumatic injuries. What assessment finding would most clearly warrant validation? A) The client's blood pressure is 148/88 mm Hg. B) The client is oriented to person and place but not to time. C) The client states that she only drinks alcohol on a social basis. D) The client states, My skin's kind of yellow because of my liver.

C) The client states that she only drinks alcohol on a social basis.

29. A nurse who provides care in a hospital setting is creating a plan of nursing care for a client who has a diagnosis of chronic renal failure. The nurse's plan specifies frequent ongoing assessments. The frequency of these nursing assessments should be primarily determined by what variable? A) The client's age B) The unit's protocols C) The client's acuity D) The nurse's potential for liability

C) The client's acuity

21. When applying Kohlberg's theory of moral development to the status of an older adult client, on what assessment finding would the nurse focus? A) The relationship between the client's stated beliefs and his actions B) The client's ability to discern the motivations of others C) The client's adherence to rules, laws, and norms D) The client's ability to tolerate differing views

C) The client's adherence to rules, laws, and norms

6. A nurse obtains the blood pressure of a client who is uncharacteristically fatigued and who is lying in bed rather than sitting in a chair. The nurse should interpret the client's blood pressure reading in light of what principle? A) The client's blood pressure will be slightly highly than the client's norm. B) Position rarely affects the client's blood pressure. C) The client's blood pressure will be slightly lower than standing readings. D) There will be questionable accuracy of the blood pressure reading.

C) The client's blood pressure will be slightly lower than standing readings.

22. The nurse is preparing to gather equipment prior to a client's head-to-toe assessment. The nurse's selection of equipment should be based primarily on what variable? A) The nurse's time allowance B) The nurse's level of expertise C) The client's health needs D) The client's level of participation

C) The client's health needs

22. An 84-year-old man has been admitted to the emergency department from an extended care facility. Facility staff suspect that the client has pneumonia, and his malaise, productive cough, shortness of breath, and adventitious breath sounds are consistent with this diagnosis. However, the nurse's assessment of the client's vital signs yields an oral temperature of 97.5∞F. How should the nurse best interpret this assessment finding? A) The client likely has a cardiac health problem, not a respiratory health problem. B) The client's signs and symptoms are related to hypothermia rather than infection. C) The client's normothermic temperature does not rule out the presence of an infection. D) The client's infection is no longer localized and has become systemic.

C) The client's normothermic temperature does not rule out the presence of an infection.

6. A nurse is admitting a client to the postsurgical unit following breast reconstruction surgery. Which of the following would the nurse use as the primary assessment for the client's pain? A) The client's spiritual view of the pain B) Current pain therapies used preoperatively C) The client's report of her pain D) Psychosocial questions related to her perceptions of pain

C) The client's report of her pain

10. The nurse is conducting a health interview and has asked the client, How would you describe yourself to others? The client's response informs the nurse's assessment of which of the following? A) The client's morality and honesty B) The client's aspirations C) The client's self-concept D) The client's superego

C) The client's self-concept

30. The nurse is assessing a client's spiritual history using the SPIRIT acronym. The nurse should begin the assessment by identifying what aspect of spirituality? A) The client's religious affiliation B) The client's state of health C) The client's sources of hope D) The client's spiritual belief system

C) The client's sources of hope

14. A review of a client's history reveals cranial nerve IV paralysis. Which of the following findings would the nurse expect to assess? A) The eye cannot look to the outside side. B) Ptosis will be evident. C) The eye cannot look down when turned inward. D) The eye will look straight ahead.

C) The eye cannot look down when turned inward.

27. The nurse is performing Ortolani's maneuver to test for congenital hip dysplasia in a newborn infant. What finding would suggest the presence of hip dysplasia? A) The infant expresses no signs of pain or discomfort during manipulation of the hip. B) The nurse is unable to perform passive range of motion of the infant's hip joint. C) The nurse hears a click from the site of the infant's hip joint. D) The nurse is unable to bring the infant's knees into alignment.

C) The nurse hears a click from the site of the infant's hip joint.

23. A woman has accompanied her 80-year-old husband to a scheduled clinic visit and expresses concern about subtle declines in his cognition. Which of the following principles should guide the nurse's assessment of the client's mental status? A) The nurse must modify the cognitive assessment to exclude assessments requiring reading or writing. B) The nurse should first explain to the couple that senility is expected among adults over age 80. C) The nurse must differentiate between age-related changes and the signs and symptoms of dementia. D) The nurse must explain that the results of the assessment will be used to determine if admission to long-term care is necessary.

C) The nurse must differentiate between age-related changes and the signs and symptoms of dementia.

27. The nurse is assessing an adult client's areolas and nipples. What assessment finding would most clearly warrant referral? A) Small Montgomery tubercles are present on the areolas. B) Supernumerary nipples are present. C) The patient's nipple has recently become inverted. D) The patient's areola puckers upon palpation.

C) The patient's nipple has recently become inverted.

23. A nurse recognizes the need to perform a spiritual assessment of a newly admitted hospital client, but the circumstances surrounding the client's diagnosis and family dynamics make this challenging. What variable is likely to have the greatest impact on enhancing the quality of data from the nurse's spiritual assessment? A) The nature of the nurse's spiritual beliefs B) The nurse's knowledge of major religions C) The quality of rapport between the nurse and the client D) The setting in which the assessment is performed

C) The quality of rapport between the nurse and the client

28. The nurse has observed that a client adheres rigidly to the norms of her family and her culture. In the context of Freud's theory of development, this pattern of behavior is attributable to the action of what component of personality? A) The id B) The ego C) The superego D) The identity

C) The superego

9. After teaching a group of students about systems theory and family, the instructor determines that the teaching was successful when the students state which of the following as a major principle of the systems theory? A) Parts are independent of one another. B) Each system has the same characteristics. C) The whole is greater than the sum of the parts. D) Information is isolated within each part the system.

C) The whole is greater than the sum of the parts.

4. The nurse is providing health education to an elderly client with dysphagia following a recent ischemic stroke. Which of the following would be most appropriate for the nurse to include? A) Sit with the head of the bed at 45 degrees during meals. B) Be aware of the possibility of temporomandibular joint pain. C) Thoroughly chew small amounts of food with each mouthful. D) Drink fluids before and after, but not during, meals.

C) Thoroughly chew small amounts of food with each mouthful.

8. A client at 32 weeks' gestation has been placed on complete bed rest due to premature labor contractions. The nurse should prioritize assessments for which of the following complications? A) Hyperglycemia B) Urinary tract infection C) Thrombophlebitis D) Leg cramps

C) Thrombophlebitis

23. The nurse is gathering the necessary equipment preparatory to examining a client's ears. The nurse will be checking bone and air conduction of sound. Which of the following should the nurse obtain? A) Penlight B) Tongue depressor C) Tuning fork D) Otoscope

C) Tuning fork

25. The nurse has completed the assessment of a family. What phenomenon would the nurse identify as a triangle? A) The youngest child in the family has been estranged from his siblings for many years. B) The father was married and divorced as a young adult, before remarrying. C) Two sisters have been in conflict and each attempts to elicit support from their mother. D) There is a total of three children in the family.

C) Two sisters have been in conflict and each attempts to elicit support from their mother.

28. The nurse is percussing a client's abdomen. What predominant sound should the nurse expect to hear over the majority of the abdomen? A) Accentuated tympany B) Hyperresonance C) Tympany D) Dullness

C) Tympany

15. A group of nursing students is reviewing information about the lymph nodes of the lower extremity and the areas drained by them. The students demonstrate the need for additional teaching when they identify which area as being drained by the superficial inguinal nodes? A) Legs B) External genitalia C) Upper abdomen D) Buttocks

C) Upper abdomen

1. The nurse is assessing a client's breasts. When assessing the area of the breast most vulnerable to breast cancer, where should the nurse to assess? A) Upper inner quadrant B) Lower inner quadrant C) Upper outer quadrant D) Lower outer quadrant

C) Upper outer quadrant

9. The nurse is unable to palpate the dorsalis pedis pulse on an older adult client. Which of the following would be most appropriate for the nurse to do next? A) Document absence of dorsalis pedis pulse. B) Auscultate the anatomic area with a stethoscope. C) Use Doppler ultrasonography to locate the pulse. D) Apply a tourniquet for 2 minutes and then reassess.

C) Use Doppler ultrasonography to locate the pulse.

16. After assessing a community, the nurse plans programs to address the community's government. Which of the following aspects of the meeting would demonstrate that the government is accessible and responsible? A) Encouraging the view of the most well-respected residents B) Limiting the number of differing views C) Using open-forum community meetings D) Maintaining the status quo

C) Using open-forum community meetings

1. A client at 22 weeks' gestation comes to the clinic complaining of earache and decreased hearing. Otoscopic examination of the ear is normal. The nurse explains to the client that her symptoms are pregnancy-induced as a result of what physiologic change? A) Thickened, dry cerumen B) Infection of the inner ear C) Vascularity of the eardrum D) Auditory nerve compression

C) Vascularity of the eardrum

3. The nurse is completing the general survey. In addition to observing the client's appearance, the nurse would assess which of the following? A) Mental status B) Cognitive abilities C) Vital signs D) Thought processes

C) Vital signs

7. A client who works in a manufacturing plant is attending a teaching session on plant safety. Which of the following would be an important risk prevention measure to teach regarding hearing? A) Limiting loud noise exposure to 1 hour per day B) Taking a 10-minute break every 2 hours C) Wearing ear protection when in the work environment D) Cleaning ears regularly to prevent ear infections

C) Wearing ear protection when in the work environment

3. A nurse is admitting a new client to the subacute medical unit and is completing a comprehensive assessment. The nurse is appropriately applying standard precautions by performing which of the following actions? A) Performing hand hygiene between examinations of each body part B) Discarding in the trash can the safety pin that was used to assess sensory perception C) Wearing gloves to palpate the tongue and buccal membranes D) Wearing a gown, gloves, and mask during the physical exam

C) Wearing gloves to palpate the tongue and buccal membranes

19. When obtaining the nutritional health history from a female client, which of the nurse's questions would best elicit information about the client's knowledge of her own health status? A) Are you now or have you been on a diet recently? B) How much fluid do you drink in a day? C) What are your height and usual weight? D) Can you tell me what you consider to be a healthy meal?

C) What are your height and usual weight?

18. When assessing an older adult about possible mistreatment, which of the following questions would be most appropriate to use initially? A) Have you ever been abused? B) Did you ever sign papers you didn't understand? C) What is a typical day in your life like? D) Are you alone often at home?

C) What is a typical day in your life like?

3. A client who had a mastectomy is being discharged home on postoperative day 1. Knowing that the client lives alone, which data would be most important for the nurse to validate for this client? A) If the client has transportation for follow-up appointments B) If the client usually functions independently C) What support systems are in place to assist the client D) If the client has a religious belief regarding illness

C) What support systems are in place to assist the client

13. The nurse is teaching a group of parents of children of various ages how to best measure a child's temperature. The nurse instructs the parents that rectal temperature measurement is indicated in which situation? A) During the newborn period B) When a child is dehydrated C) When no other route is feasible D) When rapid temperature changes occur

C) When no other route is feasible

23. A nurse is implementing appropriate infection control precautions while performing a client's skin assessment. During which of the following components of the assessment should the nurse wear gloves? A) When palpating the texture of the client's skin B) When palpating the client's hair C) When palpating lesions on the client's skin D) When palpating the client's nail beds for texture and capillary refill

C) When palpating lesions on the client's skin

18. An instructor is teaching a student about the proper use of a stethoscope. The instructor determines the need for additional teaching when the student states which of the following? A) Plastic tubing should be longer than 3 feet. B) The bell is used after using the diaphragm. C) When using the bell, push on it lightly. D) A diaphragm picks up low-pitched sounds.

C) When using the bell, push on it lightly.

22. The nurse has been applying the nursing process in the care of an adult client who is being treated for acute pancreatitis. Place the nurse's actions in their proper sequence from first to last. A) Identifying outcomes B) Determining client's nursing problem C) Collecting information about the client D) Determining outcome achievement E) Carrying out interventions

C, B, A, E, D C) Collecting information about the client B) Determining client's nursing problem A) Identifying outcomes E) Carrying out interventions D) Determining outcome achievement

25. The admission of a new resident to a long-term care facility has necessitated a thorough health history. Place the following focuses in the correct sequence in which the nurse should perform them, beginning with the section obtained first. A) Family health history B) Reason for seeking care C) Biographic data D) Review of body systems E) History of present concern F) Past health history

C, B, E, F, A, D C) Biographic data B) Reason for seeking care E) History of present concern F) Past health history A) Family health history D) Review of body systems

26. A client's recent complaints of polyuria have prompted a full diagnostic work-up for diabetes mellitus, including a nutritional assessment. To determine the client's body mass index (BMI), the nurse must know which of the following assessment parameters? Select all that apply. A) Gender B) Age C) Weight D) Waist circumference E) Height

C, E C) Weight E) Height

18. The nurse is preparing to assess an adult woman's activities related to health promotion and maintenance. Which question should the nurse ask to obtain the most objective and thorough assessment data? A) "Do you always wear your seatbelt when driving?" B) "How much beer, wine, or alcohol do you drink?" C) "Do you use condoms with each sexual encounter?" D) "Could you describe how you perform self-breast exams?"

D) "Could you describe how you perform self-breast exams?"

22. During the health interview of a client who has just learned that she is pregnant, the nurse is assessing the client's health history. What assessment question most directly addresses a known risk for congenital malformations? A) "Do you ever experience severe premenstrual symptoms?" B) "Do your menstrual periods tend to be irregular?" C) "Do you exercise on a regular basis?" D) "Do you have diabetes?"

D) "Do you have diabetes?"

10. Upon entering an exam room, the client states, "Well! I was getting ready to leave. My schedule is very busy and I don't have time to waste waiting until you have the time to see me!" Which response by the nurse would be most appropriate? A) "Our schedule is very busy also. We got to you as soon as we could." B) "No one is forcing you to be here, and you are free to leave at any time." C) "Would you like to report your complaints to someone with power?" D) "You're certainly justified in being upset, but I am ready to begin your exam now."

D) "You're certainly justified in being upset, but I am ready to begin your exam now."

7. When analyzing the nursing history recently taken on a client, which factor would most strongly alert the nurse to a significantly increased risk for chronic arterial insufficiency? A) Sedentary lifestyle B) A family history of arterial insufficiency C) Intake of 1 to 2 alcoholic drinks per day D) 14-year history of smoking a pack a day

D) 14-year history of smoking a pack a day

3. An adult client weighs 175 pounds and is 5 feet 6 inches tall. The nurse determines that the client's body mass index is which of the following? A) 12 B) 18 C) 25 D) 28

D) 28

24. An experienced nurse is aware that receding gums are an expected finding in some clients whereas in other clients this finding is abnormal. In which of the following clients would the nurse identify receding gums as an expected assessment finding? A) A 4-year-old girl who has all of her primary teeth B) A 20-year-old man who has type 1 diabetes mellitus C) A 39-year-old woman who has just finished a course of oral antibiotics D) A 77-year-old man who describes himself as being healthy

D) A 77-year-old man who describes himself as being healthy

24. A community health nurse is planning a health promotion campaign that will focus on cancer prevention. Which educational intervention should the nurse select in order to most influence participants' risks of head and neck cancers? A) Teaching about genetic screening B) A nutritional health program C) Teaching about monthly self-examination D) A smoking cessation program

D) A smoking cessation program

17. A nurse is assessing a client who is suspected to have optic atrophy. Which of the following assessment findings is most consistent with this diagnosis? A) Obscured retinal vessels B) No visible physiologic cup C) Increased appearance of the disc vessels D) A white appearance of the optic disc

D) A white appearance of the optic disc

22. The nurse assists a client into the dorsal recumbent position. Assessment of which area is contraindicated when the client is in this position? A) Chest B) Head C) Peripheral pulses D) Abdomen

D) Abdomen

6. A young adult male who comes to the emergency department complaining of abdominal pain for the past 3 days is suspected of having a ruptured appendix. The nurse auscultates the client's bowel sounds, noting them to be which of the following? A) Normoactive B) Hyperactive C) Hypoactive D) Absent

D) Absent

8. A nurse is conducting a workshop with a group of adults who are enrolled in a parenting class. Which of the following would the nurse emphasize as important in helping the school-age child achieve the psychosocial task of industry and avoid inferiority? A) Allow independence B) Encourage competition C) Increase socialization D) Acknowledge accomplishments

D) Acknowledge accomplishments

8. A nurse is planning to assess the instrumental functioning of a family. The nurse develops questions to focus on which of the following? A) Intimacy needs B) Communication patterns C) Beliefs about health D) Activities of daily living

D) Activities of daily living

27. Otoscopic examination of a 69-year-old client's tympanic membrane reveals that it is red, bulging, and distorted. The nurse also notes a diminished light reflex. To what should the nurse most likely attribute this assessment finding? A) Repeated ear infections B) Trauma C) Age-related changes D) Acute otitis media

D) Acute otitis media

29. The nurse's auscultation of a client's lung fields reveals the presence of a wheeze. The nurse should recognize that this adventitious sound results from what pathophysiological process? A) Air leaking from the alveoli into the pleural space B) Air being diverted from the trachea to the bronchi C) Air increasing in turbulence in a wide passage D) Air passing through constricted passageways

D) Air passing through constricted passageways

12. The nurse is assessing a client using the Glasgow Coma Scale following an acute hypoglycemic episode and obtains a score of 14. The nurse interprets this as indicating which of the following? A) Deep coma B) Coma C) Obtunded D) Alert and oriented

D) Alert and oriented

21. The nurse is assessing a 39-year-old woman who has a 20 pack-year history of cigarette smoking. When reviewing the client's current medication administration record, what drug would the nurse identify as increasing the woman's risk of stroke? A) Acetaminophen B) A beta-adrenergic blocker C) ASA D) An oral contraceptive

D) An oral contraceptive

16. After teaching a group of young women about breast self-examination, the nurse determines that the teaching was successful when the women state that they will palpate their breasts using which pattern? A) A circular pattern B) A clockwise pattern C) A random pattern D) An up-and-down pattern

D) An up-and-down pattern

3. The nurse is planning to assess a community. Which of the following activities would be most appropriate for the nurse to do when collecting objective data about a community? A) Interview several residents. B) Talk to key leaders. C) Observe residents' lifestyles. D) Analyze census data.

D) Analyze census data.

16. A nurse is preparing to examine a client's inguinal area. The nurse understands that this area is contained by which structure laterally? A) Symphysis pubis B) Inguinal ligament C) Inguinal canal D) Anterior superior iliac spine

D) Anterior superior iliac spine

28. An 88-year-old woman has been admitted to the acute medical unit for the treatment of a urinary tract infection that is thought to be progressing to urosepsis. When assessing the client's orientation, how should the nurse best gauge the client's orientation to time? A) Can you tell me approximately what time it is right now? B) Are you able to tell me today's date? C) Can you tell me the date and the day of the week? D) Are you able to tell the month and the year that we're in?

D) Are you able to tell the month and the year that we're in?

21. An older adult client has presented to the emergency department with signs and symptoms of dehydration. When assessing the client for risk factors that may have contributed to this condition, what question should the nurse prioritize? A) Do you use any over-the-counter dietary supplements? B) Are you familiar with the USDA's MyPlate recommendations? C) Have you ever been diagnosed with heart disease? D) Are you currently taking any diuretic medications?

D) Are you currently taking any diuretic medications?

21. A nurse is completing a general survey of a client's health and is beginning by measuring the client's vital signs. What assessment question constitutes the fifth vital sign? A) Can you tell me the date and month? B) Can I check your oxygen saturation level? C) Are you experiencing any shortness of breath? D) Are you having any pain right now?

D) Are you having any pain right now?

26. The nurse has assisted a 74-year-old woman from a chair to the examination table during an assessment, and the nurse observes that the client moves particularly slowly and stiffly. The nurse should question the client regarding a possible history of what health problem? A) Rhabdomyolysis B) Diabetes C) Kyphosis D) Arthritis

D) Arthritis

3. A client's health history reveals that she had a total hysterectomy at age 33 to treat severe endometriosis. She says that the surgeon also removed both ovaries and fallopian tubes. The nurse would interpret this as which of the following? A) Natural menopause B) Delayed menopause C) Premature menopause D) Artificial menopause

D) Artificial menopause

25. A nurse is validating assessment findings with a client, and the client proceeds to describe some of the psychological and spiritual components that she believes underlie her disease process. This understanding of the cause of illness is most closely associated with which of the following? A) Northern European cultures B) The Western biomedical model C) African-American culture D) Asian cultures

D) Asian cultures

25. The nurse is inspecting a new client's abdomen and notes the presence of a tight, distended abdomen and visible arterioles on the abdominal skin surface. How should the nurse proceed with assessment? A) Review the client's blood work for low platelets and hemoglobin. B) Assess the client for signs and symptoms of fluid volume overload. C) Assess the client's nutritional status. D) Assess the client for other signs and symptoms of liver disease.

D) Assess the client for other signs and symptoms of liver disease.

6. While examining a client's mouth, the nurse notes the presence of fasciculations (fine tremors) of the client's tongue. How should the nurse best respond to this assessment finding? A) Have the client provide a 24-hour diet recall. B) Review the client's medication regimen. C) Prepare the client for a thyroid screening. D) Assess the client's cranial nerve function.

D) Assess the client's cranial nerve function.

29. An older adult client has come to the clinic with new complaints of fatigue, constipation, and cold intolerance. This client may benefit from referral for which of the following purposes? A) Liver function testing B) Cognitive testing C) Lung function testing D) Assessment of thyroid function

D) Assessment of thyroid function

27. The nurse is assessing the genitalia and rectum of a 71-year-old client. When assessing the client's vagina, the nurse should know that age-related changes increase the client's risk of what abnormal finding? A) Trichomonas vaginitis B) Bacterial vaginosis C) Candidal vaginitis D) Atrophic vaginitis

D) Atrophic vaginitis

26. The nurse is auscultating a client's heart sounds and hears what she believes to be a murmur. How should the nurse proceed with gathering further assessment data related to the suspected murmur? A) Auscultate with the bell and then without the stethoscope. B) Ask the client to bear down (perform the Valsalva maneuver) while auscultating. C) Ask the client to inhale and exhale deeply while auscultating. D) Auscultate with the client in a variety of different positions.

D) Auscultate with the client in a variety of different positions.

16. A nurse is preparing to palpate a client's submental lymph nodes. At what anatomic location should the nurse position his or her hands? A) At the angle of the client's mandible B) At the base of the client's skull C) On the area behind the client's ears D) Behind the tip of the client's mandible

D) Behind the tip of the client's mandible

15. The nurse will palpate a client's axillae during a head-to-toe assessment. The nurse should combine this with examination of which area? A) Neck B) Anterior chest C) Heart D) Breasts

D) Breasts

15. While auscultating a client's trachea, the nurse hears a high, harsh sound with short inspiration and long expiration. The nurse would document which of the following? A) Vesicular breath sounds B) Bronchovesicular breath sounds C) Adventitious breath sounds D) Bronchial breath sounds

D) Bronchial breath sounds

4. When examining the eyes of an elderly client, the nurse observes a brownish discoloration of the lens. The nurse interprets this finding as being suggestive of what health problem? A) Conjunctivitis B) Presbyopia C) Glaucoma D) Cataracts

D) Cataracts

20. A nursing instructor is discussing cultural variations in the size of the thorax and impact on lung capacity. Which group would the instructor identify as typically having a larger thorax? A) African Americans B) Asian Americans C) Native Americans D) Caucasians

D) Caucasians

12. During the Romberg test, a client is unable to stand with the feet together and demonstrates a wide-based, staggering, unsteady gait. The nurse would interpret this finding as suggestive of which of the following? A) Spastic hemiparesis B) Parkinsonian gait C) Scissors gait D) Cerebellar ataxia

D) Cerebellar ataxia

13. The nurse is performing a respiratory assessment of a client who is palliative due to severe, uncompensated heart failure. What type of respiratory pattern should the nurse anticipate? A) Biot's B) Bradypnea C) Kussmaul's D) Cheyne-Stokes

D) Cheyne-Stokes

20. A cardiac care nurse works with a diverse client population. The nurse would assess a client from which cultural group for an increased effect of an antihypertensive medication? A) Eskimos B) Native Americans C) Hispanics D) Chinese

D) Chinese

12. The nurse is preparing a client for cancer chemotherapy treatment. While talking with the nurse, the client says, "Miracles do happen, and I'm praying for one." The nurse interprets this statement as suggesting which religious preference? A) Judaism B) Buddhism C) Islam D) Christianity

D) Christianity

16. On inspection, the nurse observes a line across the tip of an 8-year-old client's nose. The nurse should consequently focus on which area of assessment? A) History of abuse B) Chronic nose picking C) Mucosal polyps D) Chronic allergies

D) Chronic allergies

15. An older adult male client states that he has trouble cutting his toenails because they are hard and thick, and the nurse notes that they are very long and unkempt. Which system would be most important for the nurse to assess? A) Integumentary B) Digestive C) Neurologic D) Circulatory

D) Circulatory

5. The nurse is analyzing the data from the assessment of a client's heart and neck vessels. The client's first heart sound corresponds with what event in the cardiac cycle? A) Isometric contraction B) Closure of the semilunar valves C) Beginning of diastole D) Closure of the atrioventricular valves

D) Closure of the atrioventricular valves

8. A nurse is caring for a client who has been admitted with an infected venous ulcer. The nurse determines that the client will need medical interventions as well as nursing interventions. The nurse would identify which of the following? A) Actual nursing diagnosis B) Referral C) Risk nursing diagnosis D) Collaborative problem

D) Collaborative problem

1. A client has presented to the clinic for the treatment of an ovarian cyst. Which of the following would be most important for the nurse to do immediately before performing this woman's physical exam? A) Explain the purpose of the interview to the client. B) Construct the client's family genogram. C) Establish the client's reliability as historian. D) Collect necessary equipment essential to the exam.

D) Collect necessary equipment essential to the exam.

11. During the health history, a nurse asks a mother to describe the play activities of her school-age son. The mother reports activities that are typical for this age group. The nurse would document this as which type of play? A) Imitative B) Associative C) Parallel D) Competitive

D) Competitive

1. The nurse is assessing the eyes of a client who has a lesion of the sympathetic nervous system. What assessment finding should the nurse anticipate? A) Bilateral dilated pupils B) Nystagmus (involuntary eye movement) C) Argyll-Robertson pupils D) Constricted pupils, unresponsive to light

D) Constricted pupils, unresponsive to light

2. A client has sustained an injury to the cerebellum. Which area should be the nurse's primary focus for assessment? A) Vital signs B) Respiratory status C) Cardiac function D) Coordination

D) Coordination

24. During an eye assessment, the nurse is testing a client's visual acuity using a Snellen chart. In order to prepare the client for this component of assessment, what instruction should the nurse provide? A) I'm going to ask you to slowly walk forward until the last line of the chart become clear. B) Please stand at a comfortable distance from the chart and I'll get you to read each of the letters. C) Hold this chart and start to read out the letters after covering one of your eyes. D) Cover one of your eyes and then read out the letters on the chart, starting from the top.

D) Cover one of your eyes and then read out the letters on the chart, starting from the top.

22. A nurse is participating in an educational exercise in which she is conducting a self- examination of her own biases. This activity addresses what construct of cultural competence? A) Cultural desire B) Cultural knowledge C) Cultural skill D) Cultural awareness

D) Cultural awareness

13. When preparing to assess a client's thyroid gland, the nurse should ensure that which piece of equipment is readily available? A) Penlight B) Tongue depressor C) Centimeter-scale ruler D) Cup of water

D) Cup of water

11. The nurse is assessing a client whose chronic pain is poorly controlled. Which assessment finding should the nurse expect under these circumstances? A) Decreased heart rate B) Hypoglycemia C) Increased urinary output D) Decreased gastric motility

D) Decreased gastric motility

9. A nurse is working in a health care facility that uses charting by exception. Which of the following would the nurse expect to document? A) Liver palpation normal B) No tenderness on palpation C) Bowel sounds normoactive D) Decreased range of motion in right shoulder

D) Decreased range of motion in right shoulder

16. The nurse is assessing a newborn's neuromuscular maturity in light of the infant's known gestational age. Which of the following would the nurse expect to find if the newborn was premature? A) Flexed arms and legs B) Elbow position less than midline C) Heel distant from ear D) Delayed arm recoil

D) Delayed arm recoil

8. A parent of an ill infant states, We've gave him ibuprofen for a fever, and he had an allergic reaction. Which response would be most appropriate? A) Is he allergic to any other drugs? B) I will write that on his chart so he won't be given any. C) How often has he received ibuprofen? D) Describe what happens to him when he takes ibuprofen.

D) Describe what happens to him when he takes ibuprofen.

27. The school nurse is assessing a 15-year-old client. The nurse should understand that this child's current priorities will most likely reflect what developmental task? A) Exerting influence B) Learning new information C) Becoming productive D) Developing a personal identity

D) Developing a personal identity

7. A client has been to the clinic multiple times in the past year with vaginal infections, the most frequent of which was candidiasis. The nurse would assess the client for symptoms most likely related to which condition? A) Intestinal parasites B) Urinary tract infections C) Hypothyroidism D) Diabetes mellitus

D) Diabetes mellitus

3. A client complains of temporomandibular joint (TMJ) pain. Which of the following would the nurse most likely assess? A) Joint dislocation B) History of fracture C) History of dental abscess D) Difficulty chewing

D) Difficulty chewing

5. A client describes her frequent headaches as being severe and lasting for days. The client's positive response to what question would most clearly suggest to the nurse that these headaches are migraines? A) Do they occur after you have been tense or anxious? B) When you consume alcohol, do you get a headache? C) Do you have any eye symptoms, such as tearing? D) Do you have any visual changes before the headache?

D) Do you have any visual changes before the headache?

27. The nurse is assessing a middle-aged female client who is new to the clinic. The nurse observes the presence of significant facial hair that is uncharacteristic of the client's ethnicity. What assessment question should the nurse consequently ask? A) Has anyone in your family ever been diagnosed with skin cancer? B) Have you ever been assessed for diabetes? C) What dietary supplements do you usually take? D) Do you take steroid medications on a regular basis?

D) Do you take steroid medications on a regular basis?

1. During deep palpation of the client's abdomen, the nurse identifies a soft, nontender, solid mass extending 2 to 3 cm below the right costal margin. Which of the following actions would be most appropriate? A) Refer the client for medical follow-up. B) Evaluate further for a problem with the spleen. C) Assess urinary output. D) Document the position of the liver.

D) Document the position of the liver.

15. The nurse discovers a soft systolic murmur when auscultating the heart of a client at 32 weeks' gestation. Which action would be most appropriate? A) Refer her for cardiac evaluation. B) Ask another nurse to assess the heart. C) Inquire if the client has chest pain. D) Document this and continue to follow at future visits.

D) Document this and continue to follow at future visits.

29. The nurse has completed the objective and subjective assessment of a client who required care after an incident of intimate partner violence. How should the nurse document the client's injuries? A) Ask the client to write out a description of his or her injuries. B) Describe the location of the injuries in detailed prose. C) Make a video recording of the client's skin surfaces. D) Document using an injury map.

D) Document using an injury map.

24. A nurse on a medical unit is reviewing a client's electronic health record. What finding should suggest to the nurse that the client has a possible history of intimate partner violence? A) Documentation of an incomplete immunization history B) A documented history of bipolar disorder C) A lack of consistent primary health care during her pregnancies D) Documentation of three separate injuries during her pregnancies

D) Documentation of three separate injuries during her pregnancies

14. When reviewing the demographics of various communities, the nurse would identify a community with a large percentage of which demographic as most likely to have health-related concerns? A) Teenagers B) Young adults C) Middle-age individuals D) Elderly

D) Elderly

29. The nurse is experiencing challenges in eliciting information during the health interview of a 4-year-old boy. How can the nurse best foster communication with the child? A) Set a time limit for completing the interview. B) Ask the child to talk about himself in the third person. C) Explain the purpose of the interview in simple terms. D) Engage the child in play.

D) Engage the child in play.

28. A nurse will complete an initial comprehensive assessment of a 60-year-old client who is new to the clinic. What goal should the nurse identify for this type of assessment? A) Identify the most appropriate forms of medical intervention for the client. B) Determine the most likely prognosis for the client's health problem. C) Identify the status of the client's airway, breathing, and circulation. D) Establish a baseline for the comparison of future health changes.

D) Establish a baseline for the comparison of future health changes.

3. A school nurse who provides care in a middle school works exclusively with adolescents. According to Erikson's theory of psychosocial development, what task will underlie much of the students' behavior? A) Exerting influence over others B) Evaluating the merits of their parents' beliefs C) Appraising religious dogma D) Establishing a personal identity

D) Establishing a personal identity

14. The nurse is completing the assessment of a client who takes a beta-adrenergic blocker and a diuretic. Which assessment would be most important for the nurse to complete to ensure safety with a client receiving antihypertensive agents? A) Noting a widened pulse pressure B) Asking whether the client is experiencing headaches C) Assessing for a rise in blood pressure when standing D) Evaluating for orthostatic hypotension

D) Evaluating for orthostatic hypotension

6. When preparing to obtain information about a client's mental and psychosocial status, which of the following would the nurse need to do first? A) Question the patient about his or her usual lifestyle and behaviors. B) Perform a neurologic examination to determine any deficits. C) Check the client's level of consciousness for changes. D) Explain the purpose of the exam and types of questions.

D) Explain the purpose of the exam and types of questions.

9. A nurse asks a client to bring his hands together behind his head with his elbows flexed. The nurse is testing which of the following? A) Abduction B) Adduction C) Internal rotation D) External rotation

D) External rotation

25. A nurse is conducting a mental status assessment of a 70-year-old male client who is being treated for depression. When assessing the client's facial expression and eye contact, the nurse should consider which of the following? A) The nurse should inform the client that his facial expression is being assessed. B) Reduced eye contact is an age-related physiological change. C) Facial expression should be disregarded if the client has a diagnosed mental illness. D) Eye contact is strongly influenced by cultural norms.

D) Eye contact is strongly influenced by cultural norms.

25. A nurse is conducting an assessment of a client's eyes and vision and has completed the positions test. Following this test, the nurse will be able to document data that address what aspects of eye health? Select all that apply. A) Distant visual acuity B) Near visual acuity C) Accommodation D) Eye muscle strength E) Cranial nerve function

D) Eye muscle strength E) Cranial nerve function

11. The nurse is to collect a throat culture from a client who has signs and symptoms of a respiratory infection, including frequent, productive coughing. The nurse demonstrates the best adherence to standard precautions by using which of the following pieces of equipment? A) Eye goggles B) Face mask C) Cover gown D) Face shield

D) Face shield

23. A nurse is providing care for an 84-year-old client who has diagnoses of middle-stage Alzheimer disease and a femoral head fracture. What assessment tool should the nurse use to assess the client's pain? A) Graphic Rating Scale B) Numeric Rating Scale (NRS) C) Verbal Descriptor Scale D) Faces Pain Scale-Revised (FPS-R)

D) Faces Pain Scale-Revised (FPS-R)

4. Which of the following would be most important to plan and implement when a nurse notes a higher-than-expected teen birth rate in a particular community? A) Childbirth education classes B) Infant care education C) Parenting education classes D) Family planning education

D) Family planning education

1. During a client's vascular assessment, the nurse is palpating the pulse just under the client's inguinal ligament. The nurse is assessing which pulse? A) Temporal B) Brachial C) Popliteal D) Femoral

D) Femoral

9. The nurse is preparing to assess a client's apical impulse. The nurse should palpate at which location? A) Second intercostal space, left sternal border B) Third intercostal space, left axillary line C) Fourth intercostal space, left sternal border D) Fifth intercostal space, left midclavicular line

D) Fifth intercostal space, left midclavicular line

4. The nurse is assessing the anterior fontanelle of a 4-month-old infant brought to the clinic for a well-child exam. Which of the following would the nurse expect to assess? A) Sunken fontanelles B) Closed fontanelles C) Bulging fontanelles D) Flat fontanelles

D) Flat fontanelles

16. The nurse demonstrates the correct technique for assessing the psoas sign by which action? A) Applying deep palpation pressure to the client's right lower quadrant, then suddenly releasing B) Tapping fingerpads over the client's abdominal wall, feeling for a floating mass C) Percussing over the client's symphysis pubis with the client supine and then sitting upright D) Flexing the client's right hip, applying downward pressure on the right thigh

D) Flexing the client's right hip, applying downward pressure on the right thigh

17. A nurse plans to utilize formal and informal channels of communication that exist within a community. The nurse should identify which of the following as an example of informal communication? A) Newspaper B) Television C) Radio D) Fliers

D) Fliers

22. During the health interview, a client demonstrates the ability to describe healthy and unhealthy aspects of her thinking patterns. The nurse would conclude that this client has attained which level of development within Piaget's framework? A) Circular operational B) Preoperational C) Concrete operational D) Formal operational

D) Formal operational

28. The nurse is meeting the parents of an ill child for the first time and is preparing to perform the health interview. In addition to gathering health data, what additional goal should the nurse prioritize during this interaction? A) Gauge the parents' own levels of health. B) Emphasize the importance of adherence to treatment. C) Identify the family's socioeconomic status. D) Foster trust with the child's parents.

D) Foster trust with the child's parents.

10. While auscultating a client's lungs, the nurse notes the presence of adventitious sounds. Which of the following actions should the nurse do first? A) Refer the client for further medical evaluation. B) Auscultate for egophony. C) Perform bronchophony. D) Have the client cough, then listen again.

D) Have the client cough, then listen again.

15. A nurse is assessing an adult client's neck. Which of the following would be most appropriate when auscultating the client's thyroid gland for bruits? A) Hyperextend the client's neck. B) Turn the client's head to the right. C) Have the client swallow water. D) Have the client hold his or her breath.

D) Have the client hold his or her breath.

23. A clinic client's primary complaint is earache (otalgia). Consequently, the nurse's assessment is focusing on potential causes of the client's pain. What question should the nurse include in the health interview? A) What do you do for a living? B) Do you know if your vaccinations are up to date? C) Do you take over-the-counter medications or supplements? D) Have you been swimming lately?

D) Have you been swimming lately?

22. A client has presented to the emergency department (ED) with a lower leg laceration that she suffered while I was on a bender last night. The nurse recognizes the need to screen for alcohol use and will implement the CAGE questionnaire. What question will the nurse ask during this assessment? A) Have you ever experienced a memory blackout after drinking? B) Have you ever vomited blood after drinking alcohol? C) Have you ever been treated for alcohol abuse? D) Have you ever felt guilty about your alcohol use?

D) Have you ever felt guilty about your alcohol use?

22. A nurse is assessing a 49-year-old client who questions the nurse's need to know about sunburns he experienced as a child. How should the nurse best explain the rationale for this subjective assessment? A) Repeated sunburns in childhood may explain the presence of some of your moles. B) This is one of the assessments we use to determine whether your parents took good care of your skin when you were young. C) When you burn your skin as a child, it makes your skin more sensitive and slower to heal when you're older. D) Having bad sunburns when you're a child puts you at risk for skin cancer later in life.

D) Having bad sunburns when you're a child puts you at risk for skin cancer later in life.

7. During the assessment interview, the client made numerous statements that suggested his life generally exists in a state of harmony and balance. This fact would most likely prompt the nurse to identify which of the following? A) Actual nursing diagnosis B) Risk nursing diagnosis C) Collaborative problem D) Health promotion diagnosis

D) Health promotion diagnosis

24. The nurse is assessing a 69-year-old woman's risks for lung disease. The woman states, It shouldn't be a problem for me. My husband smokes quite heavily but I've been a lifelong nonsmoker. The nurse should recognize the need to teach the client about what topic? A) Strategies for making her husband quit smoking B) Genetic causes of lung cancer C) Age-related changes to respiratory function D) Health risks of secondhand smoke

D) Health risks of secondhand smoke

4. After creating a family genogram, the nurse evaluates it, usually prioritizing which of the following components? A) Psychosocial interaction problems between family members B) Communication patterns between generations C) Power structure within the family configuration D) Health-illness patterns through the generations

D) Health-illness patterns through the generations

5. During the health interview, a client tells the nurse that he can't breathe all that well at night when he is lying down and that this significantly disrupts his sleep. The nurse should assess this client further for which of the following health problems? A) Pneumonia B) Tuberculosis C) Bronchitis D) Heart failure

D) Heart failure

8. A client tells the nurse that the intravenous line must be placed in his right hand. Based on the nurse's understanding of the major religions, the nurse identifies this request as reflecting which of the following? A) Judaism B) Christianity C) Islam D) Hinduism

D) Hinduism

12. The nurse is presenting a class to a group of high school students about sexually transmitted infections. Which of the following should the nurse include as a major risk factor for cervical cancer? A) Gonorrhea B) Chlamydia C) Syphilis D) Human papilloma virus

D) Human papilloma virus

11. During a client's genitourinary exam, the nurse notes that the client's scrotum is enlarged and easily transilluminates. Which of the following should the nurse suspect? A) Tumor B) Hernia C) Varicocele D) Hydrocele

D) Hydrocele

26. A client has sought care because of the development of pruritic lesions between her toes, which the nurse suspects are attributable to a fungal etiology. How can the nurse best corroborate this suspicion? A) Test whether gentle abrasion with an emery board is painful. B) Apply hydrogen peroxide to see whether the client's pruritus is relieved. C) Perform a trial with a topical antibiotic. D) Illuminate the area using a Wood's light.

D) Illuminate the area using a Wood's light.

23. A community has experienced an increase in crime over the past several months. The nurse should recognize that this lack of safety may have what effect on residents? A) Decreased adherence to prescribed treatments by residents B) Increased resilience by residents C) Decreased educational opportunities D) Increased psychosocial stress for residents

D) Increased psychosocial stress for residents

1. When assessing a newborn, the nurse observes that the infant's hands and feet are bluish in color. The nurse interprets this finding as being suggestive of which of the following? A) Cardiopulmonary dysfunction B) Peripheral vascular disease C) Acidñbase imbalance D) Ineffective temperature regulation

D) Ineffective temperature regulation

8. The nurse is assessing a client's heart and neck vessels. Which technique would be most appropriate to use when examining the client's jugular venous pulse? A) Perform the exam with the client in a supine position. B) Have the client look straight ahead with chin slightly lifted. C) Have the client sit up at a 90-degree angle. D) Inspect the suprasternal notch or around the clavicles.

D) Inspect the suprasternal notch or around the clavicles.

13. While performing an elderly client's admission assessment, the nurse notes the presence of deep tongue fissures. Which of the following responses should take priority? A) Anterior-posterior and lateral chest x-ray B) Complete blood count with differential C) Dietitian referral D) Intravenous fluid replacement

D) Intravenous fluid replacement

4. When describing the importance of documenting initial assessment data to a group of new nurses, which of the following would the nurse emphasize as the primary reason? A) Health care institutions have established policies regarding documentation. B) Incorrect conclusions may be made without documentation of the nurse's opinions. C) It satisfies legal standards established by health care organizations and institutions. D) It becomes the foundation for the entire nursing process.

D) It becomes the foundation for the entire nursing process.

7. The nurse is completing an initial assessment of a client who is new to the ambulatory clinic. Before assessing the client's blood pressure, a nurse asks him what his usual blood pressure is. The nurse bases this action primarily on what rationale? A) It provides identifiable data about the client. B) It verifies the client's cardiac function. C) It assesses the client's distant memory recall. D) It indicates the client's involvement in his health care.

D) It indicates the client's involvement in his health care.

5. A nurse educator is presenting an in-service program to a group of nurses who will be working on an oncology unit. Which of the following characteristics of cancer pain should the nurse describe? A) Its basis is usually chronic neuropathy. B) It is most often caused by a specific recent trauma. C) It usually appears in the first month after cancer develops. D) It is typically caused by compressed peripheral nerves.

D) It is typically caused by compressed peripheral nerves.

8. A client is receiving an intradermal injection to evaluate general immunity during a nutritional assessment. Which of the following conclusions is suggested if the client has no reaction? A) It indicates high cholesterol and triglyceride levels. B) It shows a sacrifice of skeletal muscle proteins and blood proteins. C) It is indicative of unhealthy dietary habits. D) It may be immunosuppression resulting from undernourishment.

D) It may be immunosuppression resulting from undernourishment.

11. When assessing an older adult client with osteoporotic thinning and vertebral collapse, which of the following findings would the nurse expect to identify? A) Lordosis B) Increased arm swing C) Narrowed gait D) Kyphosis

D) Kyphosis

8. When assessing a rural community, the nurse would most likely identify which of the following as a major factor hindering access to health care? A) Lack of social service agencies B) Sporadic police protection C) Presence of low-income housing D) Lack of public transportation

D) Lack of public transportation

27. The nurse is auscultating a client's abdomen and is unable to discern any bowel sounds. How should the nurse proceed with assessment? A) Repeat auscultation in four to six hours. B) Palpate the client's abdomen to stimulate bowel motility. C) Perform abdominal percussion, wait three to five minutes and then repeat auscultation. D) Listen for at least five minutes before documenting an absence of bowel sounds.

D) Listen for at least five minutes before documenting an absence of bowel sounds.

1. A nurse on a postsurgical unit is admitting a client following the client's cholecystectomy (gall bladder removal). What is the overall purpose of assessment for this client? A) Collecting accurate data B) Assisting the primary care provider C) Validating previous data D) Making clinical judgments

D) Making clinical judgments

12. An older adult client has been admitted to the hospital with failure to thrive resulting from complications of diabetes. Which of the following would the nurse implement in response to a collaborative problem? A) Encourage the client to increase oral fluid intake. B) Provide the client with a bedtime protein snack. C) Assist the client with personal hygiene. D) Measure the client's blood glucose four times daily.

D) Measure the client's blood glucose four times daily.

13. The nurse is preparing to lead a health promotion activity among a group of clients from different cultures. The nurse would expect that which client would require the least amount of personal space? A) Latin American B) Asian C) American D) Middle Easterner

D) Middle Easterner

29. A male client has presented for follow-up to a diagnosis of genital warts. The nurse should expect to assess for what type of lesions? A) Reddened ulcers that occasionally bleed B) Pimple-like vesicles C) Firm, shiny nodules D) Moist, fleshy papules

D) Moist, fleshy papules

18. When palpating a female client's axillae, which finding would the nurse document as normal? A) Node size is 1.2 cm. B) Nodes are fixed. C) Nodes are hard. D) Nodes are discrete.

D) Nodes are discrete.

20. After teaching a group of students about spirituality and religion, the instructor determines that the students need additional teaching when a student states which of the following? A) Spirituality and religion are important factors that can affect health decisions and outcomes. B) Religion and spirituality are separate and distinct, but interrelated concepts. C) There has been a tremendous growth in the understanding of spirituality in the past 20 years. D) Nursing has only recently begun to incorporate spirituality into client care.

D) Nursing has only recently begun to incorporate spirituality into client care.

18. A nurse is performing an eye and vision assessment on a client who has an inner ear disorder. This disorder may contribute to what finding during the client's eye positions test? A) Strabismus B) Phoria C) Tropia D) Nystagmus

D) Nystagmus

20. A nurse is preparing to complete a comprehensive assessment on a client. When collecting objective data, which of the following should the nurse do first? A) Assess the client's vital signs. B) Take the client's body measurements. C) Assess the client's mental status. D) Observe the client's overall appearance.

D) Observe the client's overall appearance.

8. A nurse has identified the goal of becoming more culturally sensitive and competent. What is the primary rationale for cultural sensitivity in health care settings? A) Recognize that cultural diversity exists. B) Understand individual differences. C) Prevent offending the client. D) Obtain accurate assessment data.

D) Obtain accurate assessment data.

2. The nurse is performing an assessment of a client admitted to the emergency department in status asthmaticus. The nurse should carefully inspect which part of the body in an effort to differentiate central cyanosis from peripheral cyanosis? A) Nail beds B) Sclerae C) Palms D) Oral mucosa

D) Oral mucosa

27. The nurse is assessing the sinuses of a client who exhibits many of the clinical characteristics of sinusitis. When percussing the client's sinuses, what assessment finding would most strongly suggest sinusitis? A) Resonance on percussion B) Dull sounds C) Tympanic sounds D) Pain on percussion

D) Pain on percussion

24. The nurse is assessing the carotid arteries of a client with a history of heart disease. What action should the nurse perform during this assessment? A) Palpate the client's left and right carotid arteries simultaneously. B) Palpate the client's carotid arteries prior to auscultation. C) Instruct the client to inhale and exhale forcefully during auscultation. D) Palpate the client's carotid arteries gently if an occlusion is audible.

D) Palpate the client's carotid arteries gently if an occlusion is audible.

26. The nurse is assessing a pregnant client and is performing Leopold's maneuvers. For the first two maneuvers, the nurse will perform which of the following actions? A) Palpate the client's midline abdomen and then the region of the symphysis pubis. B) Palpate the client's abdomen beginning with the left flank and then moving to the right flank. C) Palpate the client's floating ribs and then gradually palpate to the level of the ischial spines. D) Palpate the client's fundal region and then the lateral sides of the abdomen.

D) Palpate the client's fundal region and then the lateral sides of the abdomen.

24. The nurse is integrating the principles of Bowen Family System theory during interactions with a family. What principle underlies this theory? A) The role of younger family members supersedes that of older members. B) Every family is equally functional, although in different ways. C) Family identity is inseparable from ethnicity. D) Patterns of relating tend to repeat over generations.

D) Patterns of relating tend to repeat over generations.

6. A young female client refuses treatment for a sexually transmitted infection. The nurse explains that lack of treatment may put her at risk for which condition? A) Endometriosis B) Urinary tract infection C) Cervical cancer D) Pelvic inflammatory disease

D) Pelvic inflammatory disease

11. A nursing educator is evaluating a colleague's examination of a client's thyroid gland. The educator would determine that the nurse needs additional instruction when the nurse demonstrates which technique? A) Inspection B) Auscultation C) Palpation D) Percussion

D) Percussion

24. A client has illuminated his call light and tells the nurse that he is having ten out of ten pain. The nurse's initial inspection reveals that the client is watching videos on his tablet computer and appears to be at ease physically and emotionally. How should the nurse validate the client's subjective complaint of pain? A) Ask the client to repeat his rating of his pain. B) Observe the client for several seconds to see if his demeanor or his behavior changes. C) Consult the client's medication administration record (MAR) to check for recent analgesic use. D) Perform further assessments addressing various aspects of the client's pain.

D) Perform further assessments addressing various aspects of the client's pain.

7. In which health condition would the nurse most likely expect to assess a capillary refill time that is longer than 2 seconds? A) Psoriasis B) Multiple sclerosis C) Malignant melanoma D) Peripheral vascular disease

D) Peripheral vascular disease

12. During a client's eye assessment, the nurse is testing for consensual pupillary constriction. Which technique should the nurse implement? A) Hold a pencil about 12 inches from the tip of the nose. B) Use an ophthalmoscope to inspect the inner eye. C) Shine a light directly into one eye of the client. D) Place a barrier between the client's eyes.

D) Place a barrier between the client's eyes.

30. The nurse is preparing to auscultate a client's lungs after completing thoracic inspection, palpation, and percussion. How should the nurse best prepare for this assessment technique? A) Keep the client's shirt or gown in place to maintain privacy. B) Begin with the bell of the stethoscope on the client's anterior chest. C) Tell the client that you will be asking him or her to breathe as quickly and deeply as possible. D) Place the diaphragm on the client's posterior chest wall.

D) Place the diaphragm on the client's posterior chest wall.

26. While assessing the health of a client's respiratory system, the nurse is palpating for fremitus. What instruction should the nurse provide to the client during this component of assessment? A) When I say so, please exhale forcefully and hold the breath. B) Say the letter 'e' and keep saying it until I tell you to stop. C) Breathe in as deeply as you can and hold your breath until I say. D) Please say the number 'ninety-nine' for me.

D) Please say the number 'ninety-nine' for me.

29. A nurse has taught a group of older adults about the high incidence and prevalence of macular degeneration. What health promotion and prevention activity should the nurse encourage these clients to perform? A) Obtain a home version of the Snellen chart and test their vision weekly B) Rinse their eyes with a warmed, normal saline solution three to four times per week C) Maintain a low-sodium diet D) Post an Amsler grid in their home and perform the test on a regular basis

D) Post an Amsler grid in their home and perform the test on a regular basis

6. A 66-year-old client states that he has increasing difficulty hearing high-pitched sounds. The patient's statement most likely suggests that he has what diagnosis? A) Vertigo B) Otalgia C) Tinnitus D) Presbycusis

D) Presbycusis

26. The nurse is examining a client's breasts and notes the presence of pronounced dimpling. How should the nurse best respond to this assessment finding? A) Confirm whether the client has breast implants in place. B) Ask the client about any history of mastitis (breast infection). C) Explain to the client that this is a normal, age-related change. D) Promptly refer the client for further medical assessment.

D) Promptly refer the client for further medical assessment.

4. Which finding would require further evaluation or referral when auscultating heart sounds on an 8-year-old client during a routine physical exam? A) Audible S3 B) Soft systolic murmur C) Sinus arrhythmia D) Pulse rate 120 beats per minute

D) Pulse rate 120 beats per minute

28. The nurse's assessment of an infant reveals a positive Barlow's sign. What collaborative problem should the nurse consequently identify? A) RC: Failure to thrive B) RC: Jaundice C) RC: Patent ductus arteriosus D) RC: Hip displacement

D) RC: Hip displacement

30. Assessment of a client's breasts reveals tenderness on palpation and diffuse redness. What collaborative problem is most clearly suggested by these data? A) RC: Breast cancer B) RC: Benign breast disease C) RC: Hematoma D) RC: Infection

D) RC: Infection

10. A nurse is preparing to document a collaborative problem for a client with pain. Which of the following would be most appropriate? A) Impaired physical mobility related to chronic pain B) Risk for powerlessness related to chronic pain C) Readiness for enhanced comfort level D) RC: peripheral nerve compression

D) RC: peripheral nerve compression

23. The nurse is performing an assessment of a hospital client at the beginning of a shift. When assessing the client's heart rate, the nurse will most likely palpate what artery? A) Femoral artery B) Aorta C) Ulnar artery D) Radial artery

D) Radial artery

6. When the nurse is examining a male client's genitalia, the client experiences an erection. Which of the following would be most appropriate for the nurse to do? A) Remain silent but continue the examination. B) Stop the exam and leave the room for a few minutes. C) Ask the client whether continuing the exam will embarrass him. D) Reassure the client that this is not unusual.

D) Reassure the client that this is not unusual.

25. The nurse's auscultation of a 22-year-old client's apical heart rate reveals the presence of S3. When the client stands upright, the S3 is no longer audible. How should the nurse respond to this assessment finding? A) Make a referral to the client's primary care provider promptly. B) Perform a focused respiratory assessment. C) Recognize this as an early sign of left-sided heart failure. D) Recognize this as a normal assessment finding in this client.

D) Recognize this as a normal assessment finding in this client.

7. Which action would be most appropriate when a nurse assesses the umbilical cord of a 4-day-old infant and finds it to be dried and black? A) Notify the newborn's physician. B) Apply warm compresses. C) Apply an antibiotic ointment. D) Recognize this as normal.

D) Recognize this as normal.

19. While interviewing a family, the nurse questions the family about external systems. The nurse would collect information about which of the following? A) Patterns of affection B) Shared activities C) Decision-making patterns D) Recreational organizations

D) Recreational organizations

9. The nurse is assessing the breasts of a Caucasian woman who has just been diagnosed with Paget disease. Which of the following would the nurse expect to find? A) Orange-peel skin B) Nipple retraction C) Dark pink areola D) Red and scaling on the areola

D) Red and scaling on the areola

14. During a well-child visit, a parent asks the nurse the best way to manage negativism in her toddler. Which suggestions by the nurse would be most appropriate? A) Implement punishment appropriate for the child's age. B) Spend more quality time with the child. C) Repeatedly tell the child not to always say no. D) Reduce the opportunities for a no answer.

D) Reduce the opportunities for a no answer.

12. The results of a client's Rinne test suggest that bone conduction and air conduction are both reduced. Which of the following would be most appropriate? A) Perform a Romberg test. B) Take a swab of the client's tympanic member. C) Repeat the test in 5 to 10 minutes. D) Refer the client for further evaluation.

D) Refer the client for further evaluation.

26. A 2-year-old girl has been brought to the ambulatory clinic by her mother who states, She's put a pea in her ear, and I think she did it 2 days ago because that was the last time we ate them. The nurse's otoscopic examination confirms the presence of this foreign body in the girl's middle ear. How should the nurse best respond to this assessment finding? A) Attempt to remove the pea using sterile forceps. B) Irrigate the ear canal with warm tap water to remove the pea. C) Instruct the mother to watch the girl's ear closely and return for care if it does not fall out in the next few days. D) Refer the girl to her primary care provider for prompt removal of the pea.

D) Refer the girl to her primary care provider for prompt removal of the pea.

19. The nurse is assessing the various lobes of the client's lungs. To gather accurate data, the nurse must assess which lobe anteriorly? A) Left upper lobe B) Left lower lobe C) Right upper lobe D) Right middle lobe

D) Right middle lobe

13. A nurse is preparing to palpate a client's spleen. Which position should the nurse use to best facilitate palpation? A) Sitting upright B) Prone C) Semi-Fowler's D) Right side-lying

D) Right side-lying

27. The nurse's assessment of a client with a decreased level of consciousness reveals that the client is incontinent of urine. During the process of data analysis, the nurse would be justified in identifying what risk nursing diagnosis? A) Risk for Injury related to urinary incontinence B) Risk for Infection related to urinary incontinence C) Risk for Bowel Incontinence related to urinary incontinence D) Risk for Impaired Skin Integrity related to urinary incontinence

D) Risk for Impaired Skin Integrity related to urinary incontinence

1. The nurse is assessing a fair-skinned, Caucasian woman with red hair and freckled skin. During health promotion, the nurse should focus education on which of the following topics? A) Management of dry skin B) Susceptibility to bruising C) Risks of fungal infections D) Risks of sun exposure

D) Risks of sun exposure

17. When considering the various cultural aspects associated with death rituals, which of the following should guide a nurse's practice? A) Most cultures have similar durations for the length of time a person grieves. B) A person's view of death is likely to be different from the original ethnic group's practice. C) Responses to death and grief are fairly consistent among different cultures. D) Rituals for burial and bereavement are likely to reflect original cultural practices.

D) Rituals for burial and bereavement are likely to reflect original cultural practices.

4. An older adult female client is concerned because her skin is very dry. She asks the nurse why she has dry skin now when she never had dry skin before. The nurse responds to the client based on the understanding that dry skin is normal with aging due to a decrease of what? A) Squamous cells B) Sweat glands C) Subcutaneous tissue D) Sebum production

D) Sebum production

5. A nurse is working in a clinic in a low-income neighborhood and assesses a female adult client who states that she has a urinary tract infection. The nurse notes that the client is unkempt, wearing stained clothing, and has a strong body odor. The client mentions that she was evicted from her apartment two weeks ago. Which nursing diagnosis would the nurse most likely identify for this client? A) Caregiver role strain related to fatigue B) Impaired skin integrity related to neurologic deficits C) Deficient fluid volume related to possible urinary tract infection D) Self-care deficit related to possible homelessness

D) Self-care deficit related to possible homelessness

11. The nurse is preparing to inspect a woman's breasts for retraction and dimpling. Which position would be most appropriate? A) Standing B) Supine C) Semi-Fowlers D) Sitting

D) Sitting

23. Assessment reveals that a young adult has failed to achieve Erikson's central task of his current stage of development. What nursing diagnosis would the nurse associate most closely with this finding? A) Risk for compromised human dignity B) Anxiety C) Ineffective sexuality pattern D) Social isolation

D) Social isolation

14. The school nurse assesses unequal shoulder and scapula height in an adolescent. Which of the following should the nurse assess next? A) Lateral aspect of the thorax B) Lung volume C) Hip levels D) Spinal column

D) Spinal column

25. A client describes herself as "dumbfounded" that she has been diagnosed with cancer, stating, "I had such a clear vision from God that I was negative for cancer. Now I have no idea what I can trust." This client's statement is suggestive of what nursing diagnosis? A) Ineffective role performance B) Complicated grieving C) Social isolation D) Spiritual distress

D) Spiritual distress

15. A nurse is determining whether the data for a client support a potential nursing diagnosis. The nurse is most likely engaged in which step in the diagnostic reasoning process? A) Step Three: Draw Inferences B) Step Four: Propose Possible Nursing Diagnoses C) Step Five: Check for Defining Characteristics D) Step Six: Confirm or Rule Out Diagnoses

D) Step Six: Confirm or Rule Out Diagnoses

12. During palpation of the client's abdomen, the nurse feels a prominent, nontender, pulsating 6-cm mass above the umbilicus. What action should the nurse take? A) Refer the client to an oncologist. B) Provide a dietician consult for the client. C) Counsel the client regarding hernia repair. D) Stop palpating and get medical assistance.

D) Stop palpating and get medical assistance.

25. A nurse is preparing to begin a community assessment. The nurse should begin with which of the following activities? A) Visit the hospitals in the community. B) Enlist the help of nurses in the community. C) Draft preliminary health goals for the community. D) Study the history of the community.

D) Study the history of the community.

21. The nurse is preparing to perform a comprehensive assessment of a client who has a diagnosis of Alzheimer's disease. How should the nurse accommodate the client's cognitive deficit when obtaining the client's health history? A) Obtain the client's history from the electronic health record and proceed with physical assessment. B) Focus the assessment on aspects of the client's history that he is able to accurately describe. C) Perform the assessment as quickly as possible in order to minimize the client's stress. D) Supplement the client's statements with data from the client's friends and family.

D) Supplement the client's statements with data from the client's friends and family.

25. Scar tissue is visible on the perineum of an adult female client. The nurse should consequently question the client about which of the following? A) Surgical correction of a rectocele B) History of sexually transmitted infections C) History of sexual abuse D) Tearing during vaginal delivery

D) Tearing during vaginal delivery

12. The nurse is assessing an older adult's psychosocial development with reference to Freud's theory of development. What observation by the nurse would most clearly suggest healthy development within this theoretical framework? A) The client is able to describe challenges that he has overcome. B) The client has eliminated conflictual relationships from his life. C) The client is able to delegate care to others when necessary. D) The client appears to have dealt effectively with recent losses.

D) The client appears to have dealt effectively with recent losses.

26. The nurse is conducting a musculoskeletal assessment of an older adult client. What aspect of the client's medical history requires the nurse to alter the usual sequence or content of this assessment? A) The client takes medications to treat hypertension. B) The client suffered a fractured humerus 1 year earlier. C) The client has a diagnosis of type 1 diabetes. D) The client had a total hip replacement 2 years ago.

D) The client had a total hip replacement 2 years ago.

29. Data analysis of assessment data from a client who presented to the emergency department has resulted in the nurse making a syndrome nursing diagnosis. What is a primary characteristic of this type of diagnosis? A) The client's health problem cannot be conveyed using standard nursing language. B) The client's current signs and symptoms are the result of a longstanding health problem. C) The client has health problems that will require multidisciplinary care. D) The client has a number of nursing diagnoses that typically occur together.

D) The client has a number of nursing diagnoses that typically occur together.

6. A nurse is planning a client's care following the completion of an initial assessment. When formulating a risk nursing diagnosis, which piece of data would be most useful? A) The client has an elevated white blood cell count. B) The client is 66 years of age. C) The client has pain in her joints, especially in the morning. D) The client is separated from her usual social supports.

D) The client is separated from her usual social supports.

20. The nurse has identified abnormal findings when reviewing a young adult client's health history. Within Kohlberg's theory of psychosocial development, what behavioral characteristic is the nurse most likely to observe? A) The client has difficulty trusting others. B) The client is easily manipulated by others. C) The client is unable to weigh options when presented with a dilemma. D) The client makes decisions without considering the impact on others.

D) The client makes decisions without considering the impact on others.

8. An 18-year-old woman complains because one breast is larger than the other. What additional interview data would suggest a need for referral? A) The client states that she is sexually active. B) The client states that she does not perform breast self-examination. C) The client states that her problem affects her body image. D) The client states that this represents a sudden change in her breast size.

D) The client states that this represents a sudden change in her breast size.

16. The nurse's interview with an older adult client reveals that he bitterly regrets some of the financial decisions that he made when he was younger. The nurse recognizes that unless the client is able to accept these undesirable aspects of life, what outcome is likely? A) The client will adopt antisocial behaviors late in life. B) The client will die prematurely. C) The client will gradually abandon significant relationships. D) The client will live with despair during his final years of life.

D) The client will live with despair during his final years of life.

7. The nurse is using the Verbal Descriptor Scale to assess a client's pain. The nurse will prioritize which of the following data? A) The client's facial expressions B) The client's report on a 0 to 10 numeric scale C) The client's rating on a 0 to 10 visual analog scale D) The client's explanation of how her pain feels

D) The client's explanation of how her pain feels

24. The nurse is performing an assessment of a client's musculoskeletal system. The nurse should begin the assessment by examining which of the following? A) The client's leg length B) The client's lateral bending ability C) The client's cervical ROM D) The client's gait

D) The client's gait

30. A nurse is obtaining subjective data from an adult client who is new to the clinic. The nurse has asked the client, "Where do you usually turn for help in a time of crisis?" What domain is this nurse assessing? A) The client's family relationships B) The client's current level of social and relational stability C) The client's critical thinking and problem-solving abilities D) The client's stress management and coping strategies

D) The client's stress management and coping strategies

23. The nurse's inspection of a client's extremities reveals a deep, circular, painful wound on the client's great toe. What should the nurse suspect as the etiology of the client's wound? A) Blood is returning from the client's toe more slowly than normal. B) There is a blockage or infection in the client's lymphatic system. C) There is a disruption in osmotic pressure in the client's extremities. D) The client's toe is receiving an inadequate supply of blood.

D) The client's toe is receiving an inadequate supply of blood.

17. A client has presented for care because of frequent sinus headaches. During transillumination of the frontal sinuses, a red glow is noted. The nurse should anticipate which of the following? A) The physician will write a prescription for antibiotics. B) The drainage will need to be cultured. C) A repeat procedure will be done in 1 week to compare findings. D) The headaches are most likely not from a sinus infection.

D) The headaches are most likely not from a sinus infection.

22. The nurse is assessing a 2-day-old infant prior to discharge home from the hospital with his mother. When assessing the infant's eyes, what finding would the nurse consider to be abnormal? A) The infant is unable to follow a moving object or light. B) The infant's periorbital area is slightly edematous. C) The infant's pupils react to light. D) The infant's sclerae have a yellowish tint.

D) The infant's sclerae have a yellowish tint.

30. A nurse is preparing for an assessment by reviewing a new client's electronic health record, which documents the presence of macules on the client's left flank and mid-back regions. The nurse should recognize what characteristic of these skin lesions? A) The lesions will be raised and have irregular borders. B) The lesions will be acutely painful. C) The lesions will produce eschar. D) The lesions will not be palpable.

D) The lesions will not be palpable.

26. A small, rural hospital is revising the policies and procedures surrounding documentation in an effort to align practices with the Health Information Technology for Economic and Clinical Health (HITECH) Act. How can the requirements of this legislation best be met? A) Expand the use of the Nursing Minimum Data Set. B) Eliminate the use of verbal handoffs between nurses. C) Increase interdisciplinary collaboration in the hospital. D) Increase the use of electronic health records (EHRs) in the hospital.

D) The man had an inguinal hernia repair in 2008.

28. The data obtained during a nurse's family assessment suggest that the mother in the family is enmeshed with the youngest son, who is a middle-aged adult. The nurse should recognize what implication of this fact? A) The son and the mother have the highest priority relationship in the family. B) The son is inferior to the mother. C) The son and mother cannot form relationships with others. D) The son and the mother cannot self-differentiate.

D) The son and the mother cannot self-differentiate.

21. A nurse has completed a community assessment and is analyzing the availability of agency services. What assessment finding would suggest that agencies provide sufficient services in the community? A) Agencies are limited to tasks that cannot be performed in a hospital or institutional setting. B) Primary care facilities perform many of the functions that are typically performed by agencies in other jurisdictions. C) Agencies are primarily faith-based. D) There is a combination of several different public agencies and volunteer agencies.

D) There is a combination of several different public agencies and volunteer agencies.

15. During a Weber test, the client reports lateralization of sound to the good ear. How should the nurse interpret this assessment finding? A) The good ear cannot receive sound vibrations. B) There is a dysfunction of the middle ear. C) The poor ear is receiving sound vibrations by air. D) There is a sensorineural hearing impairment.

D) There is a sensorineural hearing impairment.

26. A nurse is working with a 22-year-old woman of Asian ethnicity who has been diagnosed with bipolar disorder. When planning culturally appropriate care, the nurse should consider which of the following? A) There may a lack of acceptance that the client's behavior is abnormal. B) The client's family may see her illness as punishment for misdeeds. C) The client's family may see her psychiatric disorder as evidence of bad character. D) There may be shame associated with having a psychiatric disorder.

D) There may be shame associated with having a psychiatric disorder.

13. When assessing an elderly client's hip joint after a fall, which of the following should lead the nurse to suspect that the client has a hip fracture? A) Internal rotation of the affected leg B) Abduction of the affected leg C) Partial weight bearing D) Thigh pain

D) Thigh pain

9. A client at 34 weeks' gestation is lying on an examination table while the nurse asks questions. The client says she is feeling dizzy. What intervention by the nurse would be most appropriate? A) Measure orthostatic blood pressures. B) Have her get up and walk around. C) Provide her with a glass of juice. D) Turn her on her left side.

D) Turn her on her left side.

3. A nurse is palpating a client's epitrochlear nodes. The nurse is palpating which area? A) Posterior neck B) Axillary area C) Inguinal area D) Upper arm

D) Upper arm

15. A client has a documented history of hepatomegaly (liver enlargement), and the nurse recognizes the need to perform deep palpation during the physical assessment. The nurse should perform which of the following actions? A) Use one hand and depress the skin 1 centimeter. B) Use the dominant hand to depress the skin one-half to three-quarters of an inch. C) Use both hands to depress the skin one-half of an inch. D) Use both hands to depress the skin 1 to 2 inches.

D) Use both hands to depress the skin 1 to 2 inches.

10. When assessing cranial nerves IX and X, which of the following would the nurse consider as a normal finding? A) Stationary soft palate on phonation B) Deviation of uvula when client says ah C) Asymmetrical soft palate D) Uvula and soft palate rising bilaterally

D) Uvula and soft palate rising bilaterally

28. A nurse is providing care to a client who has been in a motor vehicle accident and who has facial lacerations and a pelvic fracture. How can the nurse best determine the reliability and accuracy of data obtained during a pain assessment? A) Ask the primary care provider to validate the assessment data. B) Compare the findings to the client's preinjury level of health. C) Compare the findings to the most recent previous pain assessment. D) Validate the assessment data with the client.

D) Validate the assessment data with the client.

13. A nurse is comparing the subjective data and objective data obtained from an assessment of a client who is thought to have hepatitis A. This nurse's comparison will achieve what benefit to this client's care? A) Formulation of nursing diagnoses B) Identification of missing data C) Determination of documentation form to use D) Validation of data

D) Validation of data

10. The nurse assesses a client and palpates a temporal artery that is hard, thick, and tender with absent pulsations. The nurse would gather additional information related to which aspect of health? A) Mental status B) Hearing C) Neurologic status D) Vision

D) Vision

10. The nurse is preparing to perform a physical examination on a female client who has been transferred to the medical unit from the emergency department. The nurse should begin the collection of objective data with which of the following examinations? A) Head and neck examination B) Palpation of lymph nodes C) Breast examination D) Vital signs

D) Vital signs

28. The nurse is reviewing an older adult's recent laboratory values prior to performing a physical assessment. What value would most clearly indicate the need for further nutritional assessment? A) Hemoglobin 12.2 g/dL B) Hematocrit 40% C) Serum albumin 3.9 g/dL D) Vitamin B12 91 g/ml

D) Vitamin B12 91 g/ml

8. The nurse is assessing a client's cultural identity and affiliation during the health interview. How best can the nurse elicit this information? A) What are your race and culture? B) Would you describe yourself as American? C) How would you describe your cultural values? D) With which cultural group do you most closely identify?

D) With which cultural group do you most closely identify?

9. A decrease in tongue strength is noted on examination of a client. The nurse interprets this as indicating a problem with which cranial nerve? A) III B) VI C) VIII D) XII

D) XII

29. The nurse is integrating health promotion education into the assessment of a client's heart and neck vessels. What teaching point addresses the most significant risk factor for coronary artery disease? A) If you can eliminate red meat from your diet, your risk of heart disease will drop significantly. B) Try to ensure that you're screened for heart disease at least once every six months. C) Anything that you can do to reduce stress in your life will benefit your heart health. D) Your risk for heart disease will drop greatly if you're able to stop smoking.

D) Your risk for heart disease will drop greatly if you're able to stop smoking.


Conjuntos de estudio relacionados

Business and Society Quiz Chapter 16

View Set

French - Who, What, When, Where, Why, and How

View Set

Chapter 2: The Chemistry of Life

View Set

FNP Board Practice Questions - PART 2

View Set